Você está na página 1de 170

CASEBOOK 2021

Clube de Consultoria da
Universidade Federal do Ceará
Bem-Vindo(a) ao Casebook do CCUFC!
É com muito orgulho que apresentamos o Casebook CCUFC 2021. Esse material é um compilado de vários projetos
desenvolvidos pelo Clube de Consultoria da UFC (CCUFC) e tem o objetivo de ajudar aqueles que desejam melhorar a
capacidade de resolução de cases (resolução de problemas de negócio), os quais são cobrados nos processos seletivos das
empresas de consultoria mais renomadas do mundo.

O CCUFC é um projeto de extensão formado, principalmente, por alunos da Universidade Federal do Ceará que busca
difundir a cultura de consultoria na universidade e capacitar os seus membros para os processos seletivos das grandes
empresas de consultoria de estratégia e de gestão.
 Facilitar contato entre alunos e consultores; Conteúdos abordados no clube
 Solicitar e organizar eventos e capacitações  Business Case Interview;
das consultorias;  Questões de GMAT;
 Divulgar processos seletivos e eventos das  Questões de Business Case (BC);
consultorias;  Perguntas de FIT (Storytelling);
 Divulgar conteúdos relacionados ao  Estruturação de Curriculum Vitae (CV);
Difundir Cultura mercado de consultoria. Capacitar Membros  English Group.

Considerações Projeto de extensão @clubedeconsultoria_ufc


vinculado à Pró-Reitoria de
Nosso clube já alcançou resultados relevantes com Extensão (PREX) da UFC.
Clube de Consultoria UFC
membros sendo aprovados nas principais
empresas de consultoria (e em outras excelentes Docente resonsável:
empresas de outros setores). Caso você tenha Rogério Mâsih - Pró-Reitor Clube de Consultoria UFC
interesse na carreira, venha fazer parte do clube! Adjunto de Extensão e
Coordenador do CCUFC. clubedeconsultoria_ufc@gmail.com

Clube de Consultoria UFC 2


Casebook Versão 1 - Atualizado em 14/06/2021 Feedback aqui!
Sumário Geral

1 2 3 4 5 6
Processo
Empresas de Conteúdo para Conhecimentos Resumo das
Seletivo de Cases
Consultoria Estruturação Relevantes Indústrias
Consultoria

Clube de Consultoria UFC 3


Feedback aqui!
Cases: Sumário e Classificações
# Nome do Case Complexidade Tipo Indústria Led By Quantitativo Qualitativo Pág.

1 Manhattan Boom! 1. Mini-Case Market Sizing Setor Público Interviewer 62

2 Easy Money 1. Mini-Case Market Sizing Serviços Financeiros Candidate 67

3 La Gatta Gelato 1. Mini-Case Market Sizing Varejo Interviewer 72

4 Vale do Rio Doce 2. D. Analítica Financial Analysis Mineração Candidate 77

5 Enfermeiro Anjo 2. D. Analítica Product Launch Serviços de Saúde Candidate 85

6 Tele Fortal 3. Diagnóstico Profitability Telecomunicações Candidate 93

7 Cards for Everyone 3. Diagnóstico Operations Serviços Financeiros Candidate 100

8 Where’s the Party 3. Diagnóstico Profitability Varejo Candidate 107

9 Safe Car 4. D. Estratégica Market Entry Automotiva Candidate 120

10 Greta Coffehouse 4. D. Estratégica M&A Restaurante Interviewer 126

11 BR Rodovias 4. D. Estratégica Market Entry Construção Civil Interviewer 133

12 Mobi Clean 5. Estrat. Aberta Revenue Growth CPG Candidate 141

13 Organ Donation 5. Estrat. Aberta Public Sector Setor Público Interviewer 147

14 SpaceX – Terra Nova 6. Ambíguo Public Sector Aeroespacial Interviewer 155

15 The Moto Perpetuo 6. Ambíguo Valuation Automotiva Interviewer 163

Clube de Consultoria UFC


Cases com ” “ são oficiais do CCUFC.
4
Feedback aqui!
1.1 PRINCIPAIS EMPRESAS 6
1.2 CARREIRA E BENEFÍCIOS 7

1. Empresas de
Consultoria
5
Principais Empresas
Embora existam muitas semelhanças entre as empresas de consultoria, cada empresa possui sua própria cultura, personalidade e
abordagens distintas para resolver problemas. Certifique-se de dedicar um tempo para conhecer o maior número possível de
empresas para entender qual delas é a mais adequada para você.
*Nem todas as consultorias adotam o formato de case interview presente neste casebook.

McKinsey, BCG e Bain.


As maiores e mais renomadas consultorias do setor. Grande parte
Big 3 dos projetos são realizados nas maiores empresas das mais diversas
indústrias, podendo abranger todas as funções de negócios.

AT Kearney, Oliver Wyman, Roland Berger, EY Parthenon, etc.


Grandes/Renomadas Consultorias nacionais e internacionais. Apresentam benefícios
Consultorias semelhantes às BIG 3 e algumas focam em um determinado tipo de
serviço ou setor com o fito de se diferenciar.

Falconi, Elogroup, BIG 4 (Deloitte, EY, PWC, KPMG), etc.


Outras grandes Empresas de consultoria que não aplicam, em seus processos
consultorias seletivos, o formato de case interview presente neste casebook.

Red Ventures, Nubank, Google, etc.


Empresas que não são de consultoria, mas que aplicam, em seus
Outras empresa processos seletivos, o formato de case interview presente neste
casebook.

Clube de Consultoria UFC


Consulta detalhada*: https://docs.google.com/spreadsheets/d/1BxYpQZnbkatbqLYBmqN9Ar-sqiycp-JLSuSXTyVgo14/edit#gid=0
6
Feedback aqui!
Carreira em Consultoria
Cada empresa possui sua nomenclatura de cargos e seu período médio de evolução, mas, em geral, a
carreira de consultoria é mais acelerada que a maioria das outras indústrias.
As consultorias mais renomadas patrocinam MBA nas melhores universidades do mundo, como Partner
Havard, MIT e Stanford, após o consultor passar pelo cargo de Analyst/Associate.
Director/
Algumas empresas possuem uma Principal
cultura “Up or Out”: caso a
performance do consultor não seja boa Manager -
o suficiente para subir de cargo, ele
pode ser convidado a sair da empresa.
Consultant 2 a 4 anos

Analyst / 2 anos
Associate
Intern 2 anos

2 a 4 anos
Principais benefícios da carreira: Saiba mais sobre
1 ano  Curva de aprendizado acelerada
cada consultoria:
 Valorização profissional para outros setores
 Salário e benefícios Clique aqui!

9 a 15 anos
Clube de Consultoria UFC 7
Fonte: PrepLounge.com / RocketBlocks / https://docs.google.com/spreadsheets/d/1BxYpQZnbkatbqLYBmqN9Ar-sqiycp-JLSuSXTyVgo14/edit#gid=0 Feedback aqui!
2.1 VISÃO GERAL DOS PS’s 9
2.2 CV E CL 10
2.3 TESTES 11
2.4 JOGOS e A. COMPORT. 12
2.5 FIT INTERVIEW 14
2.6 CASE INTERVIEW 15
2.6.1 Prompt & Clarify 16
2.6.2 Framework 18
2.6.3 Estimation 19
2.6.4 Hypothesis Testing 20
2.6.5 Chart Interpretation 21
2.6.6 Brainstorming 22
2.6.7 Analyses 23
2. Processo Seletivo 2.6.8 Recommendation 24
2.7 Q&A 25
de Consultoria
8
Visão Geral dos Processos Seletivos (PS)
A principal forma de ingresso em empresas de consultoria é por meio de seus processos seletivos. Abaixo estão as principais fases
desses processos. Contudo, cada empresa possui particularidades no seu processo, logo nem todas as empresas aplicam todas as
fases mostradas aqui.

Cadastro Inicial Avaliação Análise Entrevistas


Comportamental

Curriculum Vitae (CV) Teste estilo GMAT e


Vídeo Fit
Business Case

Cover Letter (CL) Jogo Dinâmica em grupo Case Interview

As empresas procuram quatro


habilidades principais Os melhores
 Capacidade de resolução de problemas Habilidade de Encaixe
candidatos Habilidades
 Impacto pessoal resolução de cultural e
devem interpessoais
 Impulso empreendedor problemas motivação
demonstrar:
 Liderança
Clube de Consultoria UFC 9
Feedback aqui!
CV e CL
Geralmente, o primeiro contato das empresas com os candidatos é por meio do Curriculum Vitae (CV) e da Cover Letter (CL).
Neles, é importante mostrar sua adequação à vaga e à empresa para a qual está aplicando. O CV é a sua marca pessoal.
Experiências profissionais e
CV Introdução pessoal Formação acadêmica
extracurriculares
Informações adicionais

 Nome e sobrenome em  Período, local e nome da


 Formação superior e
destaque empresa ou afiliação  Idiomas e nível
Deve Estar no CV instituição de ensino
 Telefone  Sua posição  Habilidades técnicas
 Período de formação
 E-mail  Suas contribuições

 Nota no curso
 Endereço  Cursos e certificações
 Premiações e  Resultados significativos e
É bom estar no CV  Nacionalidade  Premiações
reconhecimentos quantificados
 LinkedIn  Hobbies e interesses
 Outros cursos

Curriculum Vitae (CV) Cover Letter (CL)


Regras estruturais Dicas Sobre Cover Letter Dicas

 Uma página.  Seja objetivo. É uma forma de colocar suas  Uma página.
 Design e fonte simples.  Use verbos de ação ao experiências em um contexto. É uma  Seja estruturado.
 Dividido em seções. descrever suas atividades. maneira mais aberta do candidato  Expresse seus interesses.
 Atividades em ordem cronológicas  Peça para outras pessoas mostrar que possui as características  Use histórias para evidenciar suas
(as mais recentes primeiro). revisarem e lerem seu CV. que a empresa está procurando. qualidades/habilidades.
 Sem parágrafos grandes.  Quantifique seus resultados quando
 Sem erros ortográficos. possível.

Clube de Consultoria UFC 10


Feedback aqui!
Testes
Geralmente, nos processos de consultoria são aplicadas provas estilo GMAT e Estudos de Casos de Negócios (Business Case).
Algumas empresas podem ainda aplicar outros tipos de provas para avaliar características como raciocínio lógico e comportamento.
Esses testes normalmente são em inglês e no formato de questões de múltipla escolha.

GMAT
Graduate Management Admission QUANTITATIVAS VERBAIS RACIOCÍNIO
Test é normalmente necessário
para o ingresso em diversos
programas de MBA. Ele possui Data Suffiency:
Critical Integrated
tipos específicos de questões que Problem Solving: determinação se os Reasoning: Reading Reasoning:
costumam cair nas provas de dados Comprehension:
resolução de escolha do melhor interpretação e
apresentados são
problemas item baseado no compreensão de associação de
consultoria, avaliando, dentre matemáicos
suficientes para
argumento e no que texto gráficos, tabelas
responder o que foi
outras características, o raciocínio perguntado
foi solicitado etc.
lógico. Os tipos mais frequentes
são apresentados ao lado.
Business Case Características BC: Dicas:
Nesse tipo de prova, as empresas  Casos de consultoria apresentados  Mantenha-se atento
avaliam várias características, em formato de questões  Administre bem seu tempo
destacando-se senso de negócios e  Pode ter mais de um caso por  Seja crítico ao ler os itens
capacidade de associar informações. prova  Revise conceitos matemáticos/financeiros
 Pratique!
Clube de Consultoria UFC 11
Feedback aqui!
Jogos e Análise Comportamental
Algumas empresas podem incluir jogos, vídeos e dinâmicas em grupo no seu processo. Essas atividades podem avaliar tanto hard
skills quanto soft skills, assim como seu perfil e padrões de comportamento.
Jogos Exemplos de jogos Dicas
Jogos aplicados no processo têm  Entenda o que o jogo está tentado
diferentes objetivos e formatos. No geral, avaliar
não há muita preparação para essa  Leia bem as instruções e tutoriais
etapa, mas tentar conhecer um pouco  Divirta-se, mas jogue de maneira séria.
sobre eles pode ajudar a saber o que
esperar.
Vídeo Dinâmica
Nesta etapa, geralmente será solicitado a resposta a algumas Nesta etapa, normalmente, os candidatos devem trabalhar
perguntas em um tempo limitado. É importante não apenas o juntos para alcançar um objetivo. Geralmente, o principal
conteúdo da sua resposta, mas também a forma que você a objetivo é observar como o candidato trabalha com outras
expressa (estrutura). pessoas.
Dicas
Dicas  Encare os outros candidatos como seus colegas de equipe,
 Não leia! não como seus concorrentes
 Prepare-se e treine com antecedência  Expresse suas ideias e opiniões, mas também escute as dos
 Estruture sua resposta outros
 Comunique de maneira clara  Seja você mesmo
Clube de Consultoria UFC 12
Feedback aqui!
FIT Interview
5-15min 20-35min 5 min
Introdução & Fit Questões &
Case Interview
Interview respostas
No FIT, são analisadas suas soft skills, observando sua interação com os outros e adequação em consultoria e na empresa.

Perguntas Frequentes

Me fale sobre você  Prepare um pitch que inclua suas experiências passadas e presentes, assim como Dicas
seus interesses futuros
Me guie pelo seu CV  Relacione habilidades com suas experiências  Escreva e treine suas respostas
 Destaque pontos de transição e seu objetivo de carreira  Seja estruturado(a)!
 Tente encontrar algo em comum com
Por que consultoria? o entrevistador
 Entenda a cultura da empresa e como ele trabalha  Muitas empresas fazem FIT em
 Afirme diretamente suas motivações e então use experiências que as fortaleçam inglês, então é importante treinar
Por que empresa X?
nessa língua também
 Mostre paixão e entusiasmo
 Prepare suas histórias com antecedência, utilizando storytelling.
Me conte sobre uma vez que ...  Lembre-se: é uma conversa!
 Adapte sua história para responder a pergunta

Quais são seus pontos fortes e  Relacione seus pontos fortes com suas experiências
fracos?  Fale o que faz para superar seus pontos fracos
Guia Completo
...  Responda mantendo em mente os valores da empresa e a vaga para a qual está Cliique aqui!
aplicando
Clube de Consultoria UFC 13
Fonte: https://www.craftingcases.com/consulting-interview-questions/ Feedback aqui!
FIT Interview
Durante as entrevistas, é importante saber contar as histórias de maneira estruturada e mantendo o entrevistador engajado. O
método STAR e a Pirâmide de Freytag são boas sugestões sobre como montar um bom storytelling.

S ituação
Apresente o contexto da sua história. Não esqueça de detalhes,
como onde e quando, e deixe claro seu objetivo. Dicas

 Foque nas suas tarefas e contribuições


T arefa Descreva suas tarefas, problemas ou desafios enfrentados. individuais, no que você fez.

 Personalize sua história utilizando nomes,


A ção Descreva o que fez para completar sua tarefa, as ações tomadas. lugares e eventos.

 Faça a história parecer real, utilizando


R Apresente os resultados objetivos, focando não apenas nos
esultado resultados gerais, mas também no seu aprendizado. linguagem familiar e alcançando os
sentimentos do entrevistador.
Pirâmide de Freytag Climax
 Não decore! Treine sua história, mas
prepare-se para adaptá-la para responder
o que foi perguntado.
Contexto Resultado

Clube de Consultoria UFC 14


Fonte: https://www.youtube.com/watch?v=eBsER-ebrY4 / https://www.thebalancecareers.com/what-is-the-star-interview-response-technique-2061629 Feedback aqui!
Case Interview
5-15min 20-35min 5 min
Introdução & Fit Questões &
Case Interview
Interview respostas

Etapas de um case (pode apresentar 1 ou mais etapas)

1. Prompt & Clarify 2. Framework 3. Hypothesis Testing 4. Chart Interpretation


 Entender o problema  Estruturar “plano de ataque”  Criar hipótese  Entender gráfico
 Entender o modelo de negócio para solucionar o problema  Justificar lógica da escolha  Gerar insights e guiar para
da empresa  Priorizar análises  Estruturar forma de validação próximos passos
 Definir objetivo  Solicitar dados para validar

5. Estimation 6. Brainstorming 7. Analyses 8. Recommendation


 Estruturar sugestão de  Pensar (criatividade) e  Estruturar cálculos  Sintetizar análises realizadas
estimativa estruturar ideias  Calcular e interpretar resultado  Comunicar recomendação top-
 Solicitar dados ou justificar  Priorizar e justificar down com riscos e próximos
assumptions passos
 Calcular valor final

Clube de Consultoria UFC 15


Fonte: www.students.craftingcases.com/courses/take/case-interview-fundamentals/lessons / High Bridge Feedback aqui!
Prompt & Clarify
Com o intuito de facilitar o entendimento, as 8 etapas mencionadas serão detalhadas nos próximos slides. A sugestão de abordagem
será registrada na parte cinza, enquanto o exemplo de utilização em um case na parte branca e inferior do slide.
Neste momento, o entrevistador irá apresentar a história do case, muitas vezes, seguindo a estrutura Contexto  Situação  Problema. O
principal objetivo dessas etapas é entender, perfeitamente, o problema a ser resolvido. Para atingir esse objetivo, serão destacadas
3 boas práticas que podem ser realizadas em sequência.

1. Escutar e anotar: Escutar atentamente e anotar apenas o que for mais relevante.
2. Parafrasear enunciado: Confirmar as informações recebidas. A paráfrase não é o objetivo, mas sim a ferramenta para alcançá-lo.
3. Realizar perguntas de Clarify: A próxima página deste casebook apresenta este ponto em detalhes..

OBS: Apesar de ser ressaltada como uma primeira etapa, as boas práticas mencionadas podem ser utilizadas na transição de cada etapa do case.

Exemplo:
Contexto: Rony é o fundador e presidente da Relf, uma marca famosa de vestuário masculino no 1. As partes em negrito de cor azul do
Brasil, e está buscando meios de aumentar a receita da sua empresa. prompt a esquerda são exemplos de
Situação: Após um crescimento rápido, a receita de 100 milhões de reais permanece estagnada e pontos relevantes que podem ser
uma oportunidade que Rony estava analisando era de expandir seu mix de produtos entrando no anotados no papel.
mercado de relógios de luxo, visto que sua marca, atualmente, é consumida por clientes de 2. A paráfrase deve ser realizada resumindo
médio e alto poder aquisitivo. A ideia é utilizar o modelo private label com o intuito de focar o prompt informado e confirmando
esforços no marketing e na venda dos produtos. informações que não ficaram claras, visto
Problema (pergunta para Framework): Quais os principais pontos que seriam necessários analisar que comunicar todo o prompt pode ser
para decidir se vale a pena a Rony iniciar no negócio de relógios de luxo no Brasil no modelo cansativo para o entrevistador.
private-label? 3. Clarify no próximo slide...

Clube de Consultoria UFC


Fonte: Crafting Cases / High Bridge
16
Feedback aqui!
Prompt & Clarify
Normalmente, são recomendadas entre 3 a 5 perguntas clarify, todavia este número pode variar dependendo do tipo de problema e da quantidade de informações
apresentadas pelo entrevistador. A seguir serão comentados os 5 tipos mais comuns de perguntas clarify:

“If I had an hour to solve a problem I'd spend 55 minutes thinking about the problem and 5 minutes thinking about solutions.” – Albert Einstein

Definições Situação Objetivo Modelo de Indústria/


negócio Cenário
 Termos desconhecidos.  O que aconteceu para  Existe um objetivo mais  Buscar visualizar um  O problema pode ser da
 Termos ambíguos. Uma precisarem de ajuda? tangível? modelo de negócio economia, da determinada
mesma palavra pode ter  Quanto cresceu/diminuiu?  É possível quantificar? conhecido pode ajudar. indústria ou apenas da
significados em  Em quanto tempo?  O que é sucesso para o  Quem é o cliente final? empresa.
contextos diferentes.  Qual o motivo de uma cliente?  Como gera receita?  Devemos agir na nossa
decisão? Devemos  Qual a região de atuação? atual indústria ou
considerar apenas ela? compensar em outro lugar?
Ex1: A receita caiu em Ex1: Nosso cliente tem Ex1: Como nosso cliente Ex1: Os concorrentes
Ex1: O que é ROI? quanto e durante qual alguma meta quantitativa gera receita? Apenas por também apresentaram
Ex2: Lucro em reais ou período? de lucro? vendas online? queda de lucro?
margem de lucro? Ex2: Aconteceu no cenário Ex2: Qual a meta de Ex2: Negócio semelhante Ex: Iremos considerar o
mundial ou local sucesso do cliente? a um modelo de fast-food? período de pandemia?

Vamos considerar Existe algum objetivo Possui um modelo de


O que significa PRIVATE Vamos considerar o
apenas a opção de White quantitativo de aumento negócio semelhante a
LABEL? cenário de COVID?
label? de receita? loja da reserva?
Terceirizar toda a Sim. O cliente não tem
Aumentar em 5% a receita Sim. Possui loja própria,
produção do produto interesse em realizar a Não. Economia está em
atual apenas com essa mas também revende para
(adicionar apenas a marca produção, pois foge do alta.
nova família de produtos. outras lojas renomadas.
Relf e realizar venda). core business do negócio.
Clube de Consultoria UFC
Fonte: High Bridge
17
Feedback aqui!
Framework
O Framework será o plano de ataque para resolver o problema. O ideal é que ele seja MECE.

1. Definir 2. Levantar 3. Criar 4. Criar teste de


Estrutura Assuntos hipóteses hipótese
 Clarificar o problema é crucial  Anotações no papel apenas dos tópicos  Não precisa escrever no papel  Necessário para guiar o case e
 Pedir tempo neste momento para  1-2 minutos para os tópicos  Comunicar apenas as relevantes  Comunicar apenas após
etapa 1 e 2  Levantar perguntas específicas para o  Cada assunto possui uma hipótese apresentar o framework
 15-30s para os macro-buckets problema/ indústria por trás  Buscar testar 1 de cada vez
 Utilizar 5 Ways to be MECE  Adicionar pontos óbvios e pontos  Ter hipóteses ajuda a ser mais
 Utilizar técnica landscape criativos (ser MECE) específico e data-driven

 Tamanho e crescimento relevante? Exemplos de teste de hipótese:


Mercado Exemplo de hipótese:
 Fragmentado ou concentrado?
1. Realizar benchmarking com
Clientes
 Atuais gastariam mais? empresas semelhantes, analisando
“Os atuais gastariam mais?”
Clientes  Ganharíamos novos clientes? a influência da canibalização a
partir da introdução de novos
Hipótese produtos
 Capacidade de produção e distribuição
“Acredito que os clientes que
Fornecedor  Qualidade desejada (Design etc)
comprariam os relógios não 2. Realizar pesquisa com atuais
 Confiabilidade de entrega
deixariam de comprar nossos clientes, analisando a possibilidade
 Poder de barganha nos canais de distribuição outros produtos, visto que possuem de promover o cross-seling dos
Empresa  Força de marketing e divulgação um alto poder aquisitivo para relógios com o atual mix de
 Estrutura de custos necessária adquirir o que desejam” produtos da Relf

Clube de Consultoria UFC


Fonte: Crafting Cases
18
Feedback aqui!
Estimation
No estimation, deve-se mostrar a capacidade analítica de construir estruturas algébricas.

1. Clarificar 2. Estruturar 3. Escolher 4. Calcular 5. Teste de


escopo problema assumptions sanidade
Perguntas comuns:  Preferência por estruturas  Perguntar ao entrevistador  Cálculos devem ser precisos  Comparar valor final com
 Dimensão do número final top-down (issue-tree)  Visualização e eficientes números conhecidos
desejado  Adicionar a dimensão de  Breaking-down (criar uma  Falar e explicar em voz alta  Dividir por um valor chave
 Região/local cada bucket do estimation tabela ou dividir em mais  Pode pedir para arredondar (ex: # famílias)
 Delimitação do estimation  Sinais algébricos para buckets)  Ter em mente a escala de  Técnicas de assumptions
(ex: B2C e/ou B2B) relacionar matematicamente  Extrapolação e regra de 3 grandeza antes de calcular ajudam
matemática  Ampliar Fact Bank
 Ballparking e proxies

Estruturação

Pergunta Estimation
Assumptions e Cálculo
“Quantos relógios de luxo
masculino existem hoje no Teste de sanidade
Brasil?”
“200M / 4,2M = 1 relógio a cada 50
brasileiros. Aparenta ser um número
razoável.”

Clube de Consultoria UFC


Fonte: Crafting Cases
19
Feedback aqui!
Hypothesis Testing
A etapa de Hypothesis Testing será onde o candidato mostrará que ele sabe onde buscar os dados corretos para validar hipóteses.

1. Definir Hipótese 2. Estruturar/ desdobrar 3. Coletar e processar


dados
 Definir de acordo com o contexto do case e  Desdobrar a hipótese em perguntas específicas  Solicitar o dado desejado e informar como obtê-lo
comunicar ao entrevistador  Muitas vezes, não é necessário desdobrar a  Qual cruzamento de informações deve ser
 Toda hipótese deve ter uma causa, um efeito e uma hipótese, pois pode-se testá-la de forma direta realizado?
consequência (essa etapa não precisa ser  Estruturar perguntas e hipóteses secundárias que  Qual a melhor métrica para fazer comparações?
apresentada ao entrevistador) direcionem a validação ou não da hipótese principal (Caso seja um gráfico, como ele poderia ser
 Causa  Efeito  Consequência  Considerar a viabilidade de conseguir dados organizado?)
 O efeito da hipótese gera a consequência analisada,  Utilizar 5 ways to be MECE  Como tornar o dado em informação?
sendo que algo causa ou causou a hipótese

Pergunta Hypothesis Quais os critérios ganhadores de  Mapear clientes que comprariam


pedido dos clientes?  Realizar pesquisa dos principais ganhadores de pedido
Testing

“O Rony está muito confiante


Qual o nível dos concorrentes em  Levantar principais concorrentes
que a empresa conseguirá cada critério?  Definir nível dos clientes em relação a cada critério (ex: outra pesquisa)
fornecer um produto para os
cliente de forma superior aos
atuais concorrentes. Como Conseguimos fornecer um produto  Mapear potenciais fornecedores viáveis
você validaria essa hipótese?” melhor?  Nível dos potenciais fornecedores em relação aos critérios

Clube de Consultoria UFC


Fonte: Crafting Cases
20
Feedback aqui!
Chart Interpretation
A parte de chart será o momento de coletar insights que serão relevantes para resolver o problema.

1. Entender o gráfico 2. Gerar insights 3. Sintetizar e Guiar

 Confirmar com entrevistador se o entendimento do  Chegar nesta etapa apenas após total compreensão  Combinar insights e resumir em apenas uma
gráfico está correto do gráfico mensagem
 Pode-se utilizar a sequência TELE para ler o gráfico  Trazer para o entrevistador as mensagens principais  Sugerir próximos passos do que deve ser feito para
T: Título e textos do gráfico (ex: pontos fora da curva ou de mudança) o cliente atingir seu objetivo: análises
E: Eixos ou cabeçalhos (para tabelas)  Insights devem ser relevantes para o contexto do complementares, dados necessários para validar
L: Legenda e rodapé (Se houver) case hipótese, etc.
E: Exemplo (escolher um ponto ou uma linha/coluna  Buscar quantificar o insight (ex: em vez de maior,
para melhor compreensão X% maior)
 Clarificar termos, unidades e relacionamentos

Pergunta Chart Interp. Entendimento


Ponto exemplo: preço possui relevância nível 6 para o cliente, sendo que o
“Nossa equipe coletou concorrente A não atendendo as expectativas e o B está desperdiçando
energia neste critério.
alguns dados relacionados
Insights e Síntese
às preferências do cliente e Oportunidade de se destacar em design e ergonomia, visto que são critérios
o posicionamento dos 2 relevantes para o cliente cujos concorrentes não estão atendendo
principais concorrentes. (quantificar aqui). Além disso, os concorrentes estão gastando mais recurso
que o necessário em alguns critérios, como durabilidade e funções extras.
Quais insights você tiraria Caso a Ralf consiga fornecedores competentes nesses critérios, ela
desse gráfico?” poderá se destacar? Temos dados dos potenciais fornecedores?
Clube de Consultoria UFC
Fonte: Crafting Cases
21
Feedback aqui!
Brainstorming
Este é o momento para mostrar o máximo de criatividade. Não esquecer de ser MECE.

1. Definir e Estruturar 2. Priorizar Drivers 3. Gerar Hipóteses

 Clarificar de forma específica o que está sendo  Pontuar 1 ou 2 bullet points que são mais relevantes  Informar possíveis hipóteses sobre os pontos
pedido (ex: o que significa melhorar?)  Considerar o contexto do case priorizados de acordo com o contexto do case
 Definir estrutura MECE (5 Ways). Pensar em várias  Pensar nos drivers que geram mais impacto  Comentar o óbvio
formas de estrutura pode aumentar o volume de  Pensar na facilidade e na probabilidade de ser  Pensar fora da caixa
ideias executado  Analogias podem ajudar a explicar ideias e
 Utilizar visualização demonstrar criatividade
 Tornar bullet points específicos  Não deixar de pensar na viabilidade das ideias
 Sugerir next-steps

 Aluguel da fábrica
Pergunta Brainstorming  Depreciação de instalações e equipamentos Priorização
Custos Fixos  Salário da equipe operacional “Os custos mais significativos, ao meu ver,
“O cliente ficou preocupado  Manutenção (Instalação e equipamentos) seriam os relacionados à depreciação da
 Logística de transporte fábrica, visto que seria necessário muito
com a perda de controle do  P&D (Pesquisa e desenvolvimento de capital para infraestrutura, maquinário e
produto ao terceirizar a produtos) equipamentos”.
produção. Caso ele optasse  Outros (Seguro, imposto, TI) Hipótese
por realizar produção própria, “Devido à complexidade e a falta de
 Matéria prima (Vidro, metal, plástico, experiência de produção de relógios, acredito
quais seriam os diferentes Custos Variáveis peças prontas etc) que seriamos mais lucrativos com o modelo
custos a se preocupar?”  Embalagem de negócio de private-label (PL)”.
 Energia
Clube de Consultoria UFC
Fonte: Crafting Cases
22
Feedback aqui!
Analyses
Esta etapa é relevante para apresentar a capacidade analítica e o business sense após finalizar o cálculo.

1. Estruturar 2. Solicitar 3. Calcular 4. Gerar


análise dados insights
 Estruturar passo a passo da análise  Não tentar estimar algo, a menos que  Preciso e eficiente  Lembrar do “so what?”: o que o
para alcançar o resultado desejado seja solicitado  Comunicar o passo a passo do número encontrado ajuda o cliente
 Estruturas algébrica e de processo  Verificar se existem dados necessários cálculo no seu objetivo
podem ser ótimas opções que não foram informados  Buscar corrigir cálculos antes do  Os insights devem guiar o case
 Confirmar com entrevistador se a  Solicitar dados explicando o motivo de entrevistador pontuar para os próximos passos
estrutura é viável (alterar estrutura, se sua importância  Utilizar potencias de 10 ou letras ao
necessário)  Ex: a penetração que vamos calcular grande números (ex: K, M e
 Buscar uma estrutura de cálculo considerar será de 100%? B ou 10^3, 10^6 e 10^9)
simples, que precise de menos  Pré-reality check: Verificar dimensão
esforço para chegar ao resultado final do resultado final antes de calcular

Previsão do # vendas: 10.000 u/ano


Preço médio: R$ 1500,00 1.1. 1500 – 1100 = 400,00
Custo c/ fornecedor PL 1. Lucro PL 1.2. 400 x 10.000 = 4M Insight
Pergunta Analyses  R$ 1100,00/u 4M > 1,5M, assim, o investimento por
Custo para produzir meio do modelo de negócio private-
“Tenho alguns dados  Matéria prima: 830,00/u label se demonstrou mais relevante
 Embalagem: 70,00/u financeiramente. Além disso, 4M de
que talvez sejam  P&D: 500.000,00/ ano 2.1. 1500 – (830 + 70) = 600,00 lucro representa 4% da receita anual da
2. Lucro
relevantes”  Depreciação: 1.000.000,00/ano produção
2.2. 600 x 10.000 = 6M empresa de 100M, sendo, assim, um
 Pessoal: 1.500.000,00/ano 2.3. 0,5M + 1M + 1,5M + 0,5M = 3,5M negócio relevante para a empresa.
própria
 Outros: 500.000,00/ano 2.4. 6M – 3,5M = 1,5M

Clube de Consultoria UFC


Fonte: Crafting Cases
23
Feedback aqui!
Recommendation
Momento para sintetizar todas as informações coletadas e gerar uma conclusão para o caso.

1. Recomendação/ 2. Argumentações 3. Riscos e Próximos


Conclusão Passos
 Sintetizar recomendação/conclusão final em poucas  Levantar as conclusões dos principais pontos  Informar os possíveis riscos da decisão
palavras de acordo com o que foi analisado no case analisados, argumentando de acordo com a recomendada
 Em muitos cases, é possível recomendar diferentes recomendação/conclusão  Em seguida, apresentar próximos passos, podendo
ações e continuar correto  Manter-se conciso ser, por exemplo:
 Pode-se solicitar tempo para estruturar a  Pontuar argumentos pode facilitar a comunicação 1. Formas de mitigar os riscos comentados
recomendação  Finalizar argumentações reforçando a ação que o 2. Ações que devem ser feitas para apresentar uma
cliente deve tomar resposta mais confiável ao cliente
3. Formas de alavancar resultados

Riscos
1. Lucro representativo  4M de lucro = 1. Fornecedor vender produto
Pergunta Recomendação Recomendação 4% do da receita semelhante para concorrentes
2. Reação dos concorrentes
“A empresa Relf gostaria
“Vamos ter uma reunião
de saber se vale a pena
daqui a 1 hora com o Sr. 2. PL é mais lucrativo  4M > 1,5M Próximos Passos
investir no negócio de
1. Firmar contrato rígido de
Rony, você poderia montar relógios de luxo no
exclusividade de fornecimento
modelo private-label e
uma recomendação para 2. Buscar negociar com principais
minha recomendação é 3. Ganhar mais mercado  Investir em
apresentar para ele?” que sim, ele deve entrar” no longo prazo
canais de distribuição para realizar uma
Design e entrada rápida, diminuindo poder de
(critérios) Ergonomia reação dos concorrentes
Clube de Consultoria UFC
Fonte: High Bridge
24
Feedback aqui!
Q&A
5-15min 20-35min 5 min
Introdução & Fit Questões &
Case Interview
Interview respostas
Nessa parte, o candidato tem a oportunidade de mostrar seu interesse e aprender mais sobre a empresa, por meio de um
contato mais direto que palestras ou sites. Por isso, caso seja oferecido, não perca a oportunidade de fazer perguntas. Use
essa oportunidade para te ajudar a determinar se a empresa é realmente o que você está procurando.

Nessa etapa, busque: Dicas:

 Mostrar interesse e curiosidade sobre a empresa  Tente relacionar suas perguntas ao perfil do seu
 Trazer insights sobre a empresa entrevistador (ex.: sua área de especialização, experiências
etc.)
Tente evitar:  Pense em perguntas antes da entrevista
 Escute ativamente
 Perguntas que possam ser facilmente respondidas com  Mostre que as respostas de fato importam para você
uma busca rápida no Google ou site da empresa  Tente manter um tom de conversa, não de entrevista
 Perguntas cujas respostas sejam apenas “sim/não”
 Perguntas mais sensíveis (salários, horas de trabalho etc.) Guia completo
Clique aqui!

Clube de Consultoria UFC 25


Feedback aqui!
3.1 5 WAYS TO BE MECE 27
3.2 EDUCATIONAL CONTENT 29
3.2.1 5 Forças de Porter 29
3.2.2 4P’s do Marketing 30
3.2.3 3C’s Estratégicos 31
3.2.4 Técnica Landscape 32
3.2.5 Supply Chain 33
3.2.6 Matriz Ansoff 34
3.2.7 Matriz BCG 35
3.3 TYPICAL FRAMEWORK 36
3.3.1 Profitability 36
3.3.2 Market Entry 37
3.3.3 M&A 38
3. Conteúdo para 3.3.4 Price Strategy 39

Estruturação
26
5 Ways To Be MECE
MECE é um conceito utilizado para organizar informações, de modo que estas sejam agrupadas em categorias onde não haja
sobreposição entre elas e as categorias somadas cubram todas as opções possíveis.

Mutuamente Exclusivo Coletivamente Exaustivo


Nenhum elemento aparece em mais de
uma categoria.
MECE Todos os elementos relevantes são
considerados.

5 Maneiras de ser MECE


Saber criar estruturas MECE do zero é importante para que seja possível resolver qualquer tipo de problema. Existem 5 tipos de
estruturas que podem contribuir para a aplicação desse conceito nos seus problemas:

Estrutura algébrica Estrutura de processos Estrutura Conceitual


Segmentação Palavras opostas
“Todo problema “O problema é quebrado “O problema é
“O problema é “O problema é quebrado
numérico pode ser nos seus componentes, quebrado nos principais
quebrado em grupos em categorias baseadas
quebrado em equações ou seja, nas diferentes conceitos que o
menores.” em conceitos opostos.”
matemáticas.” partes do seu processo.” compõe.”

Clube de Consultoria UFC 27


Fonte: https://www.craftingcases.com/the-5-ways-to-be-mece/ Feedback aqui!
5 Ways To Be MECE
Quais são os possíveis motivos para a queda de market share da rede de farmácias A. no Brasil?
Estrutura algébrica Estrutura de Estrutura Conceitual Segmentação Palavras opostas
processos
Menos novos clientes ↓ MS da
conhecem as farmácias A. Farmácia A.
↓ MS da Os clientes mudaram Clientes ocasionais
Farmácia A.
Menos clientes tem acesso
as farmácias A. Causado por Causado por
fatores fatores
externos internos

Menos clientes interessados Os competidores


Receita da Mercado total Clientes regulares
nas farmácias A. mudaram
Farmácia A. de farmácias
Relacionado
ao marketing
Menos clientes pegam os
produtos das farmácias A.
As farmácias A.
Clientes fieis Não
mudaram
Menos clientes compram relacionado
nas farmácias A. ao marketing

Já essas duas, em muitos casos, podem ser


Essas três estruturas são a base da resolução de case. melhor utilizadas junto a outras estruturas, de
forma complementar.
Clube de Consultoria UFC 28
Fonte: https://www.craftingcases.com/the-5-ways-to-be-mece/ Feedback aqui!
Educational Content: 5 Forças de Porter
1. Objetivo 2. Abordagem 3. Aplicação
 Mensurar a competitividade de um mercado com o  Aplicável tanto para novos entrantes como para  Casos de entrada de mercado.
intuito de analisar sua atratividade. Mercado, neste empresas existentes. De forma geral, quanto maior  Reação competitiva contra novos entrantes (foco
caso, está relacionado com uma indústria específica. as forças são, menos atrativa é a indústria. Tal nas barreiras de entrada).
Ex: Automotiva, telecomunicações, etc. análise pode ajudar a definir se uma potencial  Análise de atratividade de um mercado.
estratégia será lucrativa no determinado mercado.  Criação de estratégias de longo prazo.

1. Rivalidade entre  Quantos concorrentes sua empresa tem?  Como a qualidade dos seus produtos se Considerações
Concorrentes  Concorrentes atuam de forma direta? compara com as dos demais?
 Como se diferenciam uns dos outros?  É possível uma guerra de preços?  As 5 forças fazem referências às partes
permanentes de um mercado. Assim,
2. Poder de Barganha  Quantos compradores controlam suas  Seus compradores poderiam trocar de
fornecedor? Qual custo de migração? dificilmente, se tornará uma análise
dos Clientes vendas?
obsoleta ou pouco relevante.
 Qual tamanho dos pedidos que recebe?  Qual a importância do seu produto?

 Quantos fornecedores?  Qual custo de migração para outro Ameaça de


3. Poder de Barganha
dos Fornecedores  Qual volume de suas compras em cada fornecedor? novos entrantes
um deles?  Há algum contrato que lhe prenda?

4. Ameaças de Novos  Quanto custa e quanto tempo leva para  Como tornar o negócio escalável? Poder de Poder de
entrar no seu mercado?  Você protege suas tecnologias (P&D)? barganha dos
Rivalidade da
barganha dos
Entrantes concorrência
 Quais são as barreiras de entrada?  Quão regulamentado seu mercado é? Clientes fornecedores

5. Ameaças de Produtos  Quais os diferencias em relação ao  Qual custo da mudança para o produto
substituto? substituto? Ameaça de produtos
Substitutos
 Quantos substitutos estão no mercado?  Quão difícil seria fazer essa troca? substitutos

Clube de Consultoria UFC 29


Fonte: https://blog.runrun.it/5-forcas-de-porter / preplounge.com Feedback aqui!
Educational Content: 4P’s do Marketing
1. Objetivo 2. Abordagem 3. Aplicação
 Desenhar a estratégia de marketing de um  Identificar o melhor posicionamento de um produto,  Lançamento de produto (entrada de mercado).
produto/serviço em um mercado, elaborando uma por meio da análise em conjunta dos 4 pilares, visto  Expansão regional (entrada de mercado).
estratégia integrada de preço, praça e promoção. que a mudança de atuação em 1 pilar por afetar,  Reposicionamento do mix de produtos.
fortemente, a definição dos outros.  Precificação/valuation: valor percebido.

 O que você oferece para seu consumidor?  Em qual fase do clico de vida está seu Considerações
1. Produto  Qual necessidade ou desejo você atende? produto?
 O que o consumidor busca no seu produto?  Como o produto é fisicamente?  Exemplo de marcas que são referências
no que tange seu posicionamento nos
4P’s: Coca-Cola, Havaianas, Nike, etc.
 Quanto cobrar do consumidor?  Quais os custos da venda?
2. Preço  Qual o valor percebido pelo cliente?  Qual margem esperamos obter?
 Quanto os concorrentes cobram?  O cliente é sensível ao preço?
Product
 Por qual canal vender?  Em que local físico ou digital está o
3. Praça  Como eu armazeno? meu consumidor? Price
 Como eu distribuo?  Qual tipo de esforço de venda?
Place
 Como divulgar ao consumidor?  Quais momentos em que posso me
4. Promoção  Como o consumidor se comporta ao longo conectar?
do dia?  Qual melhor momento para promover?
Promotion

Clube de Consultoria UFC 30


Fonte: https://neilpatel.com/br/blog/4-ps-do-marketing / preplounge.com Feedback aqui!
Educational Content: 3 C’s Estratégicos
1. Objetivo 2. Abordagem 3. Aplicação
 Auxiliar na elaboração de estratégias que possibilitem  Os pilares devem ser trabalhados de forma  Framework base para muitos problemas de
a vantagem competitiva, de longo prazo, de uma conjunta, de modo que a empresa deve utilizar suas negócios.
empresa em um determinado mercado. vantagens para vencer no mercado, entregando  Exemplos: Entrada de mercado; Mudança no mix de
algo para os clientes com diferenciais relevantes produtos/clientes; Reação competitiva, Estratégia de
dos concorrentes. crescimento (principalmente, long-term).

 Tipos de clientes e dados demográficos:  Critérios ganhadores de pedido, Considerações


sexo, idade, renda, etc. satisfação e lealdade dos clientes.
1. Customer  Tamanho e crescimento do mercado por  Principais canais de comunicação e  Somente integrando os três Cs em um
segmento de cliente. distribuição utilizados. triângulo estratégico é possível garantir
 Sensibilidade a preço (elasticidade)  Tendências do mercado. uma vantagem competitiva sustentável.
 Pode-se criar frameworks fortes ao
 Principais concorrentes (diretos e indiretos).  Envolvimento com os clientes. Ex: adaptar o 3C’s com a técnica
 Market-share: fragmentado ou concentrado. comunicação e atendimento. Landscape.
2. Competitor  Histórico e projeção de crescimento.  Eles possuem marca e proposta de
 Quais são seus pontos fortes e fracos? valor relevantes?
Competitor
Ex: P&D, economia de escala, etc.  Possíveis reações estratégicas.
Strategy

 Quais são seus objetivos?  Reconhecimento e reputação da marca.


 Como está sua saúde financeira?  Fidelidade da base de clientes.
3. Company Lucratividade, fluxo de caixa, etc.  Estrutura de custos Company Customer
 Cultura organizacional  Relacionamento e força nos canais de
 Oferta do produto/serviço. distribuição.

Clube de Consultoria UFC 31


Fonte: marfin.co/blog/modelo-3cs / Igotanoffer.com Feedback aqui!
Educational Content: Técnica Landscape
Possui o objetivo de facilitar a criação dos tópicos macros do framework, por meio da visualização dos principais stakeholders envolvidos no problema. Pode ser utilizada para
criar frameworks do zero ou para adaptar frameworks conhecidos, como o 3Cs, em estruturas para problemas específicos. Além do exemplo abaixo, podem-se adaptar
frameworks de profitability, 4P’s do marketing etc.

Oferta Demanda
Fornecedores Canais de
Distribuição
Empresa

Empregados Produtos
Sindicatos Clientes dos
Clientes
Acordos Clientes
Investidores

Governo
Academia Concorrência
Agências
regulatórias

Tecnologia Infraestrutura Ambiente Economia Leis Cultura e Crenças

Clube de Consultoria UFC


Fonte: Crafting Cases
32
Feedback aqui!
Education Content: Supply Chain
1. O que é? 2. Desafios e 3. Considerações
oportunidades
 Cadeia de suprimentos (supply chain) consiste num  Aprimoramento nos serviços e redução de custo  Entender onde a empresa está posicionada na
conceito que abrange todo o processo logístico de operacional, por meio de parcerias entre empresas. cadeia de suprimentos é crucial para a criação de
determinado produto ou serviço, desde a sua  Deficiência em um elo da cadeia pode prejudicar uma boa estrutura e para a resolução do case.
fabricação até a sua entrega ao consumidor final. todo o setor.  Diferente de logística, a qual diz respeito às
 Possível verticalização pode garantir vantagens integrações dentro da empresa, a cadeira de
competitivas relevantes (e prejudicar outros). suprimentos consiste no relacionamento entre
 Busca por práticas sustentáveis de forma integrada. empresas até o cliente final.

Fluxo de Informações

Fluxo Financeiro e de Produto

Clube de Consultoria UFC


Fonte: patrus.com.br / myconsultingoffer
33
Feedback aqui!
Education Content: Matriz Ansoff
1. O que é? 2. Vantagens 3. Considerações
 Abordagem que visa projetar caminhos para o  Possibilidade de projetar cenários pautados em  Utilizada para avaliar estratégias de lançamento de
desenvolvimento de uma empresa e, estratégias; novos produtos ou entrada de mercado.
simultaneamente, levar em consideração sua  Facilitar a visualização de cada caminho a ser  Brainstorming de quais produtos podem ser criados
realidade macro. Consiste no cruzamento de duas trilhado, tornando as possibilidades mais tangíveis; pela empresa.
variáveis principais, sendo estas mercado e produto.  Aplicabilidade como forma de planejamento no  Estratégica de crescimento. Aplicação direta para
curto, médio e longo prazo. crescimento no curto prazo (ampliar mix).

 Combinação de um mercado existente com produtos existentes. O foco Produto


1. Penetração de
principal nesse bloco diz respeito a buscar uma maior fatia do mercado, captando Existente Novo
Mercado
clientes da concorrência e fidelizando os seus próprios consumidores.

Existente
2. Desenvolvimento de  Combinação de um mercado novo com produtos existentes. O escopo dessa Penetração de Desenvolvimento
Mercado de Produto
Mercado estratégia é oferecer produtos que já são comercializados em um mercado para
outros.

Mercado
 Combinação de um mercado existente com produtos novos. O objetivo nesse
3. Desenvolvimento de
ponto é oferecer ao mercado habitual um novo leque de produtos, de modo a
Produtos
atender novas necessidades e aumentar seu ticket médio.

Novo
Desenvolvimento Diversificação
de Mercado
 Combinação de um mercado novo com produtos novos. A diversificação poderá
4. Diversificação ser relacionada (havendo sinergias com o modelo de negócio atual) ou não
relacionada (ausência de tais sinergias). Cenário de maior risco para a empresa.

Clube de Consultoria UFC 34


Fonte: rockcontent.com/br/blog/matriz-ansoff / Casebook Esade 2011 Feedback aqui!
Education Content: Matriz BCG
Participação Relativa no Mercado
1. O que é? 2. Considerações Alta Baixa

Crescimento do Mercado
 Utilizada para avaliar o desempenho do portfólio de  Estratégia: investir o fluxo de caixa gerado

Alta
produtos ou serviços de uma empresa em relação a pelas vacas leiteiras nos pontos de
sua aceitação pelo consumidor, a partir de duas interrogação para se tornarem estrelas.
variáveis principais: crescimento do mercado e a  Pode-se fazer uma relação da matriz BCG
participação relativa do produto no mercado. com o ciclo de vida do produto devido às
 Busca analisar quais produtos a empresa deve etapas que um produto apresenta desde o

Baixa
priorizar investimentos ou desinvestir. seu lançamento até seu declínio.

 Consiste em produtos com um bom retorno financeiro sem a


1. Vacas Leiteiras necessidade de muitos esforços, fato justificado pela sua
qualidade e boa reputação.
Ciclo de Vida do Produto
 Consiste em produtos com os piores retornos para uma empresa.
2. Abacaxis É cabível um plano de ação de imediato para recuperar o
prestígio de tal produto ou apenas removê-lo de sua carteira.

 Consiste em produtos com um bom retorno financeiro, mas


3. Estrelas havendo a necessidade de esforços para uma boa aderência e
aceitação.

 Consiste em produtos com perspectivas incertas, em que a


4. Ponto de Interrogação previsibilidade é algo intangível. São considerados as apostas de
uma empresa, podendo se tornarem um sucesso ou um fracasso.

Clube de Consultoria UFC 35


Fonte: https://rockcontent.com/br/blog/matriz-bcg / Casebook Esade 2011 Feedback aqui!
Typical Framework: Profitability
1. Objetivo 2. Como Identificar 3. Aplicação
 Diagnosticar a causa raiz dos problemas de  No prompt do case é comum observar comandos  É necessário identificar quais drivers do lucro
lucratividade, ou seja, busca conhecer o que pode do tipo “Seu cliente vem sofrendo um problema (receita ou custo) estão impactando a lucratividade e
está causando um declínio nos lucros e saber com a margem.”, “A lucratividade da empresa está isolar ao máximo até ter uma causa raiz;
como reverter esse declínio. baixando.”.  Com a causa do problema, se for necessário, adote
 Gerar brainstorming/hipóteses de aumento de uma segunda estrutura de análise para encontrar o
lucro de um negócio. motivo de isso está acontecendo e o que pode ser
feito para mudar essa situação

# unidades Considerações
vendidas
1) Segmentação útil de receitas
Receitas
 Tipo de produto;
Preço/ Desdobramento da receita dependerá do modelo  Canal de distribuição;
Unidade de negócio. Igualmente, aos custos variáveis (CV).  Tipo de cliente;
 Região.
Lucro # de unidades 2) Exemplos de Custos Fixos
 Salários e encargos;
Custos Variáveis  Aluguel;
(CV)  Marketing;
C. Variável/  Manutenção.
Custos Unidade 3) Custos Variáveis
Custos Fixos  Matéria Prima;
Margem de Contribuição (MC) = R – CV ou
(CF)  Energia;
MC (%) = (R – CV)/R*100
 Comissões;
MCunitária (MCu)= P – CVunitário
 Taxas.
Clube de Consultoria UFC 36
Fonte: www.craftingcases.com/profitability-tree-guide / igotanoffer.com/blogs Feedback aqui!
Typical Framework: Market Entry
1. Objetivo 2. Como Identificar 3. Aplicação
 Tomar decisões se é vantajoso uma empresa  De forma direta no prompt: “a empresa deseja  Realizar análise de atratividade do mercado,
entrar com um produto (novo ou atual) em um entrar na região X” ou “a empresa pretende lançar buscando entender o potencial de receitas e lucros
mercado (conhecido ou não). o produto Y”. da empresa analisada neste mercado.
 Em cases de crescimento de receita, pode-se  Avaliar a atratividade financeira: ROI/ROIC, VPL
considerar uma possível entrada de mercado. (NPV) etc.
 Definir melhor estratégia: M&A, parcerias (joint
venture) ou começar do zero.

 Tamanho e crescimento do mercado.  Sensibilidade a preço. Considerações


1. Cliente final  Segmentos de clientes (região, tipo,  Critérios de satisfação e fidelização.
renda, idade etc.  Disponibilidade para primeira compra. Etapas típicas de Market Entry:
 Estimar o tamanho do mercado.
 Mercado fragmentado ou concentrado?  Força da marca.  Cálculo do volume de vendas
2. Concorrentes  Market-share dos principais players.  Possível reação a entrada necessárias para alcançar o break even.
 Atual rentabilidade do mercado  Definir melhor segmento de atuação ao
analisar tamanho, crescimento, número
de concorrentes e lucratividade.
 Situação financeira para investimentos.  Posicionamento relevante nos  Sugerir estratégia de market entry para
3. Empresa  Expertise em entrada de mercado. principais canais de distribuição. casos que envolvem a necessidade de
 Capacidade de produção e logística.  Mix de produtos ideal para o mercado. aumento de receita.
 Definir forma de entrada mais lucrativa.
 Análise financeira e dos riscos das 3  Começar do zero.
4. Estratégia de entrada principais formas de entrada e da  Parceria.
possibilidade de não entrar.  M&A.

Clube de Consultoria UFC 37


Fonte: www.preplounge.com / https://www.youtube.com/watch?v=5LyXsn1rN7M&t=3s Feedback aqui!
Typical Framework: M&A
1. Objetivo 2. Como Identificar 3. Aplicação
 Merger & Acquisition (M&A). Utilizada em  No prompt do case é comum observar frases do  Realizar valuation da empresa alvo e comparar com
situações em que uma empresa deseja avaliar a tipo “O cliente deseja saber se a compra ou venda valor solicitado.
possibilidade de fundir-se ou comprar outro player de determinada empresa é um bom negócio.” ou  Analisar sinergias ou oportunidades de crescimento.
do mercado. Em geral, entender se o valor “Qual o valor que valeria a pena Empresa X  Indicadores financeiros, como ROI/ROIC, podem
criado, ao longo prazo, da compra do negócio oferecer pela empresa Y?”. auxiliar na decisão de viabilidade do negócio.
será relevante em relação ao valor da aquisição e  Considerar riscos independente da recomendação.
a outras possíveis oportunidades. Ex: Integração, reação do mercado etc.

 Tamanho e crescimento do mercado


1. Mercado (empresa por segmento.
 Barreiras de entrada. Considerações
 Reação da concorrência.
alvo)  Principais de concorrentes e share (%).
 Regulamentação do mercado.  Casos de fusões, normalmente, irão ser
 Rentabilidade do mercado.
de empresas de setores que possuem
 Saúde financeira: lucro e fluxo de caixa. uma quantidade relevante de sinergias.
 Ativos importantes.  Casos de aquisições por empresas de
2. Empresa Alvo  Valuation vs valor solicitado  Cultura da empresa alvo / comprador
 Market-share e tendência. setores diferentes da alvo, normalmente,
irão abordar empresas com alto potencial
de crescimento, como empresas
 Experiência no setor de atuação e com  Disponibilidade financeira para compra
familiares (pouco profissionalismo) ou
3. Empresa Compradora outras aquisições. e investimentos após a compra.
startups. Empresas compradoras podem
 Poder de barganha na compra.  Outras oportunidades de crescimento.
ser private equity ou venture capital.
 É uma boa prática desenvolver os
 Valor das entidades individuais e  Riscos do M&A: relacionados ao tópicos de acordo com o problema, como
4. Sinergias e combinadas. mercado e à integração. citar principais sinergias e riscos
Riscos  Sinergias de custos e receitas.  Risco de não realizar o M&A. específicos do caso analisado.

Clube de Consultoria UFC 38


Fonte: www.preplounge.com / www.spencertom.com / Case in Point 2013 Feedback aqui!
Typical Framework: Price Strategy
1. Objetivo 2. Como Identificar 3. Aplicação
 Utilizar uma nova estratégia de preço para um  De forma direta no prompt “A empresa Y deseja  Compreender a meta ou objetivo da empresa: alta
produto/serviço existente. Encontrar qual saber qual deve ser o preço do produto X”. rentabilidade, ganhar participação de mercado etc).
estratégia de preço será a mais lucrativa devido à  Geralmente, é necessário definir o preço do  Utilizar precificação baseada em valor. Os outros
variação do volume de vendas (elasticidade da produto em cases de valuation de patentes. métodos devem ser complementares.
demanda).  Em certos cases de market entry, principalmente,  Considerar elasticidade da demanda (preço vs.
 Precificar um novo produto/serviço. para lançamentos de novos produtos/serviços, volume). Ex: encontrar o preço ideal para garantir o
requerem uma análise de precificação. volume de vendas mais lucrativo para o negócio.

 Benefícios e valor do produto/serviço para o cliente final. Considerações


1. Baseado em valor  Existente: valor extra ($) da proposta em relação ao preço do produto existente.
 Novo: valor economizado ($) da alternativa mais próxima com o novo produto.  A precificação deve estar de acordo com
a estratégia da empresa, visto que um
produto pode ser vendido com margem
 Definir quais os custos da operação. zero, por exemplo, para promover um
2. Baseado no custo
 aplicar uma margem desejada ou uma margem comum da determinada indústria. crosselling lucrativo ou para prejudicar
um concorrente.
Exemplos (baseada em valor):
 Analisar preço cobrado pela concorrência (benchmarking).  Ex1: pneu indestrutível com vida útil de
3. Baseado no
 Diferenciais da oferta para definição de preço de acordo com o segmento de 50 anos possui um valor para o cliente
concorrente
atuação. Ex: pode-se cobrar mais por produtos premium. referente à economia com outros pneus
que seriam comprados no período.
 Capacidade de produção e distribuição.  Ex2: Cliente está disposto a pagar X por
4. Empresa  Possibilidade de crosseling com atual portfólio (venda casada). uma camisa, pois ele acredita que ela
 Estratégia de longo prazo. vale X (os motivos podem ser diversos).

Clube de Consultoria UFC 39


Fonte: www.preplounge.com / hackingthecaseinterview.thinkific.com Feedback aqui!
4.1 VIABILIDADE FINANCEIRA 41
4.2 DRE e P&L 42
4.3 TERMOS RELEVANTES 43
4.4 IND. SOCIOECONÔMICOS 47

4. Conhecimentos
Relevantes
40
Viabilidade Financeira
O Valor Presente Líquido (Net Present Value) é um método Fórmula de Cálculo:
que permite, dada a taxa juros apropriada (taxa de desconto), n – tempo do projeto
unificar o fluxo de caixa de um investimento para o período zero. r – taxa de desconto
NPV – Net Se o projeto apresentar Valor Presente Líquido maior que zero, (custo de capital)
Present Value ele será economicamente viável. t – variável de tempo
Obs: os valores
Em cases, utiliza-se o EBTIDA como uma proxy do fluxo de caixa Saiba mais sobre dinheiro no investidos estão sendo
da empresa para encontrar o valor presente (Receita – Custos). tempo e valuation. Clique aqui! considerados nos custos.

Payback: Tempo necessário para recuperar um investimento Fórmula de Cálculo:


inicial realizado. Pode-se comparar o payback entre investimentos Investimento inicial
para saber qual o mais relevante. Payback =
Payback e Saldo médio do fluxo de caixa no período
Break even Custos fixos
Break even: Equilíbrio entre receita e custos. O Breakeven point Break even Point =
representa a quantidade de unidades vendidas para que a receita (Receita por unidade − Custo por unidade)∗
seja correspondente aos custos. *Margem de contribuição

Retorno sobre o Investimento (ROI): Avalia o retorno sobre um Fórmula de Cálculo:


investimento específico. É dado pela razão entre o lucro obtido e
o investimento realizado. Deve-se definir um período de análise Receitas − Custos Lucro líquido do período
ROI – Return On para contabilizar os lucros do período.
ROI =
Investimento ROI =
Investimento 𝐼𝑛𝑖𝑐𝑖𝑎𝑙
Investiment Retorno sobre o Capital Investido (ROIC): Semelhante ao ROI,
porém avalia o retorno sobre o capital total investido. É dado pela Lucro 𝑑𝑜 𝑝𝑒𝑟í𝑜𝑑𝑜
ROIC =
razão entre o lucro operacional reduzido dos impostos e o capital Capital total investido
total aplicado. OBS: Fórmulas simplificadas
Clube de Consultoria UFC
Fonte: Business Insider / Investopedia / https://www.youtube.com/watch?v=PMENh4UWr-Y&t=318s
41
Feedback aqui!
DRE / P&L
A DRE - Demonstração do Resultado do Exercício (Income Statement) é um relatório que oferece uma síntese econômica completa das atividades de uma
organização, demonstrando claramente se há lucro ou prejuízo de acordo com a contabilidade. O mais comum em cases são os demonstrativos de lucros e perdas (P&L –
Profit and Loss statement), os quais possuem lógica de análise similar à DRE, todavia, pode-se analisar apenas um produto ou departamento específico.

Aplicações Income Statement (DRE)


(+) Revenue (Receita)
A DRE e o P&L são fontes ricas de dados para elaboração de indicadores
financeiros: (-) Sales Returns and Allowances (Devolução e Impostos s/ Venda)
(=) Net Sales (Receita Líquida)
𝐿𝑢𝑐𝑟𝑜 𝐿í𝑞𝑢𝑖𝑑𝑜 (-) Cost of Goods Sold (Custo da Mercadoria Vendida)
Lucratividade (%) 𝑥 100
𝑅𝑒𝑐𝑒𝑖𝑡𝑎 𝐵𝑟𝑢𝑡𝑎 (=) Gross Profit (Lucro Bruto)

(-) SG&A - Sales, General & Administrative Expenses (Despesas)


𝐹𝑎𝑡𝑢𝑟𝑎𝑚𝑒𝑛𝑡𝑜 𝐵𝑟𝑢𝑡𝑜 (=) EBITDA
Ticket Médio
𝑇𝑜𝑡𝑎𝑙 𝑑𝑒 𝑉𝑒𝑛𝑑𝑎𝑠 (-) Depreciation, Amortization and Exhaustion

(-) Other Income and Expenses (Outras Receitas e Despesas)


𝐿𝑢𝑐𝑟𝑜 𝐵𝑟𝑢𝑡𝑜 (=) Net Income Before Taxes (Lucro antes dos impostos)
Margem Bruta (%) 𝑥 100
𝑅𝑒𝑐𝑒𝑖𝑡𝑎 𝑇𝑜𝑡𝑎𝑙 (-) Taxes ( Impostos)
(=) Net Income ( Resultado do Período)

Clube de Consultoria UFC


Fonte: Business Insider / Investopedia
42
Feedback aqui!
Termos Relevantes
Acontece quando uma empresa lança um produto/serviço similar a outro já comercializado por ela e acaba gerando uma concorrência
Canibalização interna. Dessa forma, a organização perde parte das vendas de um produto antigo, podendo ser algo negativo ou positivo.
Negativo: reduzir as vendas de um produto mais lucrativo. | Positivo: evitar a perda de mercado para um concorrente.

A sinergia entre empresas acontece quando a união de duas companhias proporciona uma geração de lucro (ou valor) maior do que a
Sinergia simples soma de lucros das empresas separadas (1 + 1 = 3). Termo comum em casos de M&A. Exemplos de sinergias:
Receita: distribuir um produto reginal para diversos países. | Custos: concentrar compras em 1 fornecedor, gerando economia de
escala.

Fração do mercado controlada por certa empresa. Market Share de valor/receita: tem como base o valor em vendas realizado pela
Market Share empresa dentro do seu segmento em um certo período; Market Share de volume: analisa o volume ou quantidade em unidades
vendidas em relação ao volume gerado pelo mercado em dado período. Ex: receita da empresa / tamanho do mercado.

Relative Market Indicador criado para avaliar a posição de uma empresa em relação ao seu principal concorrente. % 𝒎𝒂𝒓𝒌𝒆𝒕 𝒔𝒉𝒂𝒓𝒆 𝒅𝒂 𝒆𝒎𝒑𝒓𝒆𝒔𝒂
Ou seja, mensurar sua participação relativa no mercado. O RMS do líder do mercado possui como % 𝒎𝒂𝒓𝒌𝒆𝒕 𝒔𝒉𝒂𝒓𝒆 𝒅𝒐 𝒍í𝒅𝒆𝒓 𝒅𝒆 𝒎𝒆𝒓𝒄𝒂𝒅𝒐
Share (RMS) denominador, da fórmula matemática, o 2º maior player do mercado.

Estratégia organizacional que visa a responsabilização de uma empresa sobre as diferentes etapas da sua cadeia produtiva. Com isso,
Verticalização ela passa a produzir e gerenciar, internamente, tudo que puder. Obtendo, assim, um maior controle sobre os processos.
Integração para trás: comprar um fornecedor | Integração para frente: uma fábricante de um produto construir lojas próprias.

Business to Business (B2B): quando a transação de um Business to Consumer (B2C): quando a transação de um produto
B2B | B2C produto ou serviço é feita de uma empresa para outra empresa. ou serviço é feita de uma empresa para o consumidor final.

Clube de Consultoria UFC


Fonte: Business Insider / Investopedia / Rocketblocks
432
Feedback aqui!
Termos Relevantes
Crescimento Promovido de dentro para fora, ou seja, pela assertividade de estratégias de gestão, provocando boas margens de lucro, normalmente,
Orgânico com uma maior lentidão. Ex: lançamento de novos produtos, expansão regional.

Crescimento Aquele que é promovido de fora para dentro, ou seja, por meio de operações de concentração (união de esforços), permitindo às
Inorgânico empresas crescerem com uma maior rapidez. Ex: M&A (fusões e aquisições), Joint Venture.

É um acordo comercial entre duas ou mais empresas, de ramos diferentes ou não, com intuito de desenvolver um projeto em conjunto
Joint Venture de forma temporária. Essa união permite que as empresas colaborarem de formas complementares, compartilhando lucros, custos e
riscos, para a realização de um projeto ou empreendimento. Em certos casos, cria-se um novo CNPJ.

Nome técnico para o estudo avaliativo formal (auditoria) das informações internas e externas de uma organização, normalmente, com
Due Dilligence foco em uma possível negociação, permitindo expor fraudes, erros na operação e pontos críticos. As informações abordadas não se
restringem apenas ao setor financeiro, administrativo e jurídico, pois setores como marketing também podem ser estudados.

É a terceirização do desenvolvimento de produtos e serviços, criando um molde que pode ser personalizado e redistribuído. Na prática,
White Label significa que um item ou plataforma foi desenvolvido por uma empresa, mas recebeu a marca de uma parceira que o revende para seus
clientes, os quais, muitas vezes, não sabe que a marca não é a fabricante. Ex: Nike não fabrica a maioria de seus tênis.

Diluição de É o fenômeno que ocorre quando determinada empresa torna sua marca menos valiosa devido ao não cumprimento das expectativas
do cliente. Pode-se destacar, como exemplo, o lançamento de produtos e serviços que possuem qualidade inferior ao nível esperado ou
Marca que não se encaixam na marca segundo a visão do consumidor. Ex: caso a Apple lance um produto com qualidade não satisfatória.

Clube de Consultoria UFC


Fonte: Business Insider / Investopedia
442
Feedback aqui!
Termos Relevantes
É uma forma de elasticidade cujo objetivo é determinar a sensibilidade da demanda a partir das variações dos preços. Assim, uma certa
Elasticidade da
variação nos preços pode afetar fortemente, ou não, a quantidade vendida. Ela foca no elemento dinâmico do mercado consumidor,
Demanda determinado por preferências e gostos em relação a disponibilidade de gastar com um determinado bem ou serviço.

É o meio de entrega do produto até o cliente final, podendo ser diretamente relacionado com a logística do produto até seu ponto de
Canal de venda. Assim, possui a finalidade de disponibilizar as mercadorias para o consumidor de forma rápida e sem dificuldades para a
Distribuição realização da compra. Ex: loja física (supermercados), marketplaces (Mercado Livre) etc.

Supply chain ou “cadeia de suprimentos” consiste em todo o processo logístico de determinado produto ou serviço, desde a sua
Supply-Chain fabricação até a sua entrega ao consumidor final. O supply chain é constituído por vários integrantes, que atuam em diferentes
etapas durante o processo, como: fabricantes, fornecedores, armazéns, distribuidoras, varejistas e, por fim, os consumidores.

Custo de Define-se por “custo de oportunidade” aquilo que você abre mão para obter algo. Essa definição explicita que, ao tomar qualquer
decisão utilizando recursos escassos, outra possibilidade deixa de ser tomada. Desse modo, deve-se avaliar os custos-benefícios, de
Oportunidade cada escolha, de modo a definir qual é mais conveniente aos critérios do consumidor ou do tomador de decisão.

Net Promoter NPS é uma metodologia de satisfação de clientes desenvolvida para avaliar o grau de
fidelidade dos clientes de qualquer perfil de empresa. É dado pela diferença do
Score %promotores pelo %detradores.

Share of Wallet (SOW) representa a parcela de orçamento que o consumidor investe em determinado estabelecimento, para uma
Share of Wallet categoria de gastos específica. Pode ser entendido como a quantidade de negócios que uma empresa obtém de clientes
específicos. Ex: gasto de R$ 100,00 em refrigerante por mês e R$ 60,00 em Coca-Cola, então SOW = 60%

Clube de Consultoria UFC


Fonte: Business Insider / Investopedia
452
Feedback aqui!
Termos Relevantes
Ocorre quando numerosas empresas (ou marcas) partilham o mercado, de modo que nenhuma empresa ou grupo domina grande
Mercado participação nele (não existe empresas com grande market-share). Normalmente, apresenta maior facilidade de novas entrantes
Fragmentado prosperarem, pois, nenhum concorrente terá força para prejudicar, drasticamente, o novo empreendimento.

Mercado Ocorre quando o mercado está amplamente dominado por uma ou um número reduzido de empresas ou marcas (poucos players com
alto market-share). Mercado difícil para novos entrantes, visto que os grande players possuem muita força no mercado. Monopólio é um
Concentrado caso extremo de mercado concentrado.

Refere-se à empresa que é a única na oferta de um bem, produto ou serviço, de modo que este bem não possui substitutos próximos.
Monopólio É característico de um monopólio o fato de ele não seguir o funcionamento de uma economia competitiva, visto seu grande poder de
mercado, especificamente, sobre os preços dos produtos.

Significa a perda de valor de um bem em decorrência do seu tempo de uso ou desgaste. Para fins contábeis, a depreciação é uma
Depreciação estimativa do gasto periódico relacionado à perda de valor do bem adquirido. Como exemplo de ativos depreciáveis: móveis,
Equipamentos e Instalações.

Representa a perda de valor de ativos intangíveis (como patentes, softwares e direitos autorais) em dado período de tempo. Essa perda
Amortização está atrelada à redução no tempo de contrato ou direito de utilização, devendo ser classificada como um custo ou despesa da
organização.

É uma taxa de progressão geométrica que representa a porcentagem de retorno constante VF - Valor Final
CAGR e anual que um determinado saldo inicial obteve, ou poderá obter, ao longo de um VI - Valor Inicial
N - # de períodos
período. Aplicação: taxas de crescimento de um mercado ou da receita de uma empresa. (anos)

Clube de Consultoria UFC


Fonte: Business Insider / Investopedia
462
Feedback aqui!
Indicadores Socioeconômicos
Produto Interno Bruto (PIB) Distribuição de Renda no Brasil Distribuição de Gastos: Família
O PIB é a soma de todos os bens e serviços PIB Per Capita: R$ 31.833,50 (IBGE 2017) Pode ser vantajoso utilizar distribuição de gastos
finais produzidos por um país, estado ou cidade, Salário Mínimo: R$ 1.045,00 (G1 2020) com a renda média de uma família, com o intuito
geralmente em um ano. Salário Médio Individuo: R$ 2.261,00 (IBGE (2019) de estimar valores gastos em cada categoria.
Renda Média Familiar: R$ 5.460,70 (IBGE 2019)
Cálculo do PIB (demanda):
PIB = C + I + G + (X – M) Saber dados aproximados sobre a distribuição de renda
C = Consumo das Famílias no Brasil pode ser relevante para a resolução de cases
I = Investimento das empresas (principalmente, estimations). OBS: tabela não oficial.
G = Gastos do Governo Sugerimos realizar uma criação própria.
(X-M) = Balança Comercial
Classe Renda % da Renda
Social Familiar Pop. Indivíduo
PIB’s 2019 (valores aproximados)
R$
A R$ 16.000,00 5%
PIB Mundo (nominal): US$ 85T 10.000,00
PIB Brasil: US$ 1,8T R$
 Per Capita – 8k B R$ 8.000,00 10%
5.000,00
PIB EUA: US$ 21T
R$
 Per Capita – 65k C R$ 6.000,00 45%
2.000,00 Expectativa de Vida
PIB China: US$ 14T
 Per Capita – 10k R$ Brasil: ~76 anos (mulheres-80 | homens-73)
D R$ 3.000,00 25%
1.000,00 EUA: ~79 anos
PIB Europa: US$ 22T
África do Sul: ~63 anos
 Per Capita – 35k E R$ 1.000,00 15% R$ 300,00 Mundo: ~73 anos

Clube de Consultoria UFC


Fonte: FGV Social 2014 / g1.globo.com/economia/noticia (2021/03/03) / FOB IBGE 2018-2019
47
Feedback aqui!
Indicadores Socioeconômicos
Gastos com Funcionário Grandes Empresas Populações
O salário líquido representa, normalmente, Mundo (US$) / Brasil (R$) Países Mundo
entre 30 e 70% dos custos totais de um Valor de Mercado | Receita: Mundo: 7,7T
funcionário. No geral, a representatividade Apple (Tecnologia/Eletrônicos): 1° China: 1,44T
diminui quanto maior for o salário. US$ ~2,3T (2020) | ~US$ 260B (2019) 2° Índia: 1,38T
Amazon (Tecnologia/Varejo): 3° Estados Unidos: 330M
US$ ~1T (2020) | ~US$ 390B (2020) 4° Indonésia: 267M
Custos JPMorgan Chase (Financeiro): 5° Paquistão: 221M
US$ ~400B (2019) | US$ ~140B (2018) 6° Brasil: 211M
Salário Encargos Auxílio e Toyota Motor (Automotivo):
Outros
líquido sociais benefícios US$ ~200B (2020) | US$ ~280B (2019) Cidades Brasil
INSS Transporte Treinamentos Vale (Mineração): 1° São Paulo-SP: 12M
FGTS Alimentação Hora extra R$ ~320B (2019) | R$ ~110B (2017) 2° Rio de Janeiro-RJ: 7M
13º Plano de Comissões Petrobras (Óleo e Gás): 3° Brasília-DF: 3M
Férias saúde Uniformes R$ ~180B (2020) | R$ ~300B 4° Salvador-BA: 2,9M
Seguro de Ausências Ambev (Alimentos e Bebidas): 5° Fortaleza-CE: 2,7M
vida R$ ~290B (2019) | R$ ~46B (2016)

Outros dados relevantes


 População possui maior tendência de envelhecimento em países desenvolvidos.
 Países desenvolvidos possuem, em média, menos que 3 de pessoas por família/residência. Enquanto países em desenvolvimento a média é próxima de 4.
 Existem um pouco mais de 1 bilhão de carros no mundo.
 36% da população mundial está em níveis abaixo da pobreza. Em países em desenvolvimento, esse número é cerca de 50%.
 A pandemia COVID-19 proporcionou resultados positivos para alguns mercados, como o de tecnologia, e negativos para outros, como o de turismo.
OBS: esses valores podem ser alterados constantemente, sugerimos uma pesquisa individual para garantir a confiabilidade do banco de fatos.
Clube de Consultoria UFC
Fonte: Forbes, 09-2020 / Business Insider, 09-2020 / Yahoo Finance, 2020 / Wikipédia, 2020 / ceoworld.biz/r / www.worldatlas.com/articles
48
Feedback aqui!
5.1 AUTOMOTIVA 50
5.2 VEREJO 51
5.3 COMPANHIAS AÉRIAS 52
5.4 SERVIÇOS DE SAÚDE 53
5.5 FARMAUCÊUTICA 54
5.6 SERVIÇOS FINANCEIROS 55
5.7 PRIVATE EQUITY 56
5.8 ÓLEO E GÁS 57
5.9 TELECOMUNICAÇÕES 58
5.10 MÍDIA E ENTRETENIMENTO 59

5. Resumo das
Indústrias
49
Industria Automotiva
Direcionadores de Receita
Produtos/  Veículos: Carros, Caminhões, Ônibus, Vans, Motocicletas.
 Autopeças e acessórios para substituição ou veículos novos.
Serviços  Demanda fortemente influenciada pelo
crescimento econômico;
 Táticas para aumento de demanda:
Possibilidade de financiamento e aceitar
 B2B: Frota de veículos para transportadoras, indústrias, Governo etc.
Clientes  B2C: Cliente individual de classes econômicas A, B e C.
carro antigo como parte do pagamento;
 Ganhadores de pedido: Preço, Consumo
econômico, Design.
 Tamanho do Mercado no Brasil: U$ 59,1 bilhões. Representa 22% do PIB
industrial e 4% do PIB total (2015).
Mercado  10 principais marcas dominam, aproximadamente, 90% do mercado no Brasil. Estrutura de Custos
Mercado de autopeças e acessórios é muito fragmentado.
Players: Chevrolet (GM), Volkswagen, Fiat, Ford, Hyundai, Toyota, Honda, Nissan.
Fixos:
 Custos de produção (Aluguel, Manutenção,
 Concessionárias. Depreciação...);
Canais  Atacado (Para empresas ou revendedores).
 Varejo físico e online de autopeças e acessórios.
 Logística e distribuição;
 Marketing e vendas;

 Novas tecnologias de utilização de energia, como veículos elétricos e híbridos Variáveis:


(ecológicos e sustentáveis).  Matéria prima (Aço, Vidro, plástico,
Tendências  Diminuição do desejo de posse de um veículo (mobilidade compartilhada).
acessórios...).
 IoT: Expansão da conectividade do veículo envolvendo objetos em volta.
 Comissões

Clube de Consultoria UFC 50


Fonte: https://novovarejo.com.br/a-configuracao-do-setor-automotivo-no-brasil/ Feedback aqui!
Varejo
 Mercadorias em geral. Direcionadores de Receita
Produtos/  Sub indústrias: vestuário, eletrônicos, artigos para casa e escritório, varejo de
Serviços medicamento, automotivo, alimentos (supermercados e hipermercados),
especialidades etc.
 Poder de compra do consumidor: os
resultados do varejo são muito sensíveis às
variações econômicas.
 Índice de confiança do consumidor.
 Cliente individual de diferentes classes econômicas A, B, C, D e E.
Clientes  Público alvo diverge de acordo com a indústria, localização, canais de venda e  Segmentação da população no
outros critérios. comportamento do consumidor (gênero,
idade, renda etc.).
 Promoções: redução de preço, vitrines,
 As 120 maiores empresas faturam R$ 345 bilhões em 2018.
 Categorias que se destacam: Hiper & Supermercados, Bebidas e Fumo (33,8% fachadas chamativas etc.
Mercado das vendas do varejo), Veículos, Motos, Partes e Peças (25,9%) e Tecidos,
Vestuário e Calçados (9,5%). Estrutura de Custos
Players: Carrefour, Grupo Pão-de-Açúcar, Via Varejo, Walmart.
Fixos:
 Presencial: lojas, centros comerciais, shoppings, farmácias, livrarias, porta-a-  Custos de funcionamento (Aluguel,
Canais porta etc. Manutenção, Depreciação, etc).
 À distância: sites, aplicativos, televisão, telefone etc.  Logística e distribuição.
 Funcionários (salários e outros).
 Automatizações e padronizações de processos diminuem a necessidade de
funcionários. Variáveis:
Tendências  Diminuição de margens de produtos não inovadores devido a promoções e
 COGS (geralmente, são os maiores
guerras de preço.
 Crescimento acelerado do e-commerce, principalmente, devido à pandemia. custos).
 Vendedores (comissões).
Clube de Consultoria UFC  Marketing e vendas. 51
Fonte: IBEVAR, 2019 Feedback aqui!
Companhias Aéreas
Direcionadores de Receita
Produtos/  Aeronaves: transporte aéreo de passageiros.
 Transporte de cargas.
Serviços  Serviços de bordo (comida, bebida, bar
exclusivo e outros serviços para 1ª classe);
 Vendas de passagens;
 B2B: Transportadoras de carga/mercadoria, Correios, sites e aplicativos de  Taxas de excesso de bagagem;
viagens (ex: CVC, Decolar.com).
Clientes  Escolha de poltrona;
 B2C: Passageiros individuais a lazer ou negócio, aluguel de
aeronave/helicóptero.  Sobretaxas de combustível.
Estrutura de Custos
 Aviação comercial brasileira contribuiu com 1,4% (R$ 103,4 bilhões) do PIB
nacional em 2019, segundo Associação Brasileira das Empresas Aéreas.
Mercado  Serviço oferecido na internet possibilita menor custo fixo, como o relacionado à Diretos:
loja.  Depreciação/Aluguel de aeronaves e
Players: American Airlines, Lufthansa, Qatar Airways, Emirates. equipamentos de vôo;
 Alimentação e serviços gerais;
 Call center.  Manutenção, revisão e seguro;
Canais  Atacado para agentes e agência de turismo.  Salários da tripulação;
 Varejo físico e online (Sites e aplicativos).
 Combustível;
 Voos cancelados.
 Mais cia aérea low-cost e valores de passagens caindo.
 Veículo autônomo de aeronave com motor elétrico e sem a necessidade de Indiretos:
Tendências piloto.
 Taxa de aeroporto/ instalações,
 Escassez de caminhoneiros fazendo com que a demanda por carga aérea seja
maior. infraestrutura adequada;
 Marketing e ações de divulgação;
Clube de Consultoria UFC  Salários do pessoal em terra. 52
Fonte: www.abear.com.br. Feedback aqui!
Serviços de Saúde
 Farmacêutica – patente, medicamentos de venda livre (sem receita), genéricos. Direcionadores de Receita
Produtos/  Biotecnologia – tratamentos genéticos, probióticos.
 Instalações e equipamentos de saúde – curativos, luvas, aparelhos cirúrgicos.
Serviços  Assistência médica – consultas médicas, seguro e plano de saúde.  A lucratividade depende de operações
eficientes e, em alguns casos, subsídios e
fundos federais.
Oportunidades de crescimento: pandemias,
 B2B: Hospitais, farmácias, indústrias, governo e etc. doenças modernas, fusões , convênios e
Clientes  B2C: Cliente individual e pacientes de todas as idades. etc.
Fatores determinantes na contratação:
qualidade, preço, consumo, fatores
 Alta regulamentação e barreiras de entrada; alteração na regulamentação (por culturais e etc.
exemplo, devido a uma mudança de governo).
Mercado  O Brasil possui altos gastos com saúde, representando 9,1% do PIB (2019).
Estrutura de Custos
 Mercado altamente fragmentado.
Players: Siemens, Johnson e Johnson, Medley, Fleury, Sírio Libanês.
Fixos:
 Hospitais e Consultórios médicos.  Custos administrativos: marketing,
 Casas de repouso. faturamento/cobrança, atendimento ao
Canais  Centros de cirurgia ambulatorial. cliente etc.
 Farmácias.
 Custos médicos diretos: profissionais da
saúde, serviços ambulatoriais, hospitais.
 Equipamentos modernos de alta performance.  P&D.
 Aumento contínuo na demanda (envelhecimento demográfico e busca por
Tendências melhor qualidade de vida)
 Terapia gênica Variáveis:
 Dispositivos inteligentes e banco de dados.  EPI, medicamentos, equipamentos
cirúrgicos descartáveis etc.
Clube de Consultoria UFC 53
Fonte: fehoesp360.org.br. Feedback aqui!
Industria Farmacêutica
 Marca original de medicamentos protegida por patente, durante um certo Direcionadores de Receita
Produtos/ período de tempo, para seres humanos ou animais.
Serviços  Drogas genéricas (mesmo resultado médico) com menor custo de
desenvolvimento vendidas após acabar o período da patente original.
 Sazonalidade grande para certos produtos
(ex: vacinas e remédios para resfriado).
 Compromisso dos médicos prescreverem
 Pacientes/clientes que precisam de medicamentos. os medicamentos (variação da dosagem e
 Médicos que prescrevem os medicamentos. frequência de uso).
Clientes  Clínica/Hospital e empresas de seguro saúde que pagam medicamentos.  Efetividade do medicamento e efeitos
 Governo. Distribuição para doenças ou públicos específicos
colaterais influenciam fortemente a decisão
de compra do consumidor.
 Mercado relativamente fragmentado: 10 maiores empresas brasileiras são
responsáveis por cerca de 40% do faturamento do mercado.
Mercado  Lucro das patentes deve compensar os investimentos em pesquisas. Estrutura de Custos
 ANVISA – BR / FDA-EUA: Precisam aprovar todos os medicamentos.
Players: Nc Farma, Sanofi, Bayer, Eurofarma, Hypermarcas, Novartis.
Fixos:
 Custos de fábrica (Aluguel, depreciação
 Distribuidores/Intermediários; de equipamentos e instalações, etc).
Canais  Farmácias; Hospitais/Clínicas (revenda em pontos de venda ou online)  Salários: Operação e gerência.
 Governo. Escolha e controle do canal de distribuição utilizado  P&D (desenvolvimento e aprovação).
 Logística e distribuição.
 Aumento da atuação dos farmacêuticos (prescrição, vacinação, etc)  Marketing e vendas.
 Pressão de seguradores e redes de hospitais para reduzir preços.
Tendências  P&D busca desenvolver drogas/vacinas mais rápido (ex: COVID-19) Variáveis:
 Aumento da utilização de nanotecnologia e medicamentos biológicos.
 Matéria prima.
 Embalagens e rótulos.
Clube de Consultoria UFC 54
Fonte: IMS World Review, 2015 Feedback aqui!
Serviços Financeiros
Direcionadores de Receita
Produtos/  Crédito: Empréstimos, Cartões de Crédito, Financiamentos.
 Investimentos.
Serviços  Seguros.  Rendas de Operações de Crédito (Juros).
 Rendas de Títulos e Valores Mobiliários e
Compulsório (Juros).
 B2B: Empresas de qualquer mercado e até mesmo outras instituições  Receita de Serviços: Tarifas,
financeiras.
Clientes Administração de Fundos, Mercado de
 B2C: Pessoas físicas que necessitam de contas bancárias, realizar
investimentos, seguros, entro outros.
Capitais.

 Os bancos compõem o setor mais lucrativo do país. O volume de ativos totais


das instituições financeiras atingiu R$ 7,36 trilhões ao fim de março e superou o
Mercado PIB brasileiro de 2019.
Estrutura de Custos
Players: Itaú Unibanco, Bradesco, Banco do Brasil, Santander, NUBank, Banco
Inter, Industrial and Commercial Bank of China, JP Morgan Chase, HSBC.  Aluguel de infraestrutura: agências,
escritório etc.
 Call center.  Custo de Créditos
Canais  Agências e Caixas Eletrônicos.  Marketing e ações de divulgação.
 Internet Banking, Aplicativos.
 Salários e benefícios da equipe
operacional e dos executivos.
 PIX e Open Banking melhoram a experiência do usuário, aquecem a economia  Custo de TI.
e aumentam a competitividade do mercado.  Pesquisas para investimentos e
Tendências  Aumento da bancarização da população devido ao COVID-19. oportunidade.
 Novas tecnologias. Ex: Inteligência Artificial, Blockchain, Big Data etc.  Risco de inadimplência.
 Crescimento de Bancos Digitais, devido a menores burocracias e taxas.

Clube de Consultoria UFC 55


Fonte: https://www.infomoney.com.br/economia Feedback aqui!
Private Equity
 Ações/empresas que não são negociadas publicamente. Direcionadores de Receita
 Aquisições Alavancadas (Leveraged Buyout- LBO): Pedir empréstimo para
Produtos/
investir e usar como garantia as ações da empresa.
Serviços  Distressed Investments: Empresas em crise, mas com potencial.  Taxa de administração.
 Capital para crescimento.  Taxa de performance: Dependente da
estratégia e da equipe operacional.
 Fundos de pensão. % do capital da empresa, que é revendido
 Fundo de fundos (FoF). após a empresa se valorizar.
Clientes  Investidores individuais qualificados ou profissionais.
Saída do investimento por abertura de
 Outros.
capital (IPO), normalmente, apresenta os
 Compra de participação (equity) em empresas de capital fechado, com potencial melhores retornos.
de valorização através de estratégia e gestão profissionais, para posterior venda.
Mercado  Necessidade de capital alta para negociações, normalmente, com baixa liquidez.
 Venture Capital (VC): Mercado a parte com foco em empresas em estado inicial. Estrutura de Custos
Players: Patria Investimentos, General Atlantic, Advent International.
 Custos administrativos em geral: viagens
 Redes de contatos do fundo. para negociação, energia, aluguel,
 Avaliação de bancos. contabilidade, etc.
Canais  Corretoras.
 Salários (alta despesa, devido à grande
 Boca-a-boca de investidores.
especialização dos profissionais).
 Regulação fiscal, impostos, etc.
 Continuar entregando retornos superiores aos do mercado financeiro em geral.
 Custo do investimento em empresas para
 Popularização do modelo de negócio de privaty equity e venture capital.
Tendências  Crescimento no mercado de Private Equity goblal (CAGR 11%, 2020 - 2025), reestruturação/alavancagem.
mesmo no período pós-COVID.

Clube de Consultoria UFC 56


Fonte: Mordor Intelligence, 2020 Feedback aqui!
Indústria Óleo e Gás
 Combustível: gás natural, GNV, transportes e óleo diesel marítimo, querosene Direcionadores de Receita
Produtos/ de aviação, óleo de xisto.
Serviços  Bens de Consumo: fertilizantes, plásticos, borrachas sintéticas, etc.
 Industriais: matéria-prima, como o nafta para borrachas.
 Crescimento econômico e produção
industrial elevam a necessidade desses
produtos.
 B2B: refinarias, revendedoras de combustíveis, indústrias, empresas, governo e  O preço é altamente influenciado por
etc. acordos geopolíticos (ex: OPEP) e pela
Clientes  B2C: consumidores de combustíveis, consumidores dos derivados (plástico, oferta global.
lubrificantes).
 Empresas especializadas em perfuração
fornecem plataformas para a realizar a
 Alto impacto no PIB nacional e forte jogo político. operação, alavancando a exploração do
 Grandes barreiras de entrada: Altos Custos de entrada, altos custos
óleo/Gás.
Mercado operacionais fixos, tecnologia desenvolvida, regulamentações governamentais
e normas ambientais. Estrutura de Custos
Players: Chevron, Exxon, BP, Total, Saudi Aramaco e Petrobras.
 Upstream: Custos exploração (procurar).
 Companhias de bens de consumo. Custos de extração: Instalação e
 Posto de combustível. manutenção de plataformas, licença,
Canais  Contratos de venda de combustível (ex: Aviação). brocas, salários e encargos
 Contratos de venda de matéria-prima.
 Mindstream: Custos de transporte e
logística: aquisição e manutenção de
 Aumento da preocupação em reduzir o impacto ambiental. Criação de caminhões, ferrovias, barcas e oleodutos.
regulamentações para reduzir a emissão de gases (ex: Acordo de Paris).
 Downstream: custos industriais de
Tendências  Aumento da utilização de energias renováveis, como alternativa.
 Combustível perde força no mundo pós-pandemia com o crescimento do home
transformação em combustível ou
office. produtos acabados: destilação,
refinamento etc. Custos comerciais e de
Clube de Consultoria UFC canais de distribuição. Ex: postos.
57
Fonte: energyhq.com / www.investopedia.com Feedback aqui!
Telecomunicações
 Produção de Hardware (cabos de fibra óptica, satélites de transmissão, entre Direcionadores de Receita
Produtos/ outros).
Serviços  Produção de Software (acesso à internet e sinal de telefone).
 Serviços intermediários (manutenção e gestão da rede).
 A receita vem de assinaturas, serviços de
pagamento por minuto, taxas de acesso à
rede e publicidade.
 O crescimento mais rápido vem de
 B2B: Empresas e governo. serviços fornecidos por redes móveis.
Clientes  B2C: Cliente residencial e individual.
 A receita de publicidade representa cerca
de 5% da receita total da indústria.
 Tamanho do Mercado no Brasil: U$ 9,9 bilhões.
 4 principais marcas dominam, aproximadamente, 90% do mercado no Brasil. Estrutura de Custos
Mercado  Grandes barreiras de entrada devido, principalmente, à necessidade de capital
intensivo para iniciar o negócio.
Players: Telmex (Claro), Embratel e NET, Oi e Telefônica Brasil (Vivo). Direto:
 Equipamento e Matéria-prima.
 Infraestrutura (torres, cabos e antenas).
 Varejo físico (lojas, quiosques, call centers).  Instalação da rede.
Canais  Varejo online (marketplace, e-commerce, aplicativo).
 Manutenção de rede.

Indireto:
 Crescimento Mobile e diminuição da telefonia fixa.  Marketing e vendas.
Tendências  Aumento da velocidade da internet.
 Salários: operação e gerência.
 Entrada em vigor da Lei Geral da Proteção de Dados (LGPD).
 Gerais e Administrativos.
 P&D (pesquisa e desenvolvimento).
Clube de Consultoria UFC 58
Fonte: www.telesintese.com.br / www.terra.com.br/noticias Feedback aqui!
Mídia e Entretenimento
Direcionadores de Receita
Produtos/  Programas de TV, filmes, jornais, música, notícias, videogames, livros, revistas,
Serviços programas de rádio.
 Assinaturas: cobranças periódicas para
acesso aos serviços. Modelo de negócio
“freemium” ou com período de carência
atraem mais clientes.
 Modelos tradicionais: potenciais anunciantes.  Anúncios/publicidade para exposição de
Clientes  Modelos de assinatura: consumidores individuais. anúncios e/ou aumento de visitas.
 Alcance de público e circulação são
relevantes para definir os valores a serem
 Criação, licenciamento e distribuição de conteúdo, buscando atender o interesse
cobrados pela divulgação.
da audiência para aumentar receita.
Mercado  Mercados de mídia online e televisa relativamente concentrados: 4 principais  Licenciamento/distribuição de imagem ou
players de cada mercado somam 58,75% e 71,1%, respectivamente, no Brasil. marca.
Players: Grupo Globo, UOL, América Móvil, Saraiva, YouTube, Netflix, Spotify.
Estrutura de Custos
 Produto impresso ou online.
 Infraestrutura: Estúdios, impressoras,
 Transmissão tradicional, a cabo, via satélite ou móvel.
Canais  Cinemas tradicionais. equipamentos de filmagem, hardware etc).
 Plataformas de streaming.  Marketing e publicidade.
 TI: Desenvolvimento e manutenção de
 Digitalização da mídia. softwares (Programas, Web, app etc).
 Maior difusão de conteúdos gratuitos como forma de atrair consumidores.  Hospedagem em plataformas online.
Tendências  Diminuição dos preços dos anúncios devido à diversidade de opções.  Licenciamento de conteúdo ou Royalties.
 Facilidade para pirataria.
 Plataformas OTT (Over the top).  Salários: Principalmente da equipe de
áreas criativas e/ou de geração de
Clube de Consultoria UFC conteúdo. 59
Fonte: Spbancarios, 2018 Feedback aqui!
6.1 Manhattan Boom! 62
6.2 Easy Money 67
6.3 La Gatta Gelato 72
6.4 Vale do Rio Doce 77
6.5 Enfermeiro Anjo 85
6.6 Tele Fortal 93
6.7 Cards for Everyone 100
6.8 Where’s the Party 107
6.9 Safe Car 120
6.10 Greta Coffehouse 126
6.11 BR Rodovias 133
6.12 Mobi Clean 141
6.13 Organ Donation 147
6.14 SpaceX – Terra Nova 155
6.15 The Moto Perpetuo 163
6. Cases
60
Nível de Complexidade dos Cases
É recomendável (não essencial) que a sequência Essa classificação não é absoluta, logo alguns
de aprendizado de resolução de cases passe por cases podem ser enquadrados em mais de um 6. Ambíguo
esses 6 níveis em ordem crescente, com o intuito nível de complexidade.
de catalisar a evolução. 5. Estratégia
Aberta
OBS: Complexidade não significa facilidade. Ex:
podem existir estimations mais difíceis que cases
4. Decisão Partner Cases.
ambíguos, por exemplo, mas não é comum. Estratégica Foco em
3. Analisar Entender o
Diagnóstico possíveis pensamento do
2. Decisão decisões e suas candidato com
Decisão de temas avulsos
Analítica realizar ou não relações (long-
term) (a resposta em
Buscar causa uma ação si é pouco
1. Mini Case Ex1: Como crescer
raiz ou priorizar Ex1: Devemos entrar
receita (próximos 10 relevante)
uma ação no mercado
Uma análise brasileiro? anos)? Ex1: O que fazer
principal define (short-term) Ex2: Devemos Ex2: Como aumentar com um ovo de
o número de turistas dinossauro?
Estimations + o case (lucro é Ex1: Lucro caiu comprar esta
em Fortaleza-CE?
durante os últimos empresa? Ex: Qual a curva de
Brainstorming o mais comum) anos, o que fazer? inadimplência de
Ex1: Concessão de uma empresa de
Ex1: # de barbeiros Ex2: Como aumentar
um aeroporto? seguros?
em Fortaleza-CE receita no próximo
Ex2: Formas de Ex2: Vale a pena ano?
reduzir custos de aumentar nossos
uma hamburgueria preços?

Clube de Consultoria UFC 61


Fonte: https://open.spotify.com/episode/0cpF5tHRpJ4PTySOMnlsw7?si=2EwG4lmPTsSR3zbrAFIsgA&context=spotify%3Ashow%3A3vvpOP5U5Uc44Y0bcgWrsF Feedback aqui!
1. Mini-Case

Market Sizing
Indústria: Setor Público

Led By: Interviewer

Quantitativo Qualitativo

Case 1
Manhattan Boom!
Clube de Consultoria UFC 62
Case 1: Manhattan Boom!
Introdução
Contexto: Você é atualmente o consultor número 1 dos Estados Unidos, após ter trabalhado durante 15 anos em uma grande consultoria de estratégia. Algo estranho acabou
de acontecer, um pequeno meteoro caiu em Manhattan (uma ilha). O meteorito é pequeno e por sorte ninguém foi machucado, pois ele caiu em um local pouco movimentado
do Central Park. Todavia, o meteorito está exalando um gás esverdeado que possui efeitos ainda desconhecidos relacionados ao prejuízo da saúde humana.

Situação: Você foi convocado para uma reunião urgente por chamada de vídeo com pessoas altamente influentes, incluindo o Presidente e os principais chefes militares dos
EUA, com o objetivo de resolver este problema da forma mais rápida possível.

Problema: O general chefe da marinha ressaltou a importância de evacuar toda a população de Manhattan o mais rápido possível e o Presidente dos EUA perguntou para
você: “em quanto tempo conseguiríamos tirar todas as pessoas da ilha, com segurança, contando com o apoio de todos da reunião?”.

1. Brainstorming: Primeiramente, quais os meios de transporte poderiam ser utilizados para evacuar toda a população?
Clarify (Informações adicionais, caso solicitadas)
 Manhattan é uma ilha nos EUA que possui aproximadamente 1,7 milhões de habitantes
 O meteoro prejudicou uma parte do sistema elétrico da cidade, não sendo aconselhável a utilização de metrô
 Tempo de preparação para saída é de 1 hora (preparação e comunicação para toda a população)
 É necessário considerar que sair da ilha é estar em outro ambiente térreo

Guia para o Entrevistador


 Direcionar o candidato, primeiramente, para um brainstorming de quais meios de transporte que poderiam ser utilizados, mesmo que ele já tenha identificado que é uma
pergunta de estimation.

Mandou bem! Pisou na bola...


 Clarificar sobre os possíveis meios de transporte.  Pedir tempo para estruturar sem clarificar com entrevistador.
 Realizar brainstorming estruturado dos meios de transporte.  Não citar meios de transporte comuns (Carro, ônibus, navio etc).
 Perceber que a evacuação terrestre terá como gargalo as pontes da ilha.

Clube de Consultoria UFC 63


Feedback aqui!
Case 1: Manhattan Boom!
1. Brainstorming: Primeiramente, quais os meios de transporte poderiam ser utilizados para evacuar toda a população?

Sugestão de Abordagem: Informações Adicionais (Caso solicitadas):

Marinha irá providenciar inúmeros navios eficientes.


Ponte (Carro, Ônibus,  5 pontos para grande navios da marinha, sendo
moto)
# Pessoas cada um com capacidade de carregar 600
Terra pessoas/30min
Manhattan
Metrô Aviões não terão uma grande eficiência pela
dificuldade logística de aterrissagem
 3400 pessoas/hora.
# Horas
Barcos, lanchas etc Helicópteros terão tempo de trajeto semelhante aos
navios, pois será utilizado, principalmente, por
# Pessoas / hora Água pessoas com deficiências ou internadas em hospitais.
Número infinito de helicópteros disponíveis
Navio  50 heliportos/helipontos (50 voos
concomitantes)
 30 min de trajeto
 40 pessoas/ helicóptero em cada trajeto
Avião
Guia para o Entrevistador
Ar  Guiar para próxima etapa ao ser levantado o
assunto de evacuação por pontes
Helicóptero  Outras formas de evacuação, que não foram
comentadas, podem ser desconsideradas

Clube de Consultoria UFC 64


Feedback aqui!
Case 1: Manhattan Boom!
2. Estimation: Qual a vazão de pessoas por hora por todas as pontes (evacuação terrestre) de Manhattan?

Sugestão de Abordagem:

Comprimento total – 2 km x 4
# Pontes 6 disponíveis 1.000 carros pistas (2 + 2) 8.000m

Cap. Máxima
900

Comprimento de 1 8 metros
156.600 pessoas/h # Veículos na ponte (Média:
carro
carros e
4500 ônibus)
Pessoas/hora % Utilização 90%

# Veículos/ hora

1 hora
26.100 5

# Pessoas / Ponte # Vezes que 1


atravessa / hora Distância (km) 2km
0,2 horas

Tempo médio p/
Informações Adicionais: atravessar (h)
 Dados na página posterior; Tabela 1
# Pessoas / Veículo 5% de ônibus  40 pessoas
 Criar assumptions para os dados não que Vel. Média (km/h) 10km/h
não foram disponibilizados. 95% Carros  4 pessoas

Clube de Consultoria UFC 65


Feedback aqui!
Case 1: Manhattan Boom!
2. Estimation: Qual a vazão de pessoas por hora por todas as pontes (evacuação terrestre) de Manhattan?

Sugestão de Abordagem: Informações Adicionais (caso solicitadas):


 Considerar 6 pontes
Tabela 1 - Sugestão de ponderação da % de carros e ônibus.  Cada ponte possui em média 2 pistas de ida e 2 pistas de
volta, contudo, devido à situação emergencial, pode-se
# de considerar que as 4 serão utilizadas para realizar a
Veículo % representativa # carros/hora # pessoas/hora
Pessoas evacuação
 Caso solicitado, deixar o candidato arredondar valores
Carro 4 95% 0,95*4500 = 4275 4275*4 = 17.100 para facilitar cálculos

Ônibus 40 5% 0,05*4500 = 225 225*40 = 9.000 Guia para o Entrevistador:


 Caso o candidato não tenha dificuldade de realizar a
estruturação do estimation, deixe ele fazer todas as
Cálculo final: assumptions necessárias (não tem problema ser diferente
da sugestão de abordagem, desde que faça lógica)
1 hora  O case finaliza após o candidato informar o número de
11 1,7M horas, sendo a interpretação do número encontrado e a
realização reality check boas práticas se realizadas
# Horas # pessoas
10 Notas:
Ponte 156,6k
# horas de  Bons candidatos irão realizar perguntas clarify sobre os
evacuação 170k meios de transporte e construírão uma estrutura lógica do
Evacuação por estimation de número de carros que passam pela ponte
Navio 6k
hora  Caso o candidato termine rapidamente, sugira uma análise
de riscos que podem aumentar o tempo encontrado
Avião e
helicóptero 3,4 + 4 = 7,4k

Clube de Consultoria UFC 66


Feedback aqui!
1. Mini-Case

Market Sizing
Indústria: Serviços Financeiros

Led By: Candidate

Quantitativo Qualitativo

Case 2
Easy Money
Clube de Consultoria UFC 67
Case 2: Easy Money
Introdução (prompt)
Contexto: Nosso cliente é a startup Easy Money, a qual atua no mercado financeiro e que está iniciando sua operação no Brasil. Seu modelo de negócio consiste em
armazenar dinheiro de seus clientes e fazer esse dinheiro render em diversas aplicações, como o CDB – Certidão de Depósito Bancário. (Pode-se explicar, de forma leiga, que
o CDB será um investimento em que o investidor deposita uma quantia de dinheiro e esse valor é valorizado periodicamente a uma certa porcentagem e que os demais
investimentos da startup seguem uma linha de raciocínio semelhante).

Situação: Você é um consultor e amigo do criador dessa startup. Um potencial investidor anjo, ao analisar a empresa, percebeu que seria possível alcançar uma parcela de
10% de todo o dinheiro investido no mercado financeiro do país e que a receita anual da startup seria de 1% do montante total.

Problema: Seu amigo agora te pergunta qual seria a potencial receita da Easy Money, caso as informações do investidor anjo sejam verdadeiras?

Clarify (Informações adicionais, caso solicitadas)


 Deixar claro para o candidato que é necessário encontrar o dinheiro total em aplicações financeiras no Brasil. Ex: se uma pessoa colocar 1000 reais em um banco e esse
valor render 10 reais por mês, O dinheiro aplicado será equivalente aos 1000 reais. A receita da Easy Money será 1% de todo valor aplicado nela.
 O tipo de aplicação, regulamentação e taxas devem ser desconsiderados.

Guia para o Entrevistador


 Esse é um case fora da caixa. A maioria dos candidatos terão dificuldades de iniciá-lo de forma correta. Sendo assim, é necessário guiar o candidato para o caminho
correto.
*Case candidate-led: a sequência de etapas para a resolução deste case é apenas uma sugestão, visto que o candidato deve guiar a resolução.

Mandou bem! Pisou na bola...

 Estimar uma maior porcentagem economizada para as classes com maior  Iniciar o estimation sem entender que será necessário encontrar a quantidade de
poder aquisitivo (ricos economizam mais). dinheiro investido no mercado financeiro no Brasil.
 Não criar uma estrutura quantitativa para encontrar o valor desejado.

Clube de Consultoria UFC 68


Feedback aqui!
Case 2: Easy Money
1. Estimation: Qual a potencial receita da Easy Money?

Sugestão de Abordagem (1): Guia para o Entrevistador

População 200M  A sugestão de resolução apresenta apenas a


70M estrutura necessária para encontrar o dinheiro
investido em aplicações financeiras, visto que o
# de pessoas que Renda (%) 70% restante dos cálculos parar encontrar a receita da
investem Easy Money são de baixa complexidade.
 O candidato deve realizar todas as assumptions
necessárias para resolver o caso. Mais importante
Penetração (%) Idade (%) 70% que o número sugerido, são as ideias e o
7,875 Trilhões raciocínio lógico para argumentar do motivo dos
(22+
anos) números assumidos fazerem sentido.
Dinheiro investido 0,7 x 0,7 = 0,5  x 0,7 = 0,35
(R$)  Conhecimentos sobre indicadores
Realizam invest. socioeconômicos são cruciais para a resolução do
Financeiros (%) 70% case (isso pode ser estudado por este casebook
25 anos
na seção de conhecimentos relevantes)
~R$ 8 trilhões # de anos médio de # de anos trabalhados na vida / 2  Na próxima página, é apresentado uma forma
112,5k economia (metade da vida de trabalho) alternativa de realização dos cálculos, sendo
ambas aceitáveis.
Média (R$) investido 2,5K*12
por pessoa Salário médio (R$)
4500 = 30K
Economia média /
ano
% economizado 15%

Clube de Consultoria UFC 69


Feedback aqui!
Case 2: Easy Money
1. Estimation: Qual a potencial receita da Easy Money?

Sugestão de Abordagem (2): Guia para o Entrevistador

 A tabela abaixo é a base para realizar os cálculos


População 200M  100M (50%) possui idade e decide realizar investimentos financeiros
(o candidato deve ter uma noção básica para
Pessoas mais estimar esses valores). Pode-se calcular por meio
ricas poupam mais da renda familiar ou individual.
Renda Renda Anos de Total no M. Renda % da Renda
Classes % Economia X(%Pop) Classe
mensal anual economia Financeiro Familiar Pop. Ind.

25 x 36K = 5%*100M = R$ R$
A 10.000,00 120.000 30%  36K 4,5T A
16.000,00
5%
10.000,00
900K 5M
R$ R$
25 x 12K = 10%*100M = B 10%
B 5.000,00 60.000 20%  12K 3,0T 8.000,00 5.000,00
300K 10M
R$ R$
C 45%
25 x 2,4K = 50%*100M = 6.000,00 2.000,00
C 2.000,00 24.000 10%  2,4K 3,0T
60K 50M
R$ R$
D 25%
3.000,00 1.000,00
~R$ 10,5 trilhões
R$
E 15% R$ 300,00
1.000,00

Média = Média =
R$ 5.200,00 R$ 2.200,00

Clube de Consultoria UFC 70


Feedback aqui!
Case 2: Easy Money
1. Estimation: Qual a potencial receita da Easy Money?

Sugestão de Abordagem (continuação da resolução 1): Guia para o Entrevistador

 Caso o entrevistado termine o estimation de forma


 Montante aplicado: 7,875T = ~8T rápida (menos de 30 minutos), solicite um
framework a partir da seguinte indagação “Quais
 Montante endereçado na Easy Money: 10% de 8T = 800B fatores você consideraria analisar para garantir
 Receita da Easy Money: 1% de 800B = 8B que a startup Easy Money consiga alcançar essa
Easy Money tem o potencial de gerar 8 bilhões de reais, em receita, anualmente. receita potencial”. A estrutura deve conter, por
exemplo:
1. Análise da atratividade e competitividade do
Reality Check 1  PIB = 7,3BT de dinheiro aplicado. Esse não é um valor de mercado.
rendimento anual, assim, faz sentido o montante em aplicações apresentar uma ordem 2. Formas de atrair e reter clientes.
3. Formas operar e implementar.
de grandeza semelhante ao PIB. 4. Riscos envolvidos.

Reality Check 2  O Itaú, maior banco do Brasil, possui uma receita próxima de 120B
com todas suas formas de monetização. Assim, a receita da Easy Money, mesmo
sendo bem sucedida, deve ser algo, consideravelmente, menor que esse valor. Nesse
caso, o valor encontrado faz sentido ser próximo de 6% do faturamento do Itaú.

Clube de Consultoria UFC 71


Feedback aqui!
1. Mini-Case

Market Sizing
Indústria: Varejo

Led By: Interviewer

Quantitativo Qualitativo

Case 3
La Gatta Gelato
Clube de Consultoria UFC 72
Case 3: La Gatta Gelato
Introdução
Contexto: Seu cliente é uma empresa italiana que produz sovertes com sabor de fruta e formato de gatinho.

Situação: La Gatta Gelato está considerando a entrada no mercado brasileiro.

Problema: La Gatta Gelato gostaria da sua ajuda para estimar quanto de receita ele podem ter nos primeiros dois anos nesse mercado.

1. Framework: Qual será a receita da La Gatta Gelato no mercado brasileiro nos próximos dois anos?
Clarify (Informações adicionais, caso solicitadas)
 Motivação: o mercado brasileiro parece uma opção interessante de expansão para a La Gatta Gelato. Produto: o principal produto do La Gatta Gelato são picolés de
sabores variados em formato de gatinho.
 La Gatta Gelata está buscando aumentar a receita.
 La Gatta Gelato presume conseguir capturar 10% do mercado no primeiro ano e 20% do mercado no segundo.
 Pode-se assumir que o mercado não está crescendo.
 Pode-se assumir a população do Brasil igual a 200M de habitantes.
 Procura-se a receita acumulado nos dois anos.
Guia para o Entrevistador
 O candidato deve reconhecer que esse é um case de estimação, logo é recomendado que ele inicie diretamente o case com uma estrutura que o permita estimar a receita
do cliente. Um framework aqui seria desnecessário e mostraria falta de foco no que foi pedido para ser feito.
 Peça para o candidato primeiro mostrar como pretende determinar a receita (sua estrutura ou plano de cálculo) antes de começar a fazer suposições e cálculo. Chame
atenção caso alguma parte da estrutura não faça sentido.
Mandou bem! Pisou na bola...

 Validou a estrutura antes de começar a calcular  Calculou sem antes criar uma estrutura
 Fez boas suposições  Não justificou bem suas suposições
 Focou em responder o que foi solicitado

Clube de Consultoria UFC 73


Feedback aqui!
Case 3: La Gatta Gelato
1. Estimation: Qual será a receita da La Gatta Gelato no mercado brasileiro nos próximos dois anos?

Sugestão de Abordagem:

200M
Mercado
Receita Renda 80%
População
% Market share

1,04B 32,4% 90%


Condições físicas (saúde)

# Picolés vendidos por ano % compra picolé


5,2B
Tomam sorvete 90%
16 por ano
Mercado
# picolés/pessoa por ano

R$ 5,00 Tomam picolé 50%


Tabela 1
Preço/picolé Pouco  0,35 por mês
Médio  0,5 por mês
Muito  0,5 por mês
Informações Adicionais:
 Dados na página posterior.
 Criar assumptions para os dados que não
foram disponibilizados.
Clube de Consultoria UFC 74
Feedback aqui!
Case 3: La Gatta Gelato
1. Estimation: Qual será a receita da La Gatta Gelato no mercado brasileiro nos próximos dois anos?

Sugestão de Abordagem: Informações Adicionais (caso solicitadas):


 Caso solicitado, deixar o candidato arredondar valores
Tabela 1 - Sugestão de ponderação da quantidade de picolés por pessoa. para facilitar cálculos

% Guia para o Entrevistador:


Consumo # picolés/mês Representação Média
População  Os valores apresentados são apenas sugestões. O
candidato deve fazer suas próprias suposições e explicar
Pouco 70% 0,5 0,5*0,7 = 0,35 a lógica por trás delas.
 O candidato deve ter cuidado ao aplicar o Market Share.
≈ 1,35 picolés por mês
Médio 25% 2 2*0,25 = 0,5 Há varias maneiras de aplicar esse dado, logo suas
ou 16 picolés por ano
suposições devem ser explicadas.
Muito 5% 10 10*0,05 = 0,5  Se o candidato tentar aplicar os valores de Market Share
(10 e 20%) diretamente, instigue-o a pensar em outras
maneiras de aplicar essas informações.
Cálculo final: R$ 260K  Nesta sugestão de abordagem, no ano 1 está sendo
Mercado R$ 5,2B
considerado que o cliente não irá conseguir capturar 10%
Receita ano 1 de Market Share imediatamente. Ele fará isso ao longo do
ano, passando de 0 a 10% no final do ano. Logo,
R$ 1,04B % Market Share 5%
considerou-se um ponto médio de 5% para os cálculos. O
Receita mesmo raciocínio foi aplicado para o ano 2. O candidato
pode fazer suposições diferentes em relação a esses
R$ 780K Mercado R$ 5,2B valores, mas deve explicar a lógica por trás.
Receita ano 2
% Market Share 15%

Clube de Consultoria UFC 75


Feedback aqui!
Case 3: La Gatta Gelato
2. Brainstorming: Quais estratégias de entrada a La Gatta 3. Recommendation: O que podemos recomendar à La Gatta
Gelato pode tomar para alcançar esse Market Share? Gelato em relação a sua entrada no mercado brasileiro?

Sugestão de Abordagem:

Recomendação
 Crescer do zero no mercado
1. Crescimento La Gatta Gelato poderá alcançar uma receita acumulada nos dois primeiros anos
 Aumentar a força da marca no mercado
de R$ 1,04B.
orgânico  Conseguir vantagens competitivas (sabor, preço,
exclusividade etc.)

Considerações
1. Alcançará R$ 260K no primeiro ano e R$ 780K no segundo ano.
 Fazer parcerias com empresas de sorvete presentes 2. Caso consiga manter o Market Share de 20% nos anos seguintes,
no mercado brasileiro conseguirá alcançar resultados ainda maiores
2. Parcerias  Fazer parcerias com empresas que focam em
produtos simulares/complementares (brownies,
restaurantes etc.)

Riscos
1. Diminuição do tamanho do mercado
2. Dificuldade para manter o market share
3. Aquisição  Adquirir empresas de sorvete que já atuam no
mercado brasileiro Próximos Passos
1. Analisar mais a fundo as tendências do mercado
2. Definir estratégia de entrada, estudando a possibilidade de entrada através
de parcerias ou aquisição.

Clube de Consultoria UFC 76


Feedback aqui!
2. Decisão Analítica

Financial Analysis / Investment


Indústria: Mineração

Led By: Candidate

Quantitativo Qualitativo

Case 4
Vale do Rio Doce
Clube de Consultoria UFC 77
Case 4: Vale do Rio Doce
Introdução
Contexto: Nosso cliente é a Vale do Rio Doce, uma mineradora multinacional brasileira e uma das maiores operadoras de logística do país. A Vale S.A. está entre as maiores
empresas de mineração do mundo, gerando receita, principalmente, por meio da aquisição de terras para exploração de minérios.

Situação: A empresa adquiriu um terreno no Chile para a exploração de minério X (o tipo de minério é irrelevante para o caso). Todavia, após realizar a compra, foi verificado
que o solo da região, por ser extremamente duro, precisaria de uma broca especial. Assim, alguns engenheiros da Vale criaram um protótipo de uma broca mais resistente que
poderia ser utilizada no terreno, com o intuito de viabilizar a exploração.

Problema: O CEO da Vale ficou preocupado com a descoberta em relação à dureza do solo e gostaria da entender:
1. A exploração ainda é vantajosa?
2. A empresa deve fabricar a broca especial?

Clarify (Informações adicionais, caso solicitadas)


 Existe a possibilidade de terceirizar a produção da broca para uma empresa especializada.
 Lucro é o principal driver de sucesso para tomada de decisão.
 O nome do case é apenas uma homenagem. Conhecimentos específicos sobre a empresa Vale S.A. não serão relevantes para o caso.

Guia para o Entrevistador


 Este case irá avaliar a capacidade do candidato de resolver problemas de decisão analítica sem deixar de acrescentar uma visão qualitativa.
*Case candidate-led: a sequência de etapas para a resolução deste case é apenas uma sugestão, visto que o candidato deve guiar a resolução.

Mandou bem! Pisou na bola...

 Mencionar possíveis limitações de exploração, como capacidade de extração  Não priorizar uma análise financeira da exploração do terreno e da possibilidade de
e distribuição do minério. fabricar ou terceirizar a broca especial.
 Considerar tempo de exploração da mina até seu esgotamento

Clube de Consultoria UFC 78


Feedback aqui!
Case 4: Vale do Rio Doce
1. Framework: Como você analisaria a viabilidade de exploração do terreno? Devemos fabricar a nova broca?

Sugestão de Abordagem: Informações Adicionais (Caso solicitadas):


 Não considerar investimentos iniciais
 Demanda (descartando análises de ROI e payback, por
 Capacidade de exploração exemplo).
Receita  Preço do minério X
1. Viabilidade  Tempo de exploração
Guia para o Entrevistador:
Financeira  Investimento: tecnologia etc
Custos  Custo/investimento de fabricar ou terceirizar broca  O candidato deve priorizar a análise financeira.
 Outros custos: salários, logística etc Caso necessário, o entrevistador deve direcionar a
resolução para esse caminho.
 Iniciar a análise pelo lado da receita ou dos custos
 Limitações logísticas será indiferente.
2. Operação  Quantidade e qualidade da mão de obra local (Chile)
 Tecnologia para produção da broca em escala

 Vender a broca para outras mineradoras (ou a patente)


3. Estratégia  Risco de diminuição da demanda do minério X no futuro
 Oportunidades de investimento serem viabilizados com a nova broca

Clube de Consultoria UFC 79


Feedback aqui!
Case 4: Vale do Rio Doce
2. Chart/Analyses: Quais as conclusões você pode ter com esse gráfico/tabela?

Sugestão de Abordagem: 75% Informações Adicionais (Caso solicitadas):


 Desconsiderar exploração após os 20 anos.
% total (todos os 5*5%  Preço do Minério X: $375,00 por tonelada.
anos) +  Custo total (não incluindo as brocas): 1 milhão de
60K 5*10% dólares por ano.
# ton. (período com * # de anos  Desconsiderar dinheiro no tempo.
menos de 12T) vezes % de
cada ano Guia para o Entrevistador:
 É esperado que o candidato interprete os dados
180k do chart 1 e guie o case para a solução do
Demanda anual 80k problema do cliente, sugerindo encontrar a receita
# de toneladas potencial da mina. Caso isso não aconteça, guia-
lo para esse caminho.
 O candidato deve perceber que as porcentagens
67,5M estão relacionadas com a demanda anual, a qual
# de anos 10 está descrita abaixo do chart 1. Além disso, deve-
Receita ($) 120K se perceber que o último ano possuirá limitações
# ton. (período com logísticas.
mais de 12T)  Existem várias formas de realizar o cálculo. Um
375,00 bom candidato irá estruturar uma forma eficiente e
Lucro parcial *Período que atinge de fácil entendimento.
= 67,5M – 20*1M
$ / tonelada as limitações Máximo de toneladas 12k  Antes de informar o custo anual, pode-se sugerir
logísticas. (logística) um brainstorming dos principais custos na
Lucro parcial = 47,5M
operação. Deve conter na resposta: funcionários,
energia, equipamentos, máquinas e a broca.

Clube de Consultoria UFC 80


Feedback aqui!
Case 4: Vale do Rio Doce
3. Chart/Analyses: É melhor fabricar ou terceirizar a broca especial? (Financeiramente)

Sugestão de Abordagem: Guia para o Entrevistador:


 Apresentar chart 2 e aguardar abordagem do
candidato.
$ 200k  Os valores da parte de cima da estrutura são
relacionados à fabricação da broca e os valores
Custo por broca da parte de baixo à terceirização.
 O candidato deve interpretar que ambas formas
serão lucrativas, o que viabiliza a exploração da
9M $ 250k 180K mina. Todavia, fabricar será mais vantajoso.
47,5M – 36M = 11,5M  Analisar a lucratividade será um ponto positivo
Custo variável (por Demanda total
broca) (toneladas) extra para comparar os investimentos:

36M 45M 45 180K Lucratividade (fabricar) = 11,5/67,5 = ~17%


Lucratividade (terceirizar) = 2,5/67,5 = ~4%
Custo Total # de brocas
 Após a interpretação dos dados encontrados,
guiar o candidato para uma análise de riscos e,
45M 180 5*800T = 4000T
em seguida, para o fechamento do case.
Capacidade de 1
27M broca (em 5 anos)
47,5M – 45M = 2,5M
Custo fixo 5*200T = 1000T

0,00

Clube de Consultoria UFC 81


Feedback aqui!
Case 4: Vale do Rio Doce
4. Brainstorming: Quais os potenciais riscos e desvantagens 5. Recommendation: Vamos ter uma reunião com o CEO da
de fabricar a broca especial Vale S.A. dentro de 1 hora. É possível montar uma
recomendação com o que analisamos?

 Encontrar fornecedores ideais (confiabilidade etc)


 Menor poder de barganha devido às compras em Recomendação
Fornecimento pequena escala A exploração da mina é viável e a Vale deve fabricar a própria broca especial.
 Mão de obra especializada

 Mudança de foco do core business


Produção/  Dificuldade de aumentar a produção do protótipo
criação  Erros de produção Motivos
 Tempo relevante para produzir as primeiras brocas 1. Custo de fabricar a broca especial é 20% menor que o custo de terceirizar
(45M*0,8= 36M).
 Encaixe da broca nos equipamentos necessitar de 2. O lucro bruto será de 11,5M, sendo a lucratividade do investimento próxima
Utilização alterações de 20% (~17%) considerando os rendimentos dos próximos 20 anos.
 Quebra da broca (sem garantia)
 Sustentabilidade (produzir pode ser pior que a
terceirização)

Guia para o Entrevistador


 Após o brainstorming, deve-se guiar o candidato para a recomendação. Os Riscos
ricos citados devem ser semelhantes aos riscos priorizados na etapa de 1. Dificuldades de criação e produção própria
braisntorming. 2. Diminuição da demanda do minério X nos próximos anos.
Próximos Passos
1. Analisar oportunidade de vender a patente da nova broca.
2. Verificar viabilidade de otimização logística a partir do ano 15.

Clube de Consultoria UFC 82


Feedback aqui!
Case 4: Vale do Rio Doce
Chart 1: Demanda/produção do minério X

Período (em anos) após início da Produção da Vale S.A. na mina do Chile
exploração (volume-share do minério X)

1-5 5%

6-10 10%

11-15 15%

16-20 20%

*Mercado disponível do minério X: 80.000 toneladas (constante nos próximos 20 anos)


*Capacidade máxima da mineradora: 12.000 toneladas/ano (limitações logísticas)

Clube de Consultoria UFC 83


Feedback aqui!
Case 4: Vale do Rio Doce
Chart 2: Custos da broca especial

Fabricar Terceirizar

Custo Fixo Total R$ 27.000.000,00 Custo Fixo Total R$ 0,00

Custo por broca R$ 200.000,00 Custo por broca R$ 250.000,00

Capacidade anual 800T Capacidade anual 200T

Vida útil 5 anos Vida útil 5 anos

Clube de Consultoria UFC 84


Feedback aqui!
2. Decisão Analítica

Product Launch / Investment


Indústria: Serviços de Saúde

Led By: Candidate

Quantitativo Qualitativo

Case 5
Enfermeiro Anjo
Clube de Consultoria UFC
Fonte: Casebook Kellogg 2012 – Health Coachs
85
Case 5: Enfermeiro Anjo
Introdução (prompt)
Contexto: Nosso cliente é uma das maiores provedoras de seguro saúde do Brasil (fornece planos de saúde, como Sul América, Unimed etc).

Situação: Ele está explorando a lançar um programa chamado “enfermeiro anjo”, com o intuito de melhor atender a atual base de clientes de 5 milhões de membros. A ideia é
contratar e treinar um time de enfermeiros especializados em uma determinada doença, como asma, diabetes e doenças do coração. Cada enfermeiro irá gerenciar um
determinado portfólio de clientes com o intuito de reduzir os gastos de saúde com a utilização do plano (exemplos de ações: lembrar medicações, sugerir dietas e compartilhar
conselhos e boas práticas de saúde). Estudos mostraram que, a partir de um contato mensal, os custos com os pacientes podem ser reduzidos em, aproximadamente, 5%.

Problema: O CEO da empresa está com dúvidas se lançar esse novo serviço será algo vantajoso e contratou você para ajudar nessa decisão. Nosso cliente deve lançar o
programa Enfermeiro Anjo? Em caso afirmativo, com o que ele deve se preocupar?

Clarify (Informações adicionais, caso solicitadas)


 Não considerar período de pandemia.
 Os contatos com o clientes serão realizados por meio de telefonemas.
 Além de melhor atender seus clientes, o CEO da empresa tem o objetivo de fazer este plano ser lucrativo.
 O programa não irá solicitar gastos extras dos atuais clientes e, para este caso, não irá atrair novos clientes.
 Modelo de negócio de seguros: clientes pagam uma taxa mensal pelo plano e a seguradora terá como custo principal o sinistro (utilização do plano. Ex: consulta médica).
 Nosso cliente será o pioneiro na criação desse programa.
Guia para o Entrevistador
 Um bom candidato irá priorizar uma análise financeira, com o intuito de verificar se o programa será lucrativo para a empresa. Guiar o candidato, caso ele não consiga
direcionar o case para este caminho.
*Case candidate-led: a sequência de etapas para a resolução deste case é apenas uma sugestão, visto que o candidato deve guiar a resolução.

Mandou bem! Pisou na bola...

 Considerar uma possível visão negativa de médicos e hospitais em relação à  Não priorizar uma análise financeira com foco na redução de custos
confiabilidade do programa.  Não adicionar tópicos qualitativos, no framework, relacionados à dificuldade de
 Adicionar segmentação de clientes no impacto da redução de custos. implementar o programa.

Clube de Consultoria UFC


Fonte: Casebook Kellogg 2012 – Health Coachs
86
Feedback aqui!
Case 5: Enfermeiro Anjo
1. Framework: Nosso cliente deve lançar o programa Enfermeiro Anjo? Em caso afirmativo, com o que ele deve se preocupar?

Sugestão de Abordagem: Guia para o Entrevistador

 Um bom framework deverá proporcionar uma


 Segmento de clientes (buscar segmentos com maior análise quantitativa (financeira) e qualitativa da
Redução de custos potencial de redução de custos) viabilidade do programa.
 % de clientes que irão adotar o programa  Desconsiderar quaisquer ganhos de receita.
1. Financeiro  Desconsiderar investimentos iniciais.
 Investimento e infraestrutura  Dependendo das perguntas realizadas na etapa
Custos adicionais  Capacidade do enfermeiro (# de clientes no portfólio) de clarify, o framework pode conter pontos extras,
 Custo por enfermeiro: salário e equipamentos como um possível aumento de receita. Tal ponto
não deverá ser considerado como algo negativo,
caso o candidato informe que o principal ganho
 Dificuldade de contratar e treinar enfermeiros será a redução de custos (informado no prompt).
 Tecnologia: gerenciar dados de todos os membros do programa  Um bom candidato buscará verificar a viabilidade
2. Operação  Forma de divulgação do programa para a base de clientes do Enfermeiro Anjo por meio de uma análise de
 Parcerias com médicos (ajudar na promoção do programa) lucro incremental, considerando apenas a
diferença entre custos reduzidos e custos
adicionados pelo novo programa. Buscar calcular
todo o custo da empresa antes e depois do
programa não será um caminho eficiente.
 Reação de concorrentes  Repassar as informações da etapa de clarify
3. Riscos  Reação dos hospitais (irão ter menor receita com o novo programa) durante o case, ao serem solicitadas pelo
 Leis e regulamentações candidato.

Clube de Consultoria UFC


Fonte: Casebook Kellogg 2012 – Health Coachs
87
Feedback aqui!
Case 5: Enfermeiro Anjo
2. Chart 1: Quais os principais insight deste gráfico? 3. Chart 2: O que pode ser feito com essas informações?

Formas de segmentação de clientes: Insights do gráfico:


 Doença  Segmento de clientes diabéticos 65+ são os mais custosos (foco principal).
 Tipo de plano  Diabéticos também são os mais custosos e representativos em todos os tipos
 Custo para a empresa (sinistro) de plano de acordo com o chart 1.
 Idade
 Renda Próximos passos:
 Região geográfica  Verificar se os custos, de implementação do programa, seriam os mesmos para
cada segmento (capacidade dos enfermeiros por doença.
Insights/hipóteses do gráfico:  Calcular a viabilidade do programa em algum segmento.
 Segmento Plano Grupo possui a maioria dos membros saudáveis, logo deverá
ser o menos vantajoso (redução de 5% em cima dos gastos). Informações Adicionais (Caso solicitadas):
 Segmento 65+ possui os membros mais doentes (apenas 15% saudável).  Custo do enfermeiro(a): R$ 60 mil por ano.
 Hipótese: segmento 65+ poderá ser o mais vantajoso, pois é o que apresenta  Outros custos (equipamentos etc): 20% do custo por enfermeiro(a).
maiores gastos (idosos utilizam mais o plano do que jovens).  Outros custos/investimentos devem ser desconsiderados.
 1 enfermeiros consegue ligar para 8 clientes por dia.
Informações Adicionais (Caso solicitadas):
 Considerar 25 dias úteis no mês.
 Plano Grupo: plano coorporativo ou plano família.
Guia para o Entrevistador Guia para o Entrevistador
 Caso o candidato priorize outro caminho do case, direcione-o para uma análise  Mesmo que seja solicitado pelo candidato, evitar mostrar o chart 2 antes do 1.
dos segmentos de clientes.  Antes de apresentar o chart 2, perguntar “qual grupo de clientes focar?”, caso o
 Pode-se sugerir a análise perguntando “como você segmentaria os candidato não tenha apresentado hipóteses em busca do segmento de cliente
pacientes deste programa” e, em seguida, apresentar o chart 1. mais vantajoso.
 O candidato deve perceber que o segmento mais vantajoso será o mais custoso  A parte (4x) da tabela significa que os diabéticos custam 4 vezes mais.
para a empresa, visto que a redução é de 5% em relação ao valor bruto. Assim,  Após o candidato sugerir um possível próximos passos para dar continuidade
hipóteses devem ser formadas em busca do segmento com maior custo. ao case, iniciar etapa 4 de análise.

Clube de Consultoria UFC


Fonte: Casebook Kellogg 2012 – Health Coachs
88
Feedback aqui!
Case 5: Enfermeiro Anjo
4. Analyses: O CEO da empresa de seguro saúde gostaria de saber a viabilidade financeira Enfermeiro Anjo. Qual o lucro anual
(R$/ano) do programa, caso ele seja implementado apenas no segmento mais lucrativo?
Sugestão de Abordagem: Guia para o Entrevistador
(300*4)*12  O cálculo pode ser estruturado de várias formas.
Custos do cliente/ ano
720 = 14.400 Todavia, não estruturar o cálculo deverá ser
considerado um ponto negativo.
Custo reduzido/  Outra forma de estruturação do cálculo: número
cliente
288M de enfermeiros e lucro por enfermeiro(a).
 O candidato deverá utilizar os chats e as
% de redução 5% informações repassadas durante o case para
Redução de custos
realizar todos os cálculos necessários.
Gráfico:  Candidatos podem ter problemas na definição da
# de clientes* 5M*0,2*0,4 capacidade dos enfermeiros, caso não
144M = 400K considerem que o contato com os clientes deve
ser mensal (conforme prompt).
Lucro (R$/ano)  Bons candidatos irão fazer o “so what?” do
2000 # de clientes* 400K número final como sendo algo muito vantajoso,
pois não precisa de investimentos iniciais e a
# de enfermeiros redução de custos líquida é igual ao aumento de
custos necessários.
144M
Capacidade de 1 25*8
enfermeiro (portfólio) = 200
Aumento de custos

60K + 60K*0,2
R$/ enfermeiro(a)
= 72K

Clube de Consultoria UFC


Fonte: Casebook Kellogg 2012 – Health Coachs
89
Feedback aqui!
Case 5: Enfermeiro Anjo
4. Recomendation: Vamos ter uma reunião com o CEO da empresa daqui a 1 hora, você poderia montar uma recomendação de
acordo com o que analisamos sobre o programa?
Sugestão de Abordagem: 5. Pergunta extra (se o tempo permitir): Existem
650K diabéticos do Plano Grupo deixados "sem
cobertura". Existe uma forma de rentabilizar o
Recomendação segmento?
O cliente deve lançar o programa Enfermeiro Anjo com foco, inicialmente, no segmento de 65+ diabéticos. As ideias incluem:
 Programa de mais eficiente (por exemplo,
treinamento no trabalho, contato bimestral,
correspondência automatizada por aplicativos etc).
Motivos  Buscar fontes de receita adicionais (por exemplo,
1. É o segmento que seria possível obter a maior redução de custos. os empregadores podem estar dispostos a pagar
2. Redução de custos de 144 milhões de reais por ano sem a necessidade de investimentos (custos upfront). uma taxa, apoio governamental).
3. Após de um projeto piloto desse programa, será possível validar as assumptions de redução de custos por  Como 5% é a economia média, o programa pode
cliente (5%), capacidade dos enfermeiros (portfólio de clientes) etc. ter como alvo os membros que responderão com
economia bem acima de 5% (segmentação extra).

Riscos Próximos Passos


1. Reação dos hospitais e médicos 1. Mitigar riscos por meio de parcerias e melhor
(menosprezar o programa). estruturação do plano.
2. Não adoção 100% ao plano (cliente não 2. Analisar possibilidade de expansão do
atender o telefone ou não querer seguir as programa para outros segmentos.
recomendações, por exemplo). 3. Analisar possibilidade de aumento de receita,
por meio da atração e retenção de clientes.
4. Buscar contratar e treinar os enfermeiros.
Clube de Consultoria UFC
Fonte: Casebook Kellogg 2012 – Health Coachs
90
Feedback aqui!
Case 5: Enfermeiro Anjo
Chart 1: Receita e base de clientes anual por plano

Individual Grupo 65+


100% 6%
19% 14% 3%
90% 6%
2% 2%
80% 3% 7% 15%
5% 5%
70%
% of members

10% 5% Outros
60% 8% 20% 10% Alzheimer
50% Pós-câncer
40% 30% Colesterol Alto
40%
30% Asma
50% Hipertensão
20%
25% Diabetes
10% 15% Saudável
0%
0% 10% 20% 30% 40% 50% 60% 70% 80% 90% 100%
Segment (%)

Clube de Consultoria UFC


Fonte: Casebook Kellogg 2012 – Health Coachs
91
Feedback aqui!
Case 5: Enfermeiro Anjo
Chart 1: Média de custo dos membros dos segmentos

Segmento Média de custos PMPM* Média Diabéticos

Plano Individual R$ 150,00

Plano Grupo R$ 100,00 4x

Plano 65+ R$ 300,00

*Por Membro Por Mês

Clube de Consultoria UFC


Fonte: Casebook Kellogg 2012 – Health Coachs
92
Feedback aqui!
3. Diagnóstico

Profitability
Indústria: Telecomunicações

Led By: Candidate

Quantitativo Qualitativo

Case 6
Tele Fortal
Clube de Consultoria UFC 93
Case 6: Tele Fortal
Introdução
Contexto: Estamos no final de 2015, no Brasil. A Tele Fortal é uma concessionária de telefonia móvel com escritório sediado em Fortaleza, Ceará. A empresa possui um
modelo de negócio por assinatura, em que seus clientes pagam um valor fixo mensal, de acordo com o plano escolhido, e têm direito de realizar ligações e utilizar internet de
acordo com o limite de cada plano. Caso ultrapasse o valor estabelecido, o cliente paga valores extras na cobrança final.

Situação: A empresa sempre buscou atender um público premium, o qual possui renda acima da média brasileira. Porém, a cerca de 2 anos a Tele Fortal decidiu entrar em um
mercado low-end (baixo custo), oferecendo planos com pacotes de ligação e internet menores, com o intuito de crescer com o aumento do número de clientes,

Problema: Apesar de ter conseguido aumentar o número de clientes, a empresa manteve seus lucros sem grandes variações durante esse período de 2 anos. O CEO da Tele
Fortal buscou sua ajuda para entender o motivo do ocorrido e como solucionar esse problema.

Clarify (Informações adicionais, caso solicitadas)


 Simplificar: a empresa possui apenas uma fonte de receita que é a de planos pós-pago.
 O cliente não possui uma meta específica.

Guia para o Entrevistador


 O candidato deve perceber que esse é um case de diagnóstico. Antes de buscar soluções, o candidato deve encontrar a causa raiz do problema, por meio da criação de
hipósteses e da análise de dados que validem essas hipótese.
*Case candidate-led: a sequência de etapas para a resolução deste case é apenas uma sugestão, visto que o candidato deve guiar a resolução.

Mandou bem! Pisou na bola...

 Apresentar, no framework, outras possíveis causas do problema além de uma  Criar um framework não focado em diagnosticar o problema
possível canibalização (sem priorizá-las), como novos entrantes etc.  Não priorizar uma hipótese relacionada ao novo plano low-end que foi lançado

Clube de Consultoria UFC 94


Feedback aqui!
Case 6: Tele Fortal
1. Framework: Qual o motivo dos lucros não crescerem proporcionalmente com a receita? Como solucionar este problema?

Sugestão de Abordagem: Guia para o Entrevistador


 A principal hipótese para este case, devido ao que
foi apresentado no prompt, é de que o novo
 Canibalização serviço low-cost tenha canibalizado as vendas dos
 Política de descontos planos preminum, diminuindo a lucratividade (%)
Ação interna  Erros internos. Ex: atendimento ruim da empresa (lucro/receita).
para clientes mais lucrativos  Para realizar a validação da hipótese, o candidato
Margem bruta por deve solicitar dados referente à variação das
plano receitas por plano nos últimos anos.
 Concorrentes com melhores ofertas
 Outra hipótese válida é o aumento dos custos de
Mercado  Novos entrantes
marketing e vendas para divulgação do novo
 Recessão econômica
plano lançado. Todavia, deve-se guiar o candidato
para uma análise de receita, caso ele priorize
Lucro
outra opção.
 Cabeamento  A parte de custos possui apenas tópicos de alguns
Diretos (sinal  Torres e antenas custos da indústria de telecomunicações. Porém,
telefônico)  Manutenção o candidato deve abordar, por meio de hipóteses,
os motivos que possam ter aumentado tais custos,
Custos fixos de forma desproporcional com a receita, durante o
 Marketing e vendas período analisado. Assim, justificando a
 Salário e encargos diminuição da lucratividade (%).
Indiretos
 Alugueis/infraestrutura (agências, SAC)

Clube de Consultoria UFC 95


Feedback aqui!
Case 6: Tele Fortal
2. Chart: Quais os principais insights deste gráfico?

Sugestão de Abordagem: Informações Adicionais (Caso solicitadas):


 O “plano 3’ foi o plano low-cost lançado no final de
Pontos principais do gráfico: 2013.
 Custos são proporcionais a receita por cliente.
1. Aumentamos o número de clientes e a receita do plano 3;
 O mercado manteve um crescimento estável de
2. Perdemos clientes do plano 2;
5% ao ano no período analisado.
3. Plano 1 se manteve quase estável;
4. Os preços não mudaram significantemente nos últimos 2 anos;
5. O preço do plano 1 > plano 2 > plano 3. Guia para o Entrevistador
 Mostrar os gráficos que estão ao final deste case.
 Aguardar o candidato sugerir próximos passos
Conclusão/Insights: para diagnosticar o problema (candidate-led).
Apesar do número total de clientes ter aumentado, perdemos clientes de uma oferta que possui maior receita e isso
fez que nossas receitas ficassem quase estáveis. Isso pode ter afeitado nosso lucro, caso as margens forem
 Insight extra: a canibalização não
proporcionais a receita.
necessariamente é algo prejudicial para a
Hipótese: Plano 3 possui menor margem de lucro e canibalizou as vendas do plano 2 (clientes que antes eram do
empresa, pois ela pode ser fruto de uma reação
plano 2 agora são do plano 3).
competitiva do cliente a uma alteração no
mercado, por exemplo. É possível que um novo
Next-Steps: entrante tenha chegado com um serviço mais
O entrevistado pode ser proativo e sugerir a análise de custos ou entender o motivo de ter ocorrido a diminuição de barato e, caso a Tele Fortal não tivesse lançado o
clientes cadastrados no plano 2. plano low-cost, ela poderia ter tido a mesma ou
maior diminuição de lucro. Porém, os clientes
seriam agora de uma outra empresa. Dessa
forma, o prejuízo para a Tele Fortal seria bem
maior.

Clube de Consultoria UFC 96


Feedback aqui!
Case 6: Tele Fortal
3. Analyses: Qual o lucro por cliente de cada plano no ano de 4. Brainstorming/Hypothesis: Quais os principais motivos
2015? que podem ter feito nossa empresa perder clientes no plano 2
e quais dados você precisaria para validar isso?
Sugestão de Abordagem:
Sugestão de Abordagem:
Plano 1  60.500.000 / 240.000 = 250,00 x 0,3 = R$ 75,00
 Substitutos  Pesquisa / Análise de novos
Plano 2  43.100.000 / 238.000 = 181,00 x 0,3 = R$ 54,00 substitutos
Mercado
diminuiu  Recessão  pandemia / econômia
Plano 3  64.600.000 / 660.000 = 98,00 x 0,3 = R$ 29,00  Mudança de preferências do cliente 
Pesquisa de tendências
Conclusão:
Estamos focando na venda de um plano que gera para a empresa um lucro por  Canibalização  Dados
cliente equivalente a quase metade do plano 2 (seguir com a hipótese de de cadastro do cliente
# de clientes
canibalização).  Qualidade/oferta ruim 
Motivo da saída dos
Interno clientes
Informações Adicionais (Caso solicitadas):  Atendimento  Problemas
 Os custos totais de cada plano podem ser representados com 70% da receita. registrados no SAC
Perdemos
 Ao analisar o sistema de gestão da empresa (ERP) foi verificado que os CPF’s Market-
da maioria dos clientes que estão no plano 3 são os mesmos que estavam Share
 Concorrentes ou novos
cadastrados no plano 2 no ano de 2013. A diminuição de clientes do plano 2 foi entrantes: oferta melhor
devido à canibalização. para a faixa de preço 
Externo
Guia para o Entrevistador Comparar oferta da Tele
 Aproximar valores para facilitar os cálculos. Fortal com os concorrentes

Clube de Consultoria UFC 97


Feedback aqui!
Case 6: Tele Fortal
5. Recomendação: Vamos ter uma reunião com o CEO da Tele Fortal em 1 hora, o que você apresentaria para ele?

Sugestão de Abordagem: Informações Adicionais (Caso solicitadas):


 Nenhum concorrente afetou drasticamente o
negócio nos últimos 2 anos.
Recomendação/Conclusão
A diminuição de lucro da Tele Fortal, mesmo com o aumento da base de clientes, ocorreu devido à
canibalização do plano 2 após o lançamento do plano 3. Isso foi confirmado, pois muitos dos CPF’s que estão
cadastrados no plano 3 eram do plano 2 em anos anteriores. Guia para o Entrevistador
 Antes de preparar a recomendação, converse com
o candidato para saber o que poderíamos fazer
diante dessa situação e peça a ele que defenda
Ações sua recomendação de acordo com seu ponto de
Algumas ações que podem ser feitas para aumentar nossos lucros são: vista.
1. Aumentar o preço do plano 3
2. Desligar plano 3
3. Diminuir preços dos planos 1 e 2, buscando migrar clientes do plano 3
4. Intensificar marketing e benefícios dos planos 1 e 2

Riscos Próximos Passos


1. Reação dos clientes 1. Analisar as ofertas do concorrentes atuais e
2. Reação dos concorrentes comparar com a nossa
2. Fazer um estudo de sensibilidade de preço
dos clientes

Clube de Consultoria UFC 98


Feedback aqui!
Case 6: Tele Fortal
Chart 1: Receita e base de clientes anual por plano

Receita Anual (em R$ MM) Numero de Clientes (em MM)


200 1200
180 1
160 1000
32,6 10
140 64,6 800 660
270
120
123,5
100 79,5 600
680
80 43,1
400 430
60 238
40 200
50,8 55,6 60,5
20 200 220 240
0 0
2013 2014 2015 2013 2014 2015
Plano 1 Plano 2 Plano 3 Plano 1 Plano 2 Plano 3

Clube de Consultoria UFC 99


Feedback aqui!
3. Diagnóstico

Operations / Profitability
Indústria: Marketing Digital / Serviços Financeiros

Led By: Candidate

Quantitativo Qualitativo

Case 7
Cards for Everyone
Clube de Consultoria UFC 100
Case 7: Cards for everyone
Introdução
Contexto: Seu cliente é uma empresa de marketing digital chamada DigiVendas. Dentre os diversos serviços que a DigiVendas oferece, desenvolvimento e gerenciamento de
plataformas digitais é um dos mais relevantes.

Situação: Recentemente, a DigiVendas fechou uma parceria com um banco para aumentar a venda digital de cartões de crédito. Nessa parceria, a DigiVendas assumiu o risco
de operação, o que significa que sua remuneração será diretamente ligada à quantidade de cartões aprovados. O processo de aquisição de clientes ocorre da seguinte forma: o
potencial cliente entra em contato com anúncios que o guia até a plataforma na qual poderá solicitar seu cartão de crédito, ele então preenche um formulário com suas
informações. Esses dados são enviados e analisados pelo banco que irá decidir se o cartão será aprovado.

Problema: A parceria foi fechada com o objetivo de aumentar a quantidade de cartões aprovados e diminuir os custos de aquisição de clientes.

Clarify (Informações adicionais, caso solicitadas)


 A DigiVendas é remunerada por cada cartão aprovado. O montante está ligado ao custo de aquisição de clientes, de modo que, quanto maior o custo, menor o valor.
 Custo de aquisição de clientes: custos por cada cartão aprovado
 Objetivo: 300 mil cartões aprovados por ano e custo por cartão aprovado de até 200 reais.
 Os critérios de aprovação do cartão são definidos pelo banco, logo a DigiVendas não possui controle direto da etapa de análise de informações.
 A DidiVendas não atua com venda física, logo o único canal de venda a ser considerado é digital,

Guia para o Entrevistador


 O candidato deve reconhecer que esse case busca analisar e melhorar o funil de vendas de uma plataforma digital. Portanto, dentro da sua análise devem ser inclusas as
etapas necessárias para a aprovação do cartão.
*Case candidate-led: a sequência de etapas para a resolução deste case é apenas uma sugestão, visto que o candidato deve guiar a resolução.

Mandou bem! Pisou na bola...

 Considerou as etapas do funil de vendas  Não focou no processo de aprovação do cartão


 Trouxe boas ideias sobre como melhorar os custos e quantidade de cartões  Trouxe ideias genéricas para o aumento de vendas
aprovados  Não conseguiu desenvolver os cálculos
 Realizou os cálculos focando em alcançar o objetivo
Clube de Consultoria UFC 101
Feedback aqui!
Case 7: Cards for everyone
1. Framework: Quais critérios devem ser analisados para alcançar esse objetivo?

Sugestão de Abordagem: Informações Adicionais (Caso solicitadas):


Guia para o Entrevistador
 O custo médio de aquisição de clientes é de R$
 40K por dia pode ser feita de várias formas.
A estrutura
 Definição do público alvo  Não é necessário
Todavia, se preocupar
é interessante com investimentos
que o candidato considere
 Anúncios  Eficiência/taxa de sucesso além
o quedesses
pode ser custos
feito com o produto, como
1. Etapas do  Custos
 Plataforma monetizar e algumas dificuldades envolvidas para
processo  Formulário  Motivos de desistência realizar a operação.
 Critérios de aprovação  Potencial de melhoria  Caso
Guia parao o Entrevistador
candidato não tenha feito perguntas
 Se o candidato
suficientes sobre não incluir nada
o produto que envolva
na etapa inicial, éas
etapas do processo
interessante que sejamde aquisição
adicionadas(funil
node vendas),
framework.
 o guie nessa
Deixar direção.
o candidato priorizar o caminho. Caso não
2. Capacidade de  Métodos e tecnologia  Principal
priorize ashipótese:
possíveis DigiVendas
ações quepode
podem melhorar
ser a
 Recursos e capital
melhorar  Limitações da parceria/tomada de decisão
eficiência
realizadas,(conversão) das etapas
forçar o caminho do funil de
da resolução para
vendas e, com
esta parte isso, aumentar a quantidade de
do case.
cartões aprovados sem aumentar os custos
envolvidos, alcançando ambos os objetivos.
 Quando o candidato comentar sobre as etapas do
 Cancelamento de contrato processo de aquisição ou eficiência do processo,
3. Riscos  Alta concorrência apresente o Chart 1 (Fim do case).

Clube de Consultoria UFC 102


Feedback aqui!
Case 7: Cards for everyone
2. Chart: Qual a quantidade de cartões e custo unitário atual?

Sugestão de Abordagem: Informações Adicionais (Caso solicitadas):

Pontos principais do gráfico:  Os valores no funil representam potenciais


clientes. Por exemplo, os anúncios atingem um
1. A taxa de aprovação em relação aos anúncios é menos de 0,5%
público de 30K pessoas, dessas, 2700 acessam a
2. A maior perda de clientes é entre o acesso e envio do formulário. A segundo maior perda é entre o envio do
plataforma e assim por diante.
formulário e a aprovação.
 As informações no funil são por dia, logo 130
3. Cartões aprovados por ano  120 cartões x 360 dias = 43,2K ≈ 43K cartões
cartões são aprovados diariamente.
4. Custo unitário  40K / 120 ≈ R$ 333,00 por cartão
 O custo unitário é determinado em relação à
quantidade de cartões aprovados.
Conclusão/Insights:
As últimas etapas do funil, envio do formulário e aprovação, são as mais preocupantes, uma vez que apresentam a Guia para o Entrevistador
maior perda de potenciais clientes. Por outro lado, essas apresentam o maior potencial de melhoria.
 Considerando a simplicidade do cálculo, é
É necessário aumentar em mais de 5 vezes a quantidade de cartões aprovados e reduzir o custo em 133 reais por esperado que o candidato realize essa etapa
cartão. É preciso avaliar a capacidade de melhoria das taxas de sucesso de cada etapa a fim de determinar se será rapidamente.
de fato possível alcançar o objetivo do cliente.  Após essas conclusões, guie o candidato para um
brainstorming sobre como a conversão de cada
Next-Steps: etapa do funil de vendas pode ser melhorada.
O entrevistado pode ser proativo e sugerir uma análise de como melhorar as etapas do funil de vendas.

Clube de Consultoria UFC 103


Feedback aqui!
Case 7: Cards for everyone
3. Brainstorming: Como a DigiVendas pode melhorar a conversão das etapas do funil de vendas?

Sugestão de Abordagem:
Informações Adicionais (após Brainstorming):
 Será possível aumentar a taxa de conversão das
 Melhores canais etapas:
1. Anúncio   Anúncios mais atrativos  Entre Acesso a plataforma e Formulário: 20%
Plataforma  Anúncios personalizados de acordo com o perfil do cliente  Entre Formulário e Envio: 25%
 Carregamento mais rápido da plataforma  Entre Envio e Cartão Aprovado: 10%

Guia para o Entrevistador


2. Plataforma   Plataforma mais simples  Bons candidatos irão trazer ideias para cada etapa
 Botões mais visíveis e acessíveis que levem ao formulário do funil.
Formulário  As informações acima devem ser passadas após
o Brainstorming. O candidato deverá em seguida
para a análise dessas dados.
 Minimização das informações necessárias
3. Formulário   Continuar formulário de onde parou
Envio  Meios alternativos para entrar com as informações (fotos, copiar etc.)
 Clareza e simplicidade

4. Envio  Cartão  Melhor definição do público alvo de acordo com critérios de aprovação do banco
aprovado  Validação dos documentos antes do envio

Clube de Consultoria UFC 104


Feedback aqui!
Case 7: Cards for everyone
4. Analysis: Com essas melhorias, DigiVendas irá alcançar 5. Recomandation: O que pode dizer à CEO da Safe Car em
seu objetivo? relação às perguntas?

Sugestão de Abordagem:

Aumento da taxa de conversão  1,2 x 1,25 x 1,1 = 1,65 Recomendação


A DigiVendas conseguirá alcançar seu objetivo ao aumentar a conversão nas
Nova quantidade de cartões aprovados  1,65 x 120 = 198 ≈ 200 cartões últimas três etapas do funil e aumentar a base de clientes para 105K.

Cartões por ano  200 x 360 = 72K cartões

Custo unitário  40K / 200 = R$ 200,00 por cartão Motivos


1. Com o aumento da taxa de conversão será alcançado o custo unitário de R$
200,00 por cartão
Conclusão: 2. Além disso, a quantidade de aprovações passará de 43K para 72K por ano.
Com o aumento da conversão, a DigiVendas consegue alcançar o objetivo de 3. Uma base de clientes 3,5 vezes maior irá resultar em 250K cartões
custo unitário, mas não o de cartões aprovados. Contudo, supondo que o custo aprovados por ano.
unitário não aumente, é possível alcançar a meta de cartões aumentando a base
de clientes. Nesse caso a base de clientes deverá ser 3,5 (250K cartões / 72K
cartões) vezes maior que a atual, ou seja, terá que ser de 105K (30K x 3,5)
potenciais clientes.
Riscos
Guia para o Entrevistador 1. Aumento dos custos devido ao aumento da base de clientes.
 Há varias formas de fazer esse cálculo. A maneira apresentada é uma mais 2. Mercado potencial limitado
simplificada. O candidato pode procurar maneiras mais longas, como, por Próximos Passos
exemplo, aplicar o crescimento da taxa de conversão em cada etapa. 1. Analisar o efeito do aumento da base de clientes nos custos
 Um aumenta da taxa de conversão reflete diretamente o aumento da valor final, 2. Analisar o tamanho do mercado potencial
logo os aumentos podem ser aplicados diretamente na quantidade de cartões.

Clube de Consultoria UFC 105


Feedback aqui!
Case 7: Cards for everyone
Chart 1: Funil de vendas digitais de cartões de crédito

Custo médio:
R$ 40K/dia Anúncio: 30K

Acessos à plataforma: 2700

Formulário de solicitação: 2200

Envio do formulário: 700

Cartão aprovado:
Objetivo:
120
Custo Unitário: R$ 200,00
Quant. de cartões aprovados por ano: 250K

Clube de Consultoria UFC 106


Feedback aqui!
3. Diagnóstico

Profitability
Indústria: Varejo

Led By: Candidate

Quantitativo Qualitativo

Case 8
Where’s the Party
Clube de Consultoria UFC
Fonte: case remodelado de um pdf (origem desconhecida)
107
Case 8: Where’s the Party
Introdução
Contexto: Nosso cliente é a Vamos Festejar (VF), um grande fornecedor de papeis de presente, laços e outros materiais similares vendidos em lojas de varejo. Eles vendem
seus produtos para lojas de varejo que fazem com que eles cheguem ao consumidor final.

Situação: Recentemente, eles descobriram que vendem o equivalente a R$100M na Target, enquanto seu competidor teve vendas equivalentes a R$180M durante o mesmo
período.

Problema: A CEO da Vamos Festejar entrou em contato conosco para ajuda-los a descobrir o que está acontecendo e como eles podem melhorar a situação.

Clarify (Informações adicionais, caso solicitadas)


 A Vamos Festejar fabrica três tipos de produtos: papel de presente, laços e cartões de presente (pequenos cartões que normalmente vão junto a presente, buquês etc.)
 A Vamos Festejar não tem suas próprias lojas de varejo, ela distribui seus produtos apenas através de grandes varejistas como Target, Wall-Mart etc. Neste case, serão
consideradas apenas as vendas na Target.
 Há apenas dois grandes fornecedores desses produtos com market share similares no mercado: Vamos festejar e seu competidor
 Os valores de venda de R$ 100M e R$ 180M equivalem a receita da Target com esse produtos, não representa necessariamente a receita da Vamos Festejar.
 A Target é uma grande varejista, com diversas lojas espalhadas na sua área de atuação.

Guia para o Entrevistador


 Esse case contém muitas informações que devem ser passadas quando solicitadas de modo que é extremamente importante que o entrevistador entenda bem o case
antes de aplica-lo.
 Neste case, o diagnóstico representa uma parte muito importante e é por onde o candidato deve começar sua análise. Passe as informações quando solicitadas, quando o
candidato comentar ao relacionado à informação e/ou quando julgar necessário para o avanço do case.
*Case candidate-led: a sequência de etapas para a resolução deste case é apenas uma sugestão, visto que o candidato deve guiar a resolução.

Mandou bem! Pisou na bola...

 Criar um framework focado em diagnosticar o problema  Criou um framework genérico que não o ajudou no diagnóstico
 Analisar e associar os dados e informações passadas de modo a diagnosticar  Considerou cada informação e Chart isoladamente
os problemas e pensar soluções apropriadas.  Não conseguiu chegar no diagnóstico final

Clube de Consultoria UFC 108


Feedback aqui!
Case 8: Where’s the Party
1. Framework: O que deve ser analisado para encontrar a(s) causa(s) raiz(ízes) do problema?

Sugestão de Abordagem: Informações Adicionais (Caso solicitadas):


 Uma loja da Target que vende produtos da VF,
 VF e o competidor estão presentes nas não vende do competidor e vice versa.
# lojas da
Target mesmas lojas?  Chart 1: Número de lojas da VF e do competidor.
 VF é vendido em menos lojas?  Os preços dos produtos dos dois players são
# vendas iguais, sendo R$1 por item (papel de presente,
laço ou cartão).
# vendas por  As margens de ambos produtores são iguais,
Volume de loja  VF tem menos clientes que o competidor? sendo R$0,30 para laços, R$0,30 para papel de
venda presente e R$0,50 para cartões.
 Os produtos são padronizados e não variam muito
Volume por em relação ao competidor.
 Cada cliente compra mais itens do competidor?
venda
Comparação Guia para o Entrevistador
de receita  Uma boa abordagem nesse tipo de case é
começar com uma” issue tree”, iniciando por
 O preço é similar ao do competidor?
Preço perguntas “macro” e ramificando a árvore ao
 Há diferenças de preço entre os diferentes produtos?
buscar problemas mais “micro”. Como isso, o
Preço médio
framework pode ficar muito extenso, logo ele pode
Mix de  Há diferenças entre os produtos dos dois players? criar uma estrutura inicial e ramifica-la conforme
produtos  Há diferenças no mix de produtos dos dois players? vai fazendo perguntas e avançando no case.
 Com essas informações, o candidato pode
perceber que o problema não está no preço médio
e que a VF está vendendo menos itens que o
competidor, logo ele deve explorar mais o
#vendas.
Clube de Consultoria UFC 109
Feedback aqui!
Case 8: Where’s the Party
1. Framework: O que deve ser analisado para encontrar a(s) causa(s) raiz(es) do problema?

Sugestão de Abordagem: Informações Adicionais (Caso solicitadas):


 Não há diferenças entre as lojas da Target
(tamanho, proporção rural/urbano etc.)
 Os gerentes de loja da Target decidem qual será
# vendas o fornecedor da sua loja. Essa decisão é tomada
através de um contrato com 2 anos de duração
 As decisões são baseadas em relações prévias
do gerente ou em conversas com gerentes de
Menos lojas da Menos vendas por outras lojas da Target. Dentre essas conversas,
Target loja eles costumam discutir receitas e lucros.
 Os dois players têm acordos similares com as
 Como são as negociações com a  Há diferenças entre as lojas na qual a lojas da Target. Não há diferenças na eficiência
Target? VF em relação ao competidor? dos seus representantes de vendas.
 Há diferenças no lucro da Target com a  O competidor possui  Chart 2: margem da Target para os diferentes
VF e o competidor? marketing/divulgação melhor? produtos e fornecedores.
 Há diferença no relacionamento da  Os produtos do competidor possuem um  Chart 3: vendas por tipo de produtos por loja
Target com a VF e o competidor? visual (embalagem etc.) mais  As lojas da Target seguem um formato padrão,
 O competidor tem representantes de atrativo/melhor adaptado ao público? loja não há diferenças no localizados dos produtos
vendas mais eficientes?  Os produtos do competidor estão entre VF e competidor.
 A Target estaria disposta a vender VF melhores localizados ou dispostos nas  Chart 4: disposição dos produtos dos dois players
ao invés do competidor? lojas? nas prateleiras
 Chart 5: disposição detalhada de cada tipo de
produto dos players nas prateleiras.

Clube de Consultoria UFC 110


Feedback aqui!
Case 8: Where’s the Party
2. Chart 1: Em quantas lojas VF e o competidor atuam? 3. Chart 2: Qual é a margem do distribuidor nos produtos
vendidos?

Sugestão de Abordagem: Sugestão de Abordagem:

A VF vende em menos lojas que seu competidor. As margens para a Target são as mesma em relação aos dois fornecedores.

Venda média da VF por loja  100M / 25 = R$ 4M por loja Cartões de presente apresentam margens maiores, seguidos por laços e papeis de
presente apresentam a menor margem.
Venda média do competidor por loja  180M / 30 = R$ 6M por loja
Conclusão:
Conclusão: A venda de cartões de presente é mais vantajosa tanto para a Target quando para
Parte da diferença nas vendas todas dos players é devido a menor quantidade de os fornecedores. Por outro lado, a venda de laços é melhor que a venda de papeis
lojas nas quais a VF vende em relação a seu competidor. Além disso, a venda de presente para a Target, ainda que não faça diferença para os produtores.
média por loja da VF é um terço menor. Ambos os fatores resultam na diferença de Impulsionar as vendas de cartões de presentes pode ser interessante para
receita de R$ 80M. melhorar as negociações com a Target. Outra maneira de fazer isso seria
Além disso, o valor menor de venda média por loja da VF pode diminuir sua aumentar a margem da Target, ganhando uma vantagem em relação ao seu
atratividade em relação às lojas da Target, dificultando sua expansão para mais competidor. Isso, contudo, resultaria na perda de lucratividade, de modo que deve
lojas. ser analisado mais detalhadamente.

Guia para o Entrevistador Guia para o Entrevistador


 Dependendo de quando esse Chart for apresentado, ele pode guiar o candidato  Essa conclusão faz referência aos dados de margem dos fornecedores
para análise das margens da Target, análise de negociações ou mesmo passados anteriormente.
concluir essa análise e passar para outra ramificação. Se necessário, guie o  Esses dados podem levar o entrevistador a uma análise de lucratividade da
candidato ao próximo passo baseado nas informações que já foram passadas Target por tipo de produto e por fornecedor, a fim de esclarecer melhor a
durante o case. posição da VF em negociações. Para isso, o candidato precisará do Chart 3.

Clube de Consultoria UFC 111


Feedback aqui!
Case 8: Where’s the Party
4. Chart 3: Quais são as vendas por tipo de produto por loja? 5. Analysis: Qual é o lucro da Target?

Sugestão de Abordagem: Sugestão de Abordagem:

Volume de vendas por loja  O preço de cada item é $1, logo valor do volume de Cartões de Papéis de
Laços TOTAL
vendas corresponde ao valor das vendas. presente presente

Em todos os tipos de produtos, o competidor vende mais que a VF. Vendas VF R$ 1,5M R$ 1,5M R$ 1M R$ 4M

Margem da R$ 0,10 ou R$ 0,05 ou R$ 0,15 ou


A VF vende menos cartões de presente que laços e papéis de presente. -
Target 10% 5% 15%

Conclusão: Lucro da Target R$ 150K R$ 75K R$ 150K R$ 375K


A VF vende menos itens que seu concorrente, o que pode dificultar sua
negociações para vender em mais lojas da Target.
Além disso, a menor proporção de vendas de cartões de presente é prejudicial não Vendas Comp. R$ 2M R$ 2M R$ 2M R$ 6M
só para a VF, mas também para a Target, já que esse produto possui maior Margem da R$ 0,10 ou R$ 0,05 ou R$ 0,15 ou
margem para ambas. -
Target 10% 5% 15%
Guia para o Entrevistador
Lucro da Target R$ 200K R$ 100K R$ 300K R$ 600K
 Essa conclusão faz referência aos dados de margem dos fornecedores
passados anteriormente e da Target (Chart 2) e ao número de lojas (Chart 1).
 Esses dados podem levar à análise da lucratividade da Target. Para isso, o Conclusão:
candidato precisará do Chart 2. Devido a maior venda por loja, em especial de cartões de presente, a Target tem
 O resultado desta análise poderá influenciar as negociações com a Target. mais incentivos para vender produtos do competidor que da VF.
 Após a análise, o candidato pode precisar voltar para seu Framework para
descobrir por que o competidor tem mais vendas por loja.

Clube de Consultoria UFC 112


Feedback aqui!
Case 8: Where’s the Party
6. Chart 4: A disposição dos produtos na prateleira está 7. Chart 5: A disposição mais detalhada dos produtos na
influenciado as vendas? prateleira está influenciado as vendas?

Sugestão de Abordagem: Sugestão de Abordagem:

Espaço na prateleira para VF  6 x ( 2/6 de 8) = 16 pés² Competidor  10 / 18 ≈ 56% do espaço da prateleira é ocupado por cartões em
uma disposição que facilita a visualização por pessoas que estão procurando papel
Espaço na prateleira para VF  8 x ( 3/9 de 6) = 16 pés² ou laço.

Conclusão: VF  mesma quantidade de cada produto; produtos com margens maiores


Observa-se que não há diferença em relação ao espaço na prateleira em si, mas localizados em posições de mais difícil alcance e disposição.
algumas hipóteses podem ser levantadas em relação as disposições:
1. Alguns produtos da VF estão muito altos para algumas pessoas alcançarem, Conclusão:
enquanto o competidor não deve apresentar esse problema. A disposição e quantidade dos tipos de produtos nas prateleiras é um outro motivo
2. Alguns produtos estão colocados muito baixo, o que pode dificultar a alcance para a diminuição das vendas, já que os produtos com maior margem encontram-
dos mesmos por pessoas que têm dificuldade de se abaixar. se em posições de difícil alcance e visualização.
3. Há perda de visualização de produtos que estão localizados muito alto ou Além disso, o competidor está usando uma estratégia de maior volume e melhor
baixo nas prateleiras. visualização do produto com maior margem, o que pode melhorar sua relação com
Esses podem ser alguns motivos para as vendas menores que do competidor. as lojas da Target.

Guia para o Entrevistador Guia para o Entrevistador


 Confirme para o candidato que isso é parte do motivo das vendas menores.  Com isso, o candidato deve perceber que conseguiu encontrar os motivos para
Deixe-o exploras outros possíveis motivos. as menores vendas por loja em relação ao competidor. Caso, já tenha
 Em seguida, o candidato pode realizar alguma associação com as vendas por determinado a menor quantidade de lojas nas quais a VF atua e já tenha
tipo de produto e solicitar uma análise mais detalhada da prateleira. Caso ele resolvidos os demais pontos do seu framework, ele deve perceber que está
não o faço, o guie nesse caminho. pronto da dar uma recomendação.

Clube de Consultoria UFC 113


Feedback aqui!
Case 8: Where’s the Party
8. Recommendation: O que você gostaria de recomendar à Vamos Festejar?

Sugestão de Abordagem: Guia para o Entrevistador


 A recomendação deve sugerir opções de soluções
para os problemas encontras, mas também deve
Recomendação apresentar os pontos mais relevantes do
A Vamos Festejar (VF) deve negociar com lojas da Target para aumentar a quantidade de lojas na qual seus diagnóstico realizado.
produtos são vendidos e focar em melhorar suas vendas por loja ao solicitar uma mudança para uma disposição  É recomendado que o candidato apresente risco e
horizontal dos produtos vendidos na prateleira aumentar a proporção de cartões de presente nas prateleiras. próximos passos ao dar sua recomendação. Caso
o candidato não comente, peça para que ele
acrescente essas partes.
Motivos  Há diversos riscos e próximos passos possíveis. O
1. A VF ar atua em 5 lojas a menos em relação ao seu competidor. candidato só deve ter para os riscos não
2. A VF apresenta R$ 2M a menos de vendas por loja em relação ao competidor devido à disposição vertical contrariarem alguma parte anterior da análise.
dos produtos na prateleira que dificulta o alcance e visualização dos produtos com maior margem.
3. Os cartões de presente apresentam margem maior para a VF e para a Target, de modo que o aumento das
suas vendas pode melhorar o relacionamento e poder de negociação da VF com a Target.

Riscos Próximos Passos


1. Baixo poder de negociação e perda de 1. Buscar melhorar o relacionamento com os
contratos com lojas devido a resultados gerentes das lojas da Target.
atuais. 2. Preparar propostas para negociações que
2. Dificuldade em convencer a Target a mudar mostrem o diagnóstico e possibilidade de
disposição da prateleiras. recuperação.

Clube de Consultoria UFC 114


Feedback aqui!
Case 8: Where’s the Party
Chart 1: Número de lojas que vendem produtos da VF e do competidor

Vendas na lojas da Número de lojas da


Targer Target

Vamos festejar R$ 100M 25

Competidor R$ 180M 30

Clube de Consultoria UFC 115


Feedback aqui!
Case 8: Where’s the Party
Chart 2: Margens da Target por tipo de produto

Laço Papel de presente Cartão de presente

Vamos festejar R$ 0,10M R$ 0,05M R$ 0,15M

Competidor R$ 0,10M R$ 0,05M R$ 0,15M

Clube de Consultoria UFC 116


Feedback aqui!
Case 8: Where’s the Party
Chart 3: Vendas por tipo de produto por loja

Laço Papel de presente Cartão de presente TOTAL

Vamos festejar R$ 1,5M R$ 1,5M R$ 1M R$ 4M

Competidor R$ 2M R$ 2M R$ 2M R$ 6M

Clube de Consultoria UFC 117


Feedback aqui!
Case 8: Where’s the Party
Chart 4: Design da prateleira

Prateleiras da Vamos Festejar na Target Prateleiras do competidor na Target

6 pés
6 pés

8 pés 8 pés

Clube de Consultoria UFC 118


Feedback aqui!
Case 8: Where’s the Party
Chart 5: Design detalhado da prateleira

L  laço
Prateleiras da Vamos Festejar na Target Prateleiras do competidor na Target P  papel de presente
C  cartão de presente
L L
L L
L L
P P C C C C L L
6 pés

6 pés
P P P P C C L L
P P P P C C C C
C C
C C
C C
8 pés 8 pés
Clube de Consultoria UFC 119
Feedback aqui!
4. Decisão Estratégica

Market Entry
Indústria: Automotiva / Seguros

Led By: Candidate

Quantitativo Qualitativo

Case 9
Safe Car
Clube de Consultoria UFC 120
Case 9: Safe Car
Introdução
Contexto: Estamos no final de 2016. Seu cliente é Safe Car, uma empresa de seguros de carros. Seus seguros são completos, com elevados serviços e benefícios inclusos.
Para oferecer tudo isso, a Safe Car atua no mercado de clientes que não são sensíveis a preço.

Situação: A CEO da Safe Car tem a ambição de aumentar seu Market Share em 5 pontos percentuais nos próximos 5 anos. Contudo, eles fizeram algumas análises e
concluíram que não conseguiram alcançar essa meta se continuarem atuando apenas com clientes não sensíveis ao preço.

Problema: Para alcançar esse objetivo, eles estão considerando entrar na parcela do mercado composta por clientes sensíveis a preço. A CEO da Safe Car quer sua ajuda
para responder duas perguntas:
1. O mercado de clientes sensíveis a preço é relevante?
2. Quanto desse mercado a Safe Car precisa capturar para atingir sua meta?

Clarify (Informações adicionais, caso solicitadas)


 O Market Share atual da Safe Car é de 10% (em relação ao mercado total)
 Mercado relevante: comparativamente grande, com bom crescimento e poucas barreiras de entrada
 Os clientes atuais da Safe Car são pessoas físicas
 Clientes sensíveis a preço consideram não apenas o preço do seguro, mas seu custo-benefício

Guia para o Entrevistador


 O candidato deve perceber que esse é um case de entrada no mercado. O candidato deve focar em responder a primeira pergunta antes de ir para a segunda. Seu
primeiro passo deve ser criar um framework que o ajude a responder a primeira pergunta.
*Case candidate-led: a sequência de etapas para a resolução deste case é apenas uma sugestão, visto que o candidato deve guiar a resolução.

Mandou bem! Pisou na bola...

 Apresentar no framework e começar sua análise pela determinação do  Criar um framework genérico de entrada no mercado
tamanho e crescimento do mercado.  Não se deixar ser guiado a segmentação entre clientes sensíveis e não sensíveis
 Focar em responder às perguntas

Clube de Consultoria UFC 121


Feedback aqui!
Case 9: Safe Car
1. Framework: O que é necessário analisar para determinar se o mercado de clientes sensíveis a preço é relevante?

Sugestão de Abordagem: Informações Adicionais (Caso solicitadas):


Guia para o Entrevistador
 Mercado total de seguros em 2016: R$30B
 Crescimento
A estrutura podeda parcela
ser feitasensível a preço:
de várias formas. 30% em
5 anos é interessante que o candidato considere
Todavia,
 Tamanho e crescimento
 Crescimento
o que pode ser dafeito
parcela
comnão sensívelcomo
o produto, a preço:
1. Mercado  Segmentação entre sensíveis e não sensíveis
10% em 5 eanos
monetizar algumas dificuldades envolvidas para
 Competidores: quem são e Market Share
realizar a operação.
 Caso
Guia parao o Entrevistador
candidato não tenha feito perguntas
 Se o candidato
suficientes sobrenão iniciar ana
o produto análise
etapapelo tamanho,
inicial, é
 Altas barreiras de entrada no mercado sensível crescimento
interessante e segmentação
que do mercado
sejam adicionadas entre
no framework.
2. Riscos  Diluição da marca  clientes
Deixar osensíveis
candidatoepriorizar
não sensíveis a preço,
o caminho. Caso o guie
não
 Canibalização nesse
priorizecaminho.
as possíveis ações que podem ser
 Quando o candidato
realizadas, perguntar
forçar o caminho dasobre a
resolução para
segmentação do mercado entre sensíveis e não
esta parte do case.
sensíveis, pergunte quais critérios são relevantes
 Conhecimento e experiência no mercado de seguros para diferenciar clientes sensíveis e não sensíveis.
3. Vantagens  Imagem conhecida Pelo menos renda deve ser citado, mas o
 Aproveitamento de parceiras para entrada no mercado sensível candidato não deve se prender apenas à isso.
 Depois desse rápido Brainstorming, apresente o
Chart 1.

Clube de Consultoria UFC 122


Feedback aqui!
Case 9: Safe Car
2. Chart: Quais desses critérios seriam considerados por 3. Analysis: O mercado sensível é relevante?
clientes sensíveis a preço ao escolher um seguro de carro?
Sugestão de Abordagem:
Sugestão de Abordagem:
Pontos principais do gráfico: Mercado atual  R$30B = Classe média (10B) + Classe Alta (20B)
1. Mais da metade da classe média leva em consideração a Cobertura básica
Mercado sensível  10B x 0,8 + 20B x 0,3 = R$ 14B
2. A classe alta costuma considerar mais o Corretor
Mercado não sensível  30B – 14B = R$ 16B

Conclusão/Insights: Mercado em 5 anos:


Os critérios que envolvem custo benefício são a Cobertura Básica e a Cobertura Mercado sensível  14B x 1,3 = R$ 18,2B
Premium, logo esses são os critérios que os clientes levaram em consideração Mercado não sensível  16B x 1,1 = R$ 17,6B
para decidir se irão adquirir ou não um seguro de carro. Isso representa 80% da
classe média e 30% da classe alta.
Conclusão:
Os clientes não sensíveis, por sua vez, consideram critérios como Marca, Corretor
O mercado sensível representa quase a metade do mercado total atualmente e
(vendedor do seguro) entre outros.
em 5 anos ele será ainda maior que o mercado não sensível no qual Safe Car
atua no momento, de modo que pode-se afirmar que o mercado sensível é
Next-Steps: relevante sob esse ponto de vista.
Utilizar as informações no gráfico para determinar a parcela do mercado que é
sensível a preço e a parcela que não é sensível. Informações Adicionais (Caso não informadas anteriormente):
 Crescimento do mercado sensível: 30% em 5 anos
Guia para o Entrevistador
 Crescimento do mercado não sensível: 10% em 5 anos
 Mostrar os gráficos que estão ao final deste case.
 Caso perceba uma confusão no entendimento do gráfico, explique-o de forma Guia para o Entrevistador
mais detalhada, por exemplo: algumas pessoas consideram Cobertura Premium  Caso o candidato não mencione acrescimento, o guie nessa direção.
como mais relevante na aquisição, enquanto outras irão tomar sua decisão  O candidato deve ser então para a análise de quanto é necessário capturar
baseada na Marca do seguro. para alcançar a meta.

Clube de Consultoria UFC 123


Feedback aqui!
Case 9: Safe Car
4. Analysis: Quanto do mercado sensível deve capturar? 5. Recomandation: O que pode dizer à CEO da Safe Car em
relação às perguntas?

Sugestão de Abordagem:

Mercado total em 5 anos  18,2B + 17,6B = R$ 35,8B Recomendação


O mercado de clientes sensíveis a preço se mostra relevante e será necessário
Parcela do mercado que deve capturar  35,8B x 0,05 = R$ 1,79B capturar R$ 1,79B dessa parcela do mercado.

Conclusão:
Para alcançar a meta de capturar 5% do mercado de seguros em 5 anos, Safe Car Motivos
irá precisar capturar R$ 1,79B da parcela do mercado de clientes sensíveis a 1. Atualmente, o mercado sensível equivale a 14B do mercado total de 30B.
preço, correspondendo a quase 10% dessa parcela. Conhecer mais sobre os 2. Em 5 anos, ele irá representar mais da metade do mercado com 18,2B, de
players desse mercado e estratégia de entrada ainda é necessário para determinar modo que para alcançar sua meta, será necessário capturar 10% desse
se será, de fato, possível alcançar esse Market Share. valor.
3. É esperada uma taxa de crescimento 3 vezes maior que a do mercado não
Informações Adicionais (Caso solicitadas):
sensível
 Pode-se assumir que a Safe Car conseguirá se manter com o Market Share de
10% do mercado, ainda que o crescimento do mercado não sensível seja mais
devagar que o crescimento do mercado total.
Riscos
1. Dificuldades de criação e produção própria
Guia para o Entrevistador
2. Diminuição da demanda do minério X nos próximos anos.
 No final desta análise, o candidato deve perceber que respondeu às duas
perguntas do case e deve passar para a recomendação. Próximos Passos
1. Analisar oportunidade de vender a patente da nova broca.
2. Verificar viabilidade de otimização logística a partir do ano 15.

Clube de Consultoria UFC 124


Feedback aqui!
Case 9: Safe Car
Chart 1: Critérios considerados na aquisição de um seguro de carro

Critérios considerados na aquisição de um seguro de carro


100%

90% 23%
80%
7%
70% 62%
10%
60%

50%
30%
40%

30% 18%
20%
5%
30%
10% 10%

0% 5%
Classe média $10B Classe alta $20B

Outros Corretor Marca Cobertura Premium Cobertura Básica

Clube de Consultoria UFC 125


Feedback aqui!
4. Decisão Estratégica

M&A / Valuation
Indústria: Varejo (restaurante)

Led By: Interviewer

Quantitativo Qualitativo

Case 10
Greta Coffeehouse
Clube de Consultoria UFC 126
Case 10: Greta Coffeehouse
Introdução
Contexto: Estamos em 2021, durante o período de pandemia da covid-19, em Miami. Apesar de a cidade ter passado por uma fase de lockdown por cerca de um ano,
consideremos um cenário pós-lockdown, em que a economia local está se recuperando e a vacinação da população dos EUA está alcançando resultados positivos.

Situação: Você é um(a) jovem consultor(a) e está jantando sushi no rooftop de um restaurante local com sua/seu namorada(o). Então você se depara com uma cafeteria
chamada Greta Coffeehouse. Greta é um restaurante temático sobre filmes dos anos 70 e possui um público majoritariamente formado por turistas. Atualmente, o negócio
possui 22 lojas espalhadas por algumas cidades dos EUA e possui uma série de ineficiências por ser uma empresa familiar. Além da geração de receita por meio da venda de
alimentos, a cafeteria apresenta um mercado interno que vende objetos temáticos dos anos 70, como camisas, canecas, baralhos, etc.

Problema: Sua/seu namorada(o) conhece um dos executivos dessa cafeteria e comenta que ela está a venda por um valor de 600 milhões de dólares. Você possui um amigo
chamado Marcus Lemonis que é proprietário de uma private equity de porte médio e acredita que essa possa ser uma ótima oportunidade para ele.

1. Framework: Quais pontos você analisaria antes de ligar para seu amigo e informar que esse negócio é uma boa oportunidade?
Clarify (Informações adicionais, caso solicitadas)
 Ela não fazia delivery durante o período de lockdown, pois seu modelo de negócio possui um foco na experiência do cliente no ambiente da cafeteria.
 Objetivo do PE: comprar a empresa, buscando valorizá-la através de estratégia e gestão profissionais, para posterior venda.
 O PE deste estudo não possui empresas de setores que poderiam promover sinergias com a cafeteria Greta.

Guia para o Entrevistador


 Esse case irá abordar a capacidade do candidato, principalmente, em duas frentes: (1) perceber as implicações da pandemia em um negócio de turismo, (2) estruturar
uma grande quantidade de informações numéricas e calcular de forma efetiva.

Mandou bem! Pisou na bola...


 Entender que a empresa pode está passando por dificuldades financeiras,  Não considerar o valuation da empresa.
aumentando o poder de barganha do comprador.  Não considerar uma possível retomada do crescimento do mercado de turismo diante
 Considerar oportunidades de crescimento do negócio no pós-pandemia. da pandemia.

Clube de Consultoria UFC 127


Feedback aqui!
Case 10: Greta Coffeehouse
1. Framework: Quais pontos você analisaria antes de ligar para seu amigo e informar que esse negócio é uma boa oportunidade?

Sugestão de Abordagem: Informações Adicionais (Caso solicitadas):


 O governo dos EUA injetou uma grande
quantidade de dinheiro na economia, esperando
uma demora na retomada das atividades de
 Valuation da empresa está abaixo do normal? negócios presenciais. Contudo, foi surpreendido
1. Greta  Saúde financeiro boa: lucro e fluxo de caixa? Teremos poder de barganha? com a rapidez da vacinação, a qual poderá
Coffeehouse  Bem renomada pelas clientes? (marca forte) garantir 85% da população imunizada dentro do
período de 1 ano.

 Existe uma projeção de crescimento do mercado de turismo? (pós-pandemia) Guia para o Entrevistador
2. Mercado de  Turistas irão priorizar restaurantes temáticos em suas viagens?  Principal hipótese do case: o PE pode ganhar
turismo  Apoio governamental durante a pandemia será satisfatório? muito dinheiro com esse negócio, através da
compra por um preço inferior ao seu valor real,
visto que a empresa está com dificuldades
financeiras devido à queda do turismo durante a
 Aumentar a receita por cafeteria: mix de produtos, fachada/layout, capacidade. pandemia. Ademais, é esperada uma retomada da
3. Oportunidades  Viabilidade de aumentar o número de restaurantes economia e, consequentemente, do turismo,
de alavancagem  Redução de custos: otimizar processos, redução de pessoal, renegociar fazendo com que a empresa possua uma
contratos. valorização natural, além das possíveis ações que
o PE fará na empresa.
 Caso o candidato não aborde tal hipótese em sua
 PE possui disponibilidade financeira para realizar a compra e os investimentos
4. Implementação necessários?
estrutura, pergunte: por que essa empresa e por
que agora?
(PE)  PE possui a expertise no setor de restaurantes e turismo?
 Caso ele aborde, afirme que pode-se adotar como
 Existem sinergias com os atuais negócios do PE? (caso não abordado no clarify)
verdade.

Clube de Consultoria UFC 128


Feedback aqui!
Case 10: Greta Coffeehouse
2. Analyses: Temos alguns dados do negócio em um período pré-pandêmico. Qual seria o valor do Greta Coffeehouse em um
período antes da pandemia?
Receita: Custos:
Ticket médio da refeição por cliente: $18,00 Custo variável: 30% da receita
Preço médio do drink: $12,00 Número de funcionários: 50
Ticket médio das compras no mercado: $20,00 Salário médio e encargos por funcionários: $70.000,00/ano
Capacidade do restaurante: 200 pessoas Aluguel: $2,7 milhões/ano
Rotatividade das mesas: 1h Marketing e outros: $650 mil/ano
Aberto 7 dias na semana: Considerar 350 dias

Sugestão de Abordagem (1): Informações Adicionais (Caso solicitadas):


 Formula de valuation simplificada:
# dias (lucro anual) / (taxa de desconto).
 Valores podem ser considerados para os demais
Receita restaurantes
 Tempo aberto: 11am-11pm
Receita/dia  Ocupação média: 80%
Lucro anual  Clientes que compram drink: 25%
 Clientes que compram no mercado: 10%
Custos variáveis  Taxa de desconto: 10%
Valuation
Custos Guia para o Entrevistador
 Fornecer todos os dados numéricos ao mesmo
Custos Fixos tempo. Um bom candidato irá confirmar os dados
Taxa de desconto recebidos e comunicar a estrutura de cálculo
antes de realizá-lo.

Clube de Consultoria UFC 129


Feedback aqui!
Case 10: Greta Coffeehouse
2. Analyses: Temos alguns dados do negócio em um período pré-pandêmico. Qual seria o valor do Greta Coffeehouse em um
período antes da pandemia?
Sugestão de Abordagem (2):
160 Capacidade 200

# dias 350 Ocupação média


1.920
10,85M Ocupação 80%
Vendas/dia
Margem de
4M Contribuição/ano
30.720 = ~31k Horário aberto (h) 12
Lucro anual 12
Margem de Rotatividade de
Contribuição/dia cliente/dia
40M 23*0,7 = ~16 Tempo médio do
Margem de Ticket Médio = 23 cliente (h) 1
Valuation (1loja) Contribuição (u) CV = 30%
50*70k = 3,5M
Custos Fixos 2,7M + 0,65M = 3,35M Ticket Médio % compra Valor Repres.
= 6,85M
40M * 22
Refeição 100% 18 18

= 880M Bebida 25% 12 3


Taxa de desconto 10%
Mercado 10% 20 2

Total - - 23
Clube de Consultoria UFC 130
Feedback aqui!
Case 10: Greta Coffeehouse
3. Brainstorming: Caso a cafeteria Greta seja comprada pela empresa de PE, quais as principais ações que poderiam ser feitas
para aumentar o valor da empresa?
Sugestão de Abordagem: Informações Adicionais (Caso solicitadas):
 Aumentar preço  Pode-se considerar que aumentar valor da
 Marketing interno para empresa seria aumentar o lucro, visto que é um
bebidas e mercadorias principais direcionadores.
Ticket médio (aumentar %)  Não é necessário se preocupar com os
 Lojas próprias  Mais combos no cardápio investimentos necessários ou a retomada do
# de lojas  Franquias mercado de turismo. Pode-se afirmar que o PE
 Upselling/crossleing no
atendimento estará disposto a realizar investimentos grandes
caso sejam lucrativos e o período de pandemia
está ao fim nos EUA.
Aumentar receita  Delivery
 Take away
Aumentar  Aumentar capacidade Guia para o Entrevistador
Valor  Bons candidatos irão interpretar as informações
(horários de pico)
# de vendas  Divulgação: Digital, encontrados no cálculo, comparando o valor
influencers, etc sugerido e o valor da empresa encontrado.
 Parceiras com companhias  Outra análise relevante é encontrar a lucratividade
Lucro por loja aéreas, taxi/uber, etc do restaurante. Pode-se solicitar um cálculo
 Fachada mais chamativa aproximado como uma questão bônus para o case
4M/15,5M = ~26%. A qual é, consideravelmente,
alta para restaurantes (média entre 10% e 20%).
 Otimizar processos (reduzir tempo e desperdícios)
Essa informação demonstra uma oportunidade em
 Enxugar quadro de funcionários
Reduzir custos abrir mais lojas, visto que é um negócio lucrativo.
 Economia de escala com mais lojas
 Renegociar/alterar fornecedores
 Renegociar alugueis
Clube de Consultoria UFC 131
Feedback aqui!
Case 10: Greta Coffeehouse
4. Recommendation: Por sorte seu amigo te liga é após uma conversa rápida sobre assuntos avulsos, você comenta que possui
algo para falar para ele. O que você recomendaria para seu amigo, com base no que você encontrou no caso?
Sugestão de Abordagem: Guia para o Entrevistador
 De acordo com as informações fornecidas e
encontradas durante o caso, a recomendação
Recomendação possui uma tendência de realização da compra.
Marcus, tenho uma ótima notícia para você! Eu estava analisando a empresa Greta Coffeehouse e ela pode ser
Todavia, também é aceitável o candidato realizar
uma oportunidade muito boa para seu Private Equity, por 4 motivos principais:
uma conclusão de não realização do investimento,
caso ele consiga defender suas opiniões com
boas argumentações.
Motivos  Por ser um telefonema para um amigo, é
1. Ela está sendo vendida por um preço menor do que ela valia no período antes da pandemia. 700M < ~1B interessante analisar a flexibilidade do candidato
2. É possível ter um poder de barganha relevante na negociação, pois a empresa está com dificuldades em ser menos formal, porém continuar
financeiras. estruturado.
3. O mercado de turismo possui uma tendência de recuperação para os próximos anos, o que pode fazer a  Lembrar de mencionar o nome do amigo durante a
empresa valorizar sem mudanças bruscas. recomendação é um ponto positivo. As pessoas
4. É possível aumentar o valor da empresa de forma significativa, por meio do aumento do número de lojas, gostam de serem chamadas pelo nome.
visto que sua lucratividade é de aproximadamente 26%.

Riscos Próximos Passos


1. Pandemia piorar e prejudicar o turismo. 1. Barganhar o valor da empresa para valores
2. PE não possuir a expertise necessária para o menores que o solicitado de 600M.
negócio de cafeteria temática. 2. Análises financeiras de possíveis projetos que
para valorização da empresa, como abrir mais
lojas e otimizar custos dos processos.

Clube de Consultoria UFC 132


Feedback aqui!
4. Decisão Estratégica

Market Entry
Indústria: Construção Civíl / Concessão

Led By: Interviewer

Quantitativo Qualitativo

Case 11
BR Rodovias
Clube de Consultoria UFC
Fonte: Casebook Wharton 2017 – Brazilian Highway Concessions
133
Case 11: BR Rodovias
Introdução
Contexto: Nosso cliente é a BR Rodovias, uma das principais empresas brasileiras de concessões de rodovias (desconsiderar período de pandemia).

Situação: A empresa busca se expandir internacionalmente, pois o crescimento econômico no Brasil está estagnado. Assim, com intuito de continuar a aumentando suas
receitas e lucros, o cliente deseja diversificar seu portfólio e diminuir sua exposição à economia brasileira.

Problema: Como existem diversas oportunidades internacionais, o cliente te contratou para analisar qual a mais rentável pensando a longo prazo e como implementar uma
entrada de mercado com sucesso.
1. Framework: Quais fatores o cliente deve considerar ao pensar em sua expansão internacional?
Clarify (Informações adicionais, caso solicitadas)
 A BR Rodovias atua apenas no Brasil.
 Os clientes da BR Rodovias são sempre municipais, estaduais ou nacionais (demais instituições governamentais).
 A empresa responsável por construir e operar as rodovias em uma determinada região é escolhida através de licitações. Atualmente, o cliente concentra-se apenas em
concessões rodoviárias (construção e operação de vias públicas). Operar: se responsabilizar por toda a administração da rodovia.
 Pode-se considerar todas as geografias, com uma tendência para oportunidades na América do Sul (proximidade com o Brasil).
 Definição de concessão: o Governo transfere, a uma organização terceira, o direito de realizar/explorar algo que, normalmente, seria de responsabilidade governamental.
Diferentemente de privatização, ao final do prazo estabelecido em contrato, o controle volta para o Governo.
Guia para o Entrevistador
 Esse case irá testar o business sense do candidato para problemas de market entry e análise financeira. Caso ele não possua conhecimento técnico em finanças, será
necessária a explicação detalhada de algumas etapas para dar seguimento ao caso.

Mandou bem! Pisou na bola...

 Levantar fatores culturais e distância geográfica como relevantes para a  Ignorar riscos políticos.
tomada de decisão.  Não realizar uma analise comparando a atratividade dos mercados (países).
 Considerar diferentes formas de entrada, como M&A e Joint Venture.

Clube de Consultoria UFC


Fonte: Casebook Wharton 2017 – Brazilian Highway Concessions
134
Feedback aqui!
Case 11: BR Rodovias
1. Framework: Quais fatores o cliente deve considerar ao pensar em sua expansão internacional?

Sugestão de Abordagem: Guia para o Entrevistador


 O framework apresentado possui tópicos que
 Quantidade e tipos de concessões programadas para o futuro devem ser explorados e comparados nos
1. Atratividade dos  Rentabilidade dos projetos potenciais mercados (países). Caso o candidato
Projetos do país  Tamanho e complexidade (necessidade de inovação?) tenha feito algo semelhante, ele deve informar
 Tendência de crescimento da economia do país antes de comunicar seu framework.
 O candidato deve sugerir uma abordagem, após a
 Principais concorrentes do mercado apresentação do framework, com pontos que
2. Ambiente ajudem a priorizar quais os melhores mercados
 Market-share dos concorrentes (fragmentado ou concentrado)
Competitivo (países) para a empresa atuar. Caso necessário,
 Força para vencer licitações (grande concorrência?)
deve-se guiar o candidato para esse caminho.

 Barreiras culturais e linguísticas


3. Cultura e
 Fit do país com a atual liderança da BR Rodovias
gerenciamento
 Distância geográfica (dificulta o gerenciamento?)

 Receptividade do Governo para novos negócios


4. Ambiente político  Atuais legislações (benéficas para concessões/privatizações?)
 Instabilidade política (risco de mudança da legislação)

 Iniciar do zero
5. Formas de Entrada  M&A
 Joint Venture

Clube de Consultoria UFC


Fonte: Casebook Wharton 2017 – Brazilian Highway Concessions
135
Feedback aqui!
Case 11: BR Rodovias
2. Chart 1: Após várias conversas com o cliente e uma análise 3. Brainstorming: Supondo que o cliente opte por entrar em
inicial da nossa equipe, decidimos que, devido as diferenças um ou mais desses mercados quais as possíveis formas de
culturais e a complexidade gerencial, as oportunidades fora da entrada no mercado? Quais os pontos positivos e negativos?
América do Sul não compensariam. A equipe reuniu os dados
abaixo para avaliar quais países da América do Sul seriam os Formas de
Positivos Negativos
mais atraentes. Entrada

Conclusão/Insights:  Maior controle  Conhecimento


Países do quadrante 3 (México, Colômbia, Chile e Peru), principalmente o México, Iniciar do zero limitado do mercado
são os mais vantajosos, pois:
1. Apresentam grandes mercados.
 Conhecimento do mercado  Menor controle
2. Possuem bons indicadores de facilidade de fazer negócios.
Joint Venture  Velocidade de entrada (limitações do
Next-Steps:
 Menor investimento que M&A contrato)
 Analisar ambiente competitivo entre os países.
 Priorizar México e analisar formas de entrada  Conhecimentos do mercado  Grande investimento
M&A (fusões
 Relacionamento c/ Governo  Riscos de integração
Informações Adicionais (Caso solicitadas): e aquisições)
 Velocidade de entrada
 Índice de Facilidade de Fazer Negócios é uma medida do desempenho das
economias quanto à regulação do ambiente de negócios em um determinado
período. Quanto menor o indicador, maior a facilidade. Guia para o Entrevistador
 Caso o candidato não tenha conhecimento sobre as formas de entrada de
Guia para o Entrevistador mercado, deve-se explicar antes de seguir com o case.
 Após o chart 1 ser apresentado, o candidato deve perguntar, se necessário,  Após o brainstorming ser finalizado, pergunte ao candidato “Quais os principais
sobre o índice do eixo X antes de realizar conclusões. indicadores financeiros que poderíamos utilizar para avaliar esses
 Caso o candidato não realize conclua os principais insights do chart 1, realizar investimentos?”. Exemplo de resposta: lucro anual (EBITDA), payback, valor do
pergunta “Quais mercados nosso cliente deve focar seus esforços? Quais investimento (valuation) e ROIC.
devemos eliminar da nossa análise?”.

Clube de Consultoria UFC


Fonte: Casebook Wharton 2017 – Brazilian Highway Concessions
136
Feedback aqui!
Case 11: BR Rodovias
4. Chart 2/Analyses: Devido aos melhores resultados nos indicadores, seguiremos uma análise mais detalhada do mercado do
México. Foi determinado que não há oportunidades viáveis ​de JV e, portanto, devemos decidir entre iniciar o investimento do zero
ou realizar um M&A. Com os dados a seguir, qual dos investimentos é o mais vantajoso?
Sugestão de Abordagem: Informações Adicionais (Caso solicitadas):
 Lucro anual = (Receita anual)*(1 – Opex).
 Payback = Invest. inicial / Lucro anual.
 Valor do Invest. = Lucro Anual / Taxa de
Tópicos Iniciar do Zero M&A Desconto.
 ROIC = (Valor do invest. / Invest. Inicial) – 1
Receita Anual ($) 300*5*20.000 = 30M 120M
 Sinergias: lucro extra do investimento (não
considerar Opex).
Lucro Anual ($) 30M*0,7 = 21M 120M * 0,6 + 120*0,15 = 90M

Payback (anos) 150M / 21M = ~7 anos 750M / 90M = ~8,33 anos Guia para o Entrevistador
 Caso o candidato não tenha conhecimentos em
finanças, informar indicadores financeiros após a
Valor do Investimento ($) 21M / 0,1 = 210M 90M / 0,1 = 900M
divulgação do chart 2.
 O candidato deve ressaltar alguns pontos antes de
ROIC (%) (210M / 150M) – 1 = 40% (900M / 750M) – 1 = 20% iniciar os cálculos, como a opção de M&A possuir
investimento 4x maior, assim como um maior
Opex (A empresa comprada talvez seja menos
eficiente que a BR Rodovias).
 Após ser calculado o ROIC, guiar o candidato para
a recomendação.

Clube de Consultoria UFC


Fonte: Casebook Wharton 2017 – Brazilian Highway Concessions
137
Feedback aqui!
Case 11: BR Rodovias
4. Recommendation: Vamos ter uma reunião com o CEO da BR Rodovias daqui a 1 hora. Qual seria sua recomendação de acordo
com as análises que fizemos?
Sugestão de Abordagem: Guia para o Entrevistador
 De acordo com o que foi analisado no case, a
melhor opção e a mais fácil de ser defendida é a
Recomendação de iniciar do zero e crescer organicamente. Outras
A BR Rodovias deve entrar no mercado do México por meio de um investimento primário (iniciando do zero) e
recomendações irão exigir uma boa argumentação
crescer organicamente:
do candidato e uma série de assumptions.

Motivos
1. Países da América do Sul apresentam altos níveis de similaridade cultural e baixos níveis de
complexidade gerencial para a BR Rodovias.
2. México é o país que apresenta o maior mercado para os próximos 5 anos e a melhor facilidade para
realizar negócios.
3. Em comparação com a opção de M&A, O investimento primário (iniciando do zero) possui um menor
payback (7 anos contra 8,3 anos) e maior ROIC (40% contra 20%).

Riscos Próximos Passos


1. Reação dos concorrentes. 1. Analisar probabilidade de vencer licitações
2. Fatores macroeconômicos influenciarem os iniciando do zero.
dados encontrados e dificultar o crescimento 2. Renegociar valor cobrado no M&A.
orgânico. 3. Analisar outros países que possuem bons
indicadores, como Colômbia, Chile e Peru.

Clube de Consultoria UFC


Fonte: Casebook Wharton 2017 – Brazilian Highway Concessions
138
Feedback aqui!
Case 11: BR Rodovias
Chart 1: Atratividade dos países da América do Sul

9
Tamanho do Mercado em Bilhões $
(previsão para os próximos 5 anos)
México
8
7 Colômbia

6 Chile
Argentina
5
Peru
Equador
4
3
Paraguai
2 Bolívia
Venezuela
Costa Rica Panamá
1 Honduras

0
0 20 40 60 80 100 120 140 160 180 200
Índice de Facilidade de Fazer Negócios do Banco Mundial

Clube de Consultoria UFC


Fonte: Casebook Wharton 2017 – Brazilian Highway Concessions
139
Feedback aqui!
Case 11: BR Rodovias
Chart 2: Comparação de Investimentos

Tópicos Iniciar do Zero M&A

Km 300 -

$/Km 5,00 -

Tráfego esperado
20.000 -
(veículos/ano)

Receita anual (Km)*($/ano)*(veículos/ano) $ 120.000.000,00

Opex 30% 40%

Investimento Inicial $ 150.000.000,00 $ 750.000.000,00

Taxa de desconto 10% 10%

Sinergias - 15% (das receitas)

Clube de Consultoria UFC


Fonte: Casebook Wharton 2017 – Brazilian Highway Concessions
140
Feedback aqui!
5. Estratégia Aberta

Revenue Growth
Indústria: Varejo (CPG)

Led By: Candidate

Quantitativo Qualitativo

Case 12
Mobi Clean
Clube de Consultoria UFC
Fonte: Casebook Ross 2010 – Household Cleaners Growth
141
Case 12: Mobi Clean
Introdução
Contexto: Nosso cliente é a Mobi Clean, uma empresa global de bens de consumo (CPG), que fabrica produtos de limpeza de residências. Podem-se dividir os produtos
comercializados em 3 categorias: papel toalha, detergente e desinfetante.

Situação: O conselho de administração da empresa estabeleceu uma meta agressiva de dobrar a receita líquida em um período de 4 anos.

Problema: O CEO da Mobi Clean veio até você, especificamente, com o intuito de avaliar a posição da empresa e desenvolver uma estratégia de crescimento que alcance o
objetivo estabelecido. Qual seria seu plano para ajudar o CEO bater sua meta? Quais ações a Mobi Clean deve realizar?

Clarify (Informações adicionais, caso solicitadas)


 Não considerar período de pandemia.
 Receita líquida da empresa: 2 bilhões de dólares.
 Vendas líquidas: vendas no varejo (receita bruta) menos gastos comerciais. Gastos comerciais é o que os fabricantes pagam para distribuidores ou varejistas para
incentivá-los a vender seus produtos aos consumidores finais.
 Os principais canais de distribuição são supermercados e lojas de materiais de construção.
 A empresa possui um grande foco em sustentabilidade.

Guia para o Entrevistador


 Esse é um case de growth strategy, em que o candidato deve considerar a dinâmica do mercado, ações internas e ações externas para alcançar o objetivo.
 Caso o candidato pergunte se a empresa possui alguma estratégia específica, peça que ele monte uma estrutura com as possíveis ações e considerações.
*Case candidate-led: a sequência de etapas para a resolução deste case é apenas uma sugestão, visto que o candidato deve guiar a resolução.

Mandou bem! Pisou na bola...

 Considerar o crescimento do mercado.  Não focar em possíveis ações que possam aumentar a receita da empresa.
 Considerar crescimento inorgânico.  Priorizar análises relacionadas à custos/margem/retorno.
 Levantar prós e contras de algumas ações principais, como a de lançar novos  Esquecer, durante o case, de direcionar as ações para o atingimento da meta.
produtos.

Clube de Consultoria UFC


Fonte: Casebook Ross 2010 – Household Cleaners Growth
142
Feedback aqui!
Case 12: Mobi Clean
1. Framework: Qual seria plano para o CEO bater a meta? Quais ações a Mobi Clean deve realizar?

Sugestão de Abordagem: Informações Adicionais (Caso solicitadas):


 A receita é dividida igualmente entre as 3
categorias (33% para cada).
 A empresa tem margens de lucro baixas e não
 Crescimento natural do mercado quer assumir dívidas adicionais, então o dinheiro
 Estratégia dos concorrentes disponível para investimento é cerca de $ 600
1. Mercado  Possibilidade de novos entrantes milhões. (Não diga "para aquisição").
 Leis e agentes regulatórios  O Mercado atual seguirá com um CAGR de 10%
nos próximos 4 anos (crescimento anual).
 Preço e política de descontos
 Novos produtos
Receita por canal  Melhorias dos produtos atuais (ganhar market-
share): qualidade, embalagem etc Guia para o Entrevistador
2. Crescimento  O candidato pode apresentar uma abordagem
 Posicionamento nos canais: prateleiras etc
Orgânico exploratória, considerando o ambiente competitivo
da empresa e a capacidade de implementação
# de canais  Novos canais: farmácias, e-commerce etc
das estratégias. Todavia, devem ser analisadas,
 Expansão regional
de forma quantitativa, as possíveis estratégias que
podem ser realizadas para atingir a meta. Caso
não seja feito, pode-se considerar um ponto
3. Crescimento  M&A: players do mercado ou verticalização negativo e o entrevistador deverá guiar o
 Joint Venture: viabilidade de parcerias com players do mercado candidato para essa linha de raciocínio.
Inorgânico  Devido à meta ousada, o candidato deverá buscar
mais de uma forma de aumentar a receita.

Clube de Consultoria UFC


Fonte: Casebook Ross 2010 – Household Cleaners Growth
143
Feedback aqui!
Case 12: Mobi Clean
2. Brainstorming/Analyses: Quais novos produtos podem 3. Chart/Analyses: Qual empresa a Mobi Clen deve adquirir?
ser lançados pela Mobi Clean?

 Álcool: em gel e 70% 1. Crescimento acompanhado pelo mercado: 2B*(1,1)^4 = 2B*1,5 = ~3B
1. Novo produto no  Sabão em barra e em pó 2. Novo produto: (10*0,8)*40M = 320M
mesmo mercado  Amaciante
 Água sanitária Falta para bater a meta: 4B – 3B – 320M = 680M

 Papel toalha para banheiros, janelas etc Receita Organo Clean:


2. Produto do portfólio 300M*(1,1)^4 = 300M*(1,5) = ~450M
 Detergente (produto de limpeza) para veículos
em um novo mercado
 Desinfetantes para animais (pet friendly)
Receita Home Defense:
 Guardanapos 400M*(1,2)^4 = 400*(2) = 840
 Shampoo
3. Novo produto em um  Sabonete
novo mercado  Creme de barbear Informações Adicionais (Caso solicitadas):
 Esponjas  É possível adquirir apenas uma empresa. Limitações de orçamento.

Informações Adicionais (Caso solicitadas):


 Potencial e orçamento apenas para lançar um novo produto: sabão em barra. Guia para o Entrevistador
 Preço final por unidade: $ 10,00  Apresentar chart 1 para o candidato, após a pergunta.
 Taxa de venda: 20% (para encontrar receita líquida)  Solicitar que o candidato arredonde os valores encontrados
 Projeção de venda anual: 40 milhões de unidades.  O candidato ficará tentado de escolher a Home Defense para realizar a
Guia para o Entrevistador aquisição. Todavia, deve-se considerar a possibilidade de possíveis sinergias e
 O candidato deve priorizar produtos que sejam mais similares aos atuais riscos de aquisição.
comercializados pela Mobi Clean.  Após a análise de sinergias e riscos, será possível concluir que será possível
 Macro tópicos foram baseados na Matriz Ansoff excluindo a parte de mesmos bater a meta estabelecida apenas caso a Organo Clean seja adquirida.
produtos em mercados existentes (capítulo 3 deste casebook).
Clube de Consultoria UFC
Fonte: Casebook Ross 2010 – Household Cleaners Growth
144
Feedback aqui!
Case 12: Mobi Clean
4. Brainstorming: Quais as possíveis sinergias/oportunidades 5. Recommendation: Como você resumiria o que
e riscos/desvantagens enfrentados pela Mobi Clean na encontramos para o CEO da Mobi Clean
aquisição dessas empresas?
 Distribuição do produto (exposição Recomendação
Receita em mais canais de distribuição) É possível a Mobi Clean atingir a meta estabelecida pelo conselho de alcançar 4
1.Sinergias e  Co-branding (aliança entre marcas) milhões de receita por 3 motivos/ações principais.
Oportunidades  Economia de escala
Custos  Integração da produção
 Compartilhamento de boas práticas Motivos
 Redução pessoal em mesmos cargos 1. Será possível alcançar 3B de receita apenas acompanhando o crescimento
do mercado.
 Integração de cultura das empresas
2. Devemos comercializar um novo produto: sabão em barra, o qual irá garantir
 Missão ou conflito de valores
2. Riscos e 320 milhões de receitas extras.
 Fluxo de caixa disponível
Desvantagens 3. Devemos adquirir a empresa Organo Clean, teremos 450 milhões de receita
 Diluição da marca
da empresa e mais 250 milhões de sinergias.
 Leis antitruste (direito da concorrência)

Organo Clean: 450M + 250M = 700M > 680M


Home Defense: 840M*0,75 = 630M < 680M Riscos
1. Crescimento do mercado ou da Organo Clean não se concretizar.
Informações Adicionais (Caso solicitadas): 2. Novos negócios não serem lucrativos.
 Home Defense possui produtos que causam prejuízos ao meio ambiente. Caso
for adquirida pela Mobi Clean, será necessário cortar a fabricação de uma Próximos Passos
determinada linha de produtos que representa 25% da receita. 1. Avaliar potenciais custos e retornos dos investimentos.
 Organo Clean possui uma estrutura de distribuição na Europa que garante uma 2. Analisar possíveis reações dos concorrentes.
sinergia de $ 250 milhões com a Mobi Clean. 3. Iniciar um projeto de duo dilligence na Organo Clean.

Clube de Consultoria UFC


Fonte: Casebook Ross 2010 – Household Cleaners Growth
145
Feedback aqui!
Case 12: Mobi Clean
Chart 1: potenciais aquisições

Empresa Target Organo Clean Home Defense

Produtos Produtos orgânicos de limpeza Inseticidas e venenos

Valuation (ano 0) 400M 400M

Receita (ano 0) 300M 400M

CAGR projetado (da receita) 10% 20%

Clube de Consultoria UFC


Fonte: Casebook Ross 2010 – Household Cleaners Growth
146
Feedback aqui!
5. Estratégia Aberta

Public Sector / Market Sizing


Indústria: Setor Público

Led By: Interviewer

Quantitativo Qualitativo

Case 13
Organ Donation
Clube de Consultoria UFC
Fonte: Casebook Darden 2013 – Organ Donation Case
147
Case 13: Organ Donation
Introdução
Contexto: Você é um consultor experiente em uma grande empresa de consultoria estratégica e está começando um novo projeto. Seu cliente é a Comissão de Saúde do
Estado de Nova York. Uma das suas muitas funções é coordenar a doação de órgãos entre os muitos hospitais no Estado.

Situação: Nos últimos anos, a demanda por órgãos tem sido maior do que a oferta. Como resultado, muitos pacientes morrem a cada ano porque não há doações de órgãos
suficientes.

Problema: Você foi contratado para ajudar a determinar como aumentar o número de doações anuais de órgãos em Nova York.

1. Brainstorming: Quais são os fatores e drivers que determinam o número de órgãos doados em Nova York por ano?
Clarify (Informações adicionais, caso solicitadas)
 Em Nova York, os órgãos são retirados de pacientes com doenças terminais ou feridos pouco antes da sua morte, mas somente se eles forem doadores de órgãos
registrados ou se o hospital receber permissão de seus parentes próximos.
 Os residentes de Nova York podem optar por se registrar como doadores de órgãos ao solicitarem sua carteira de motorista.
 A Comissão de Saúde é um órgão público, não possuindo fins lucrativos.

Guia para o Entrevistador


 O candidato deve reconhecer que esse é um case de setor público, logo não tem fins lucrativos.
 O candidato pode dar sugestões de como prosseguir durante o case, mas deve primeiro focar em responder as perguntas direcionadas a ele.
 Caso o candidato insiste nas formas de registro, confirme que a única forma atual é ao solicitar a carteira de motorista.

Mandou bem! Pisou na bola...

 Foi criativo e “case specific” nas suas estruturas de framework e  Tentou aplicar estruturas e ideias genéricas que não se encaixam bem no case
brainstorming  Não conseguiu desenvolver o cálculo mesmo com a ajuda do entrevistador
 Desenvolveu e explicou bem os cálculos  Ficou muito preso as próprias sugestões de continuação do case, não focando em
 Focou em responder o que foi perguntado responder as perguntas
Clube de Consultoria UFC
Fonte: Casebook Darden 2013 – Organ Donation Case
148
Feedback aqui!
Case 13: Organ Donation
1. Brainstorming: Quais são os fatores e drivers que determinam o número de órgãos doados em Nova York por ano?

Sugestão de Abordagem: Informações Adicionais (Caso solicitadas):


Guia para o Entrevistador
 Órgãos doados  órgãos que chegam de fato aos
 Características da população  pacientes
A estruturanospodequais
ser serão transplantados.
feita de várias formas.Nesse
# pacientes terminais  Taxas de morte natural e acidental caso, alguns
Todavia, fatores de demanda
é interessante podemconsidere
que o candidato ser
relevantes.
o que pode ser feito com o produto, como
monetizar e algumas dificuldades envolvidas para
realizar a operação.
 Conscientização sobre registro
% pacientes terminais  Caso
Guia parao o Entrevistador
candidato não tenha feito perguntas
 Método de registro  Essa pergunta pode ser estruturada
registrados suficientes sobre o produto na etapa diretamente
inicial, é
 Motivos pessoais (religião etc.) na forma de uma equação. Caso o candidato siga
interessante que sejam adicionadas no framework.
 essa
Deixarcaminho,
o candidatolembre-o de apresentar
priorizar o caminho.os drivers
Caso não
para cada
priorize as termo da equação.
possíveis ações que podem ser
 Conscientização das famílias  Caso o candidato
realizadas, forçar onão o faça da
caminho proativamente,
resolução para
Doação de órgãos % pacientes não registrados
com consentimento da família  Informações passadas pelos médicos instigue-o
esta parte adofalar quais fatores podem ser
case.
 Motivos pessoais (religião etc.) influenciados (mudados) pela Comissão de
Saúde, isto é, a quantidade de pacientes
registrados e a taxa de famílias que dão seu
 Causa da morte consentimento.
# órgãos por paciente
 Estado do paciente antes da morte

 Disponibilidade do hospital
Limitadores de recebimento  Disponibilidade de quem irá receber
dos órgãos
 Tempo e distância para chegar no hospital

Clube de Consultoria UFC


Fonte: Casebook Darden 2013 – Organ Donation Case
149
Feedback aqui!
Case 13: Organ Donation
2. Analysis: Sua equipe decidiu se concentrar em aumentar o número de doadores registrados e está especificamente interessada
em doações de rins. A Comissão de Saúde sabe que precisa de 9.200 rins por ano. Qual é a representatividade dos doadores
registrados em relação aos doadores para que 9.200 rins sejam doados em um ano?

Sugestão de Abordagem: Informações Adicionais (Caso solicitadas):


 População de Nova York é de 10M.
Oferta de rins = Demanda de rins (9,2K)  Porcentagem de pacientes terminais (doenças ou
acidentes) a cada ano é de 0,1%.
Oferta de rins  (Doadores registrados (DR) + Doadores não registrados que doam (DNR)) x 2  Porcentagem de famílias que dão consentimento
Doadores registrados  DR = Pacientes terminais x % DR = 10M x 0,001 x %DR é de 10%.
Doadores não registrados  DNR = Pacientes terminais x % famílias x % DNR = 10M x 0,001 x 0,1 x (1 - %DR)  Desconsidere possíveis doadores de rins ainda
em vida.
Oferta de rins = (10M x 0,001 x %DR + 10M x 0,001 x 0,1 x (1 - %DR)) x 2 = 9,2K  Pode-se assumir que não haverá perda de órgãos
10K x %DR + 1K – 1k x %DR = 4,6K
9K x %DR = 3,6K  %DR = 0,4 ou 40%
Guia para o Entrevistador
 Existem outras maneiras para determinar esse
Conclusão/Insights: valor. O que é importante é que o candidato tenha
O valor encontrado parece razoável, contudo poderia ser melhor buscar aumentar essa porcentagem já que mais da
uma abordagem estruturada e analítica para o
metade das doações vêm de casos não registrados e, portanto, estão sujeitas à decisão de familiares passando por
problema e não entre em pânico. O candidato
momentos deliciados. Além disso, esses casos não registrados podem passar por processos mais lentos,
deve conduzir o entrevistador ao longo dos
comprometendo a disponibilidade dos órgãos.
cálculos e enquanto resolve a equação. Se o
candidato tiver dúvidas, dê dicas apropriadas.
 Cada doador pode doar dois rins, logo, no cálculo
da oferta, o candidato deve lembrar de multiplicar
o número de doadores por 2 para igualar ao
número de rins.

Clube de Consultoria UFC


Fonte: Casebook Darden 2013 – Organ Donation Case
150
Feedback aqui!
Case 13: Organ Donation
3. Framework: O que a Comissão de Saúde deve considerar fazer para aumentar o número de doadores registrados?

Sugestão de Abordagem:

 Quantificar e segmentar os doadores registrados


 Levantar campanhas atuais de incentivam o registro
Interno  Levantar métodos atuais de registro de registro
 Analisar da capacidade de investimento e influência para novas campanhas

Situação atual

 Benchmark com outras campanhas públicas


Externo  Benchmark com outros estados e países sobre doação de órgãos

Aumento número de
registros

 Determinação da metodologia de comunicação para a conscientização


Campanhas de  Determinação de meios para realizar a conscientização (TV, online etc.)
conscientização
 Realização de teste de comunicação e de meios

Possíveis melhorias
Guia para o Entrevistador
 Ao fazer consultas médicas  Há outras formas de abordar essa pergunta. O
Meios de registro  Ao tirar documentos oficias importante é o candidato focar em atividades que
 Em eventos especiais podem de fato ser realizadas pela Comissão e
tentar se manter MECE.

Clube de Consultoria UFC


Fonte: Casebook Darden 2013 – Organ Donation Case
151
Feedback aqui!
Case 13: Organ Donation
4. Chart: A equipe conduziu uma segmentação dos doadores 5. Brainstorming: A equipe decidiu se concentrar em
registrados e seu gerente pediu para você tirar alguma aumentar a taxa de registro entre os hispânicos. O que
conclusões iniciais desses dados. O que você diria a ela? podemos fazer para aumentar os registros desse segmento?
Sugestão de Abordagem:  Campanhas com linguagem e
expressões comuns do segmento
O segmento Caucasiano tem o maior número de potenciais doadores, mas Conscientização  Adequação às crenças culturais
também já apresenta uma alta penetração (quase 50%)  Utilização de figuras (artistas etc.)
respeitadas pelo segmento
O segmento Afro-americano é o oposto, com a menor penetração e menor  Apresentação dos benefícios
potencial.
Meio de  Eventos e panfletos em bairros e
Campanhas
comunicação locais frequentados pelo segmento
O segmento Hispânico apresenta uma penetração um pouco maior que o Afro-
americano, mas tem um número maior de potencias doadores, percebido por sua  Divulgação em mídias e horários
população maior. mais utilizados pelo segmento

Incentivos  Diminuição de impostos/taxas


Conclusão:  Cupons de descontos
O segmento Hispânico apresenta a melhor oportunidade devido a baixa penetração
e maior população. Alguns riscos podem ser possíveis barreiras de linguagem ou
diferenças culturais.
Métodos de  Registro ao ir ao médico/hospitais
registro  Registro ao tirar/renovar visto
Guia para o Entrevistador
 Registro em eventos em comunidades hispânicas
 Apresenta o Chart 1 juntamente à pergunta.
 Caso o candidato não pergunte, explique que os dados estão em milhões de
Guia para o Entrevistador
pessoas.
 Essa pergunta deve testar a criatividade do candidato.
 Caso o candidato não comente proativamente, pergunte: qual segmento
 Se necessário, puxe o candidato a pensar em mais ideias perguntando “O que
representa a melhor oportunidade para aumentar a quantidade de
mais?”.
doadores registrados?

Clube de Consultoria UFC


Fonte: Casebook Darden 2013 – Organ Donation Case
152
Feedback aqui!
Case 13: Organ Donation
6. Recommendation: O chefe da Comissão de Saúde acabou de entrar e quer uma recomendação sobre o que ele deve fazer para
alcançar rapidamente os hispânicos. Qual é a sua recomendação?
Sugestão de Abordagem: Guia para o Entrevistador
 De acordo com a pergunta, a recomendação deve
ser baseada no brainstorming anterior que busca
Recomendação formas de alcançar os hispânicos. O candidato
A Comissão de Saúde deve lançar campanhas voltadas para a comunidade hispânicas nos seus locais e meios pode trazer alguns elementos de outras partes do
de comunicação mais utilizados, aplicando linguagens e expressões dessa comunidade e mostrando possíveis case, contato que eles sejam relevantes para
incentivos como redução de impostos, além de diversificar os meios por quais podem ser feito os registros. responder a pergunta que refere a como alcançar
a comunidade hispânica.
 Nesta recomendação o candidato não deve
apresentar todas suas sugestões do brainstorming
Motivos anterior, mas priorizá-las e justificar sua
1. As comunidades hispânicas possuem práticas e cresças culturais específicas que podem estar dificultando
priorização.
o alcance atualmente.
 É recomendado que o candidato apresente
2. A forma de registro apenas ao tirar a carteira de motorista pode estar limitando o alcance.
também os riscos envolvidos com suas sugestões
e os próximos passos que a Comissão de Saúde
deve seguir.
Riscos Próximos Passos
1. Crenças culturais ou religiosas que dificultem 1. Analisar a cultura das comunidades
ou impossibilitem o alcance dessa população. hispânicas, criar campanhas considerando
2. Custos inviáveis relacionados à criação de suas características específicas e realizar
incentivo testes.
2. Analisar implementação de incentivos

Clube de Consultoria UFC


Fonte: Casebook Darden 2013 – Organ Donation Case
153
Feedback aqui!
Case 13: Organ Donation
Chart 1: Segmentação de doadores registrados

Doadores
População (M)
Registrados (M)

Caucasianos 2,600 5,500

Afro-Americanos 0,075 1,500

Hispânicos 0,250 2,500

Outros 0,100 0,500

Clube de Consultoria UFC


Fonte: Casebook Darden 2013 – Organ Donation Case
154
Feedback aqui!
6. Ambíguo

Public Sector / Operations


Indústria: Aeroespacial / Outros

Led By: Interviewer

Quantitativo Qualitativo

Case 14
SpaceX – Terra Nova
Clube de Consultoria UFC
Fonte: Casebook Duke 2015 – Mission Eternity
155
Case 14: SpaceX – Terra Nova
Introdução
Contexto: Estamos em 2050 e nosso cliente é a SpaceX. Recentemente, foi descoberto um asteroide gigante que está vindo em direção à Terra.

Situação: Alguns cientistas determinaram que existe uma probabilidade de 10% do asteroide colidir com a terra. Caso isto ocorra, o asteroide criará uma grande tempestade de
poeira bloqueando a luz solar e impedindo a fotossíntese nas plantas. Isso acabará por destruir o resto do ecossistema da Terra pelos próximos 100 anos.

Problema: A SpaceX precisa de ajuda para determinar se eles devem lançar a Terra Nova, uma missão de exploração espacial para encontrar um planeta adequado para a
população humana viver no caso do impacto catastrófico de um asteroide.

1. Framework: Quais pontos você consideraria para a missão Terra nova ser um sucesso?
Clarify (Informações adicionais, caso solicitadas)
 O objetivo da SpaceX é de encontrar um planeta habitável para evacuar no mínimo 20% da população mundial, caso ocorra o impacto do asteroide.
 O asteroide atingiria a Terra em 25 anos. A SpaceX e outras organizações envolvidas estimam que serão necessários 15 anos para construir veículos espaciais
suficientes e evacuar, com segurança, 20% da população mundial. Portanto, a missão Terra Nova precisa ser concluída em 10 anos.
 Outras organizações, como a Nasa, não seguiram a evolução tecnológica dos últimos anos e foram incorporadas pela SpaceX (desconsiderar outras entidades).
 As organizações mundiais analisaram outras opções para salvar a população da Terra, como atingir o asteroide para quebrá-lo em pedaços menores, e determinaram que
isso não é possível. Além disso, outros esforços para viver no subsolo estão sendo pesquisados ​separadamente.
Guia para o Entrevistador
 Esse case irá demandar que o candidato realize várias questões de clarify, com o intuito de entender, perfeitamente, a situação apresentada. Caso sejam levantadas
indagações relevantes que não são necessárias para este case, contar como um ponto positivo, mas pedir para o candidato desconsiderar.

Mandou bem! Pisou na bola...

 Priorizar a distância da terra e, consequentemente, o tempo como fatores  Focar em fatores relacionados após a missão. Ex: governança após encontrar o
limitantes da escolha do planeta. planeta. (podem ser abordados, mas não como tópicos principais).
 Avaliar a visão da sociedade em relação à missão.  Não considerar recursos necessários para o sucesso da missão.
 Não considerar viabilidade da vida humana nos potenciais planetas.

Clube de Consultoria UFC


Fonte: Casebook Duke 2015 – Mission Eternity
156
Feedback aqui!
Case 14: SpaceX – Terra Nova
1. Framework: Quais pontos você consideraria para a missão Terra nova ser um sucesso?

Sugestão de Abordagem: Informações Adicionais (Caso solicitadas):


 SpaceX teve grandes avanços em viagens
espaciais e construiu espaçonaves que viajam na
velocidade da luz.
 Potenciais planetas
 Devido aos avanços em viagens espaciais,
1. Possíveis  Principais métricas da viabilidade de vida humana prosperar
cientistas desenvolveram máquinas de hipotermia
planetas  Confiabilidade dos dados dos planetas
terapêutica para hibernação, com o intuito de
 Riscos futuros (outro asteroide, terremotos etc)
viabilizar viagens de longas distâncias no espaço.
 Todos os governos do planeta se reuniram para
contribuir financeiramente com a missão. Assim,
2. Recursos  Custos envolvidos (espaçonaves, equipe, infraestrutura etc) limitações de orçamento podem ser
envolvidos  Espaçonaves com tecnologia suficiente (velocidade, resistência, comunicação etc) desconsideradas.

Guia para o Entrevistador:


 Vários tópicos podem ser priorizados para a
 Sobrevivência do humano durante a viagem (alimentação, hibernação etc) resolução do case. Solicitar que o candidato
3. Implementação
 Distância dos planetas (limitações de tempo) priorize 1 ou 2 pontos, informando as principais
da missão
 Forma de analisar a capacidade da vida humana prosperar hipóteses e os dados necessários para validá-las.
 Guiar o candidato para o chart 1, após a
realização das follow-up questions.

4. Riscos  Confrontos entre governos (distribuição dos gastos, por exemplo)


 Perda de credibilidade em caso da falha da missão

Clube de Consultoria UFC


Fonte: Casebook Duke 2015 – Mission Eternity
157
Feedback aqui!
Case 14: SpaceX – Terra Nova
2. Chart 1: Esses são os 4 potenciais planetas para a 3. Analyses: É possível realizar a missão Terra Nova em 10
realização da missão Terra Nova. O que podemos concluir anos para todos os planetas?
com este gráfico?
Sugestão de Abordagem:
Pontos principais do gráfico:
1. A distância entre a Terra e os potenciais planetas podem influenciar na
escolha final devido ao fator tempo. Distância
Distância Coleta de Envio do Tempo
da Terra
2. O tamanho dos planetas pode ser um fator determinante devido à capacidade Planeta da Terra dados sinais Total
(trilhões
de pessoas que ele pode suportar. (ano-luz) (anos) (anos) (anos)
de Km)
Informações Adicionais (Caso solicitadas): Xin 38 4 1,5 1 6,5
 O tamanho dos planetas são proporcionais ao tamanho da terra no chart 1.
 Assim que a espaçonave chegar a um planeta, levará em média 18 meses Zeta 52,25 5,5 1,5 1,375 8,375
terrestres para coletar dados suficientes para determinar se o planeta pode
suportar vida humana. Yale 57 6 1,5 1,5 9
 A SpaceX também descobriu uma maneira de enviar sinais que transportam os
dados de volta à Terra mais rápido do que a velocidade da luz. Levará três Wocta 76 8 1,5 2 11,5
meses por ano-luz de distância para que os sinais cheguem à Terra.

Guia para o Entrevistador


 Caso o candidato não tenha perguntado em relação às limitações de tempo, Guia para o Entrevistador
informá-lo após compartilhar o chart 1.  Pode-se calcular o tempo de envio dos sinais dividindo o tempo da viagem por
 Informar também: velocidade da aeronave e % da população evacuada. 4, visto que eles são 4x mais rápidos que a velocidade da espaçonave.
 O candidato deve perceber que a distância está relacionada ao tempo e terá  O candidato deve determinar que o Planeta Wocta não retornará informações a
impacto na decisão de explorar certos planetas dentro do período de tempo tempo para a SpaceX construir a espaçonave de evacuação e, portanto, deve
alocado de 10 anos. Deve-se encontrar o tempo necessário para realizar a ser eliminado da consideração.
missão em cada planeta, somando o tempo da viagem (ano-luz) com os tempos  O candidato deve conduzir o caso e solicitar métricas adicionais em relação à
de coleta de dados e envio dos sinais (guiar o candidato, se necessário). água, condições atmosféricas, etc. com o intuito de determinar qual planeta
seria mais adequado para sustentar a vida.
Clube de Consultoria UFC
Fonte: Casebook Duke 2015 – Mission Eternity
158
Feedback aqui!
Case 14: SpaceX – Terra Nova
4. Brainstorming: Como a SpaceX deve determinar qual 5. Chart 2: SpaceX possui mão de obra e recursos suficientes
planeta explorar primeiro? Que métricas você gostaria de apenas para visitar um planeta. De acordo com o chart 2, qual
observar ao comparar os 3 planetas restantes? o melhor planeta?

 Viabilidade de transferir população com pouco risco


Pontos principais do gráfico:
Transporte 1. Planeta Yale é o planeta com o melhor IST.
 Tempo de transporte
2. Planeta Yale é o maior e possui maior porcentagem de zona habitável.
3. Plana Xin deve ser excluído da análise, visto que é menor e possui baixa
 Solo/ambiente para plantio porcentagem de zona habitável (hipótese de que não conseguiria suportar
Alimentação  Água potável 20% da população da Terra.
 Capacidade de criar animais

Condições  Qualidade do ar (utilizar máscaras?)


atmosféricas e  Presença de gases tóxicos
climáticas  Temperatura
Informações Adicionais (Caso solicitadas):
 Comprimento do dia/noite
 Em astronomia, a zona habitável é a região ao redor de uma estrela dentro da
 Energia (luz, combustíveis) qual objetos de massa planetária com pressão atmosférica suficiente podem
Recursos naturais  Força gravitacional conter água líquida em suas superfícies. Em outras palavras, isso indica a
 Terreno para construir edificações probabilidade de água.
 O Índice de Similaridade da Terra (IST) é uma medida de quão fisicamente
 Potencial de desenvolvimento de doenças semelhante um objeto de massa planetária é à Terra. É uma escala de zero a
Outros riscos  Possíveis predadores locais um, com a Terra tendo o valor de um.
 Risco de catástrofes naturais (terremotos, vulcão, etc
Guia para o Entrevistador
Guia para o Entrevistador  Após o candidato concluir que o Planeta Yale é o que possui os melhores
 O candidato deve conduzir o caso e se concentrar em priorizar uma ou duas indicadores para a missão Terra Nova, solicitar uma recomendação.
métricas principais, solicitando se existem dados disponíveis. Em seguida,
fornecer o chart 2.

Clube de Consultoria UFC


Fonte: Casebook Duke 2015 – Mission Eternity
159
Feedback aqui!
Case 14: SpaceX – Terra Nova
6. Recommendation: Vamos ter uma reunião com os diretores e os principais membros do conselho da SpaceX, com a
participação exclusiva do fundador da empresa: Elon Musk. Quais conclusões podemos apresentar?
Sugestão de Abordagem: Guia para o Entrevistador
 Apesar de existirem outros dados para se
considerar, o candidato deve priorizar o planeta
Recomendação Yale na recomendação devido às análises
A SpaceX deve realizar a Missão nova Terra para o planeta Yale.
realizadas.
 Na recomendação, considerações de destruir o
asteroide no futuro, devido à evolução da
tecnologia, são bem-vindas, assim como a
Motivos possibilidade de viver no subsolo terrestre.
1. Porque os dados relacionados a viabilidade do planeta serão retornados antes do prazo de 10 anos.
2. Esse planeta possui os maiores indicadores para sustentar a vida humana, com um IST de 0,8.
3. Ele possui a maior quantidade de zona habitável, por ser o maior planeta e apresentar o maior percentual
de zona habitável: 80%, sendo, assim, possível suportar 20% da população terrestre.

Riscos Próximos Passos


1. O planeta Yale é o mais distante da Terra, 1. Preparar a missão para o Planeta Yale
tornando a viagem e a hibernação mais 2. Dependendo dos recursos disponíveis, lançar
arriscadas. outra missão para o Planeta Zeta em até 4,5
2. É, possivelmente, mais custoso devido o anos, pois os dados retornariam em tempo
maior tempo de viagem. suficiente para evacuar a população. Além
disso, ele possui bons indicadores de zona
habitável e IST.
Clube de Consultoria UFC
Fonte: Casebook Duke 2015 – Mission Eternity
160
Feedback aqui!
Case 14: SpaceX – Terra Nova
Chart 1: Distância dos planetas da Terra

Terra Xin Zeta Yale Wocta

38 Trilhões de Km

52,25 Trilhões de Km

57 Trilhões de Km

76 Trilhões de Km

*Ano-luz é o termo utilizado por astrônomos para medir distâncias no espaço. É definido pela distância que um feixe de luz viaja em 1 ano terrestre.
* 1 ano-luz = 9,5 trilhões de quilômetros.
Clube de Consultoria UFC
Fonte: Casebook Duke 2015 – Mission Eternity
161
Feedback aqui!
Case 14: SpaceX – Terra Nova
Chart 2: Características dos planetas

100
90
Percentual de zona habitável

80
70 Yale
Zeta
60
50
40
30
20
Xin
10
0
0 0,1 0,2 0,3 0,4 0,5 0,6 0,7 0,8 0,9 1
Índice de Similaridade com a Terra (IST)

Clube de Consultoria UFC


Fonte: Casebook Duke 2015 – Mission Eternity
162
Feedback aqui!
6. Ambíguo

Valuation / Pricing / Product Launch


Indústria: Automotiva

Led By: Interviewer

Quantitativo Qualitativo

Case 15
The Moto Perpetuo
Clube de Consultoria UFC 163
Case 15: The Moto Perpetuo
Introdução
Contexto: Você está trabalhando como consultor após 1 ano de formatura. Um conhecido seu muito nerd da engenharia mecânica da sua universidade bate na porta da sua
casa às 2 da manhã para te mostrar um produto muito interessante.

Situação: Após uma série de pesquisas e experimentos, ele criou um motor que gera energia infinita: vamos chama-lo de Moto Perpetuo. Apesar de ser muito inteligente, seu
amigo é muito introspectivo e você foi a pessoa que ele teve mais contato na universidade. Então, você foi a primeira pessoa que ele entrou em contato para informar sobre a
descoberta, pois ele não conhecia outra pessoa confiável que poderia ajudá-lo.

Problema: A ideia do seu amigo é ganhar dinheiro com esse produto e gostaria que você montasse um plano do que deveria ser feito. Como recompensa, ele pretende dividir
os lucros desse negócio meio a meio com você.

1. Framework: Como seria o seu plano nesta situação? Quais os principais pontos considerar para sabermos o que fazer?
Clarify (Informações adicionais, caso solicitadas)
 Não considerar período de pandemia.
 Objetivo do amigo nerd é apenas financeiro.
 Sobre o produto: 1. Tamanho pode variar de acordo com a necessidade de energia. 2. Gera energia utilizando força da própria gravidade (tópico não relevante).
3. Durabilidade alta de 10 a 20 anos (semelhante a um motor comum). 4. Produção semelhante a um motor comum.
 Pode-se considerar que teremos a patente desse produto. Para simplificar o case, considerar apenas a patente para a região do Brasil.

Guia para o Entrevistador


 Esse é um case difícil e ambíguo. O candidato deve ser guiado no case (interviewer-led), porém iniciantes talvez não consigam apresentar os insights necessários para
garantir um bom fluxo de resolução.

Mandou bem! Pisou na bola...

 Considerar as ações que podem ser feitas e priorizar uma possível venda da  Não considerar quais mercados são relevantes para este produto.
patente, por ser uma forma rápida de ganhar dinheiro.  Não adicionar tópicos financeiros na estrutura.
 Abordar riscos e dificuldades de produzir o Moto Perpetuo.  Iniciar o framework sem entender sobre o produto.

Clube de Consultoria UFC 164


Feedback aqui!
Case 15: The Moto Perpetuo
1. Framework: Como seria o seu plano nesta situação? Quais os principais pontos considerar para sabermos o que fazer?

Sugestão de Abordagem: Guia para o Entrevistador

 A estrutura pode ser feita de várias formas.


Todavia, é interessante que o candidato considere
 Vender patente
o que pode ser feito com o produto, como
1. Ações  Iniciar negócio do zero
monetizar e algumas dificuldades envolvidas para
 Realizar parcerias
realizar a operação.
 Caso o candidato não tenha feito perguntas
suficientes sobre o produto na etapa inicial, é
 Investimentos e estrutura de custos necessária interessante que sejam adicionadas no framework.
2. Produto/  Deixar o candidato priorizar o caminho. Caso não
 Disponibilidade de matéria prima e tecnologia para escalar
operação  Efeitos prejudiciais do produto. Ex: explosão, sustentabilidade priorize as possíveis ações que podem ser
realizadas, forçar o caminho da resolução para
esta parte do case.
 Principais mercados: Indústrias, empresas de geração de energia, etc
 Tamanho e crescimento
3. Mercado  Precificação: valor agregado do produto
 Disponibilidade financeira e poder de barganha dos compradores

 Governo e legislação
4. Riscos  Risco de ser morto (criação de algo revolucionador)
 Reação de possíveis empresas que seriam prejudicadas

Clube de Consultoria UFC 165


Feedback aqui!
Case 15: The Moto Perpetuo
2. Brainstorming: Quais as principais ações que seu amigo 3. Brainstorming: Iremos vender a patente. Quais as
pode fazer junto com você? Quais os prós e os contras de principais indústrias ou mercados que poderiam estar
cada ação? interessados em comprar o nosso produto?

 Hidráulica
Prós Contras  Carvão
1. Geram energia  Nuclear
 Retorno rápido  Potencial de retorno
 Renováveis e outras
 Não precisar operar menor
Vender Patente
 Reconhecimento pela
criação  Automóveis/veículos
2. Dependem de  Eletrodomésticos
 Maior potencial de retorno  Dificuldade de levantar energia  Aviões
 Reconhecimento pela dinheiro para execução
criação e execução  Possível falta de
Executar sem expertise  Grandes fábricas: siderúrgicas, refinarias
ajuda  Dificuldades 3. Utilizam muita  Extração de combustíveis: petróleo e mineração
operacionais  Espacial: foguetes
 Confiabilidade dos
energia
 Autogeração: residências e comércios
stakeholders

 Potencial de retorno maior  Dificuldade de Guia para o Entrevistador


que vender a patente relacionamento com  O candidato pode desdobrar o brainstorming de várias formas, porém é
Executar com importante que, ao final, ele realize uma priorização lógica para a venda da
 Ajuda operacional e possíveis
parceria/sócios patente devido, principalmente, às dificuldades de operar.
estratégica de possíveis parceiros/sócios
parceiros  Guiar o candidato, caso necessário, para a realização da venda da patente para
a indústria automobilística. Caso o candidato esteja com uma boa
performance até esta parte do case, perguntar as vantagens dessa indústria.

Clube de Consultoria UFC 166


Feedback aqui!
Case 15: The Moto Perpetuo
4. Estimation: Por quanto devemos vender a patente para um grande cliente do setor automobilístico?

Sugestão de Abordagem: Informações Adicionais (Caso solicitadas):


 Não haverá investimentos ou custos incrementais
Guia para o Entrevistador: Tamanho do mercado na venda de cada carro. Toda a infraestrutura das
600K 3M
 Sugestão de cálculo do tamanho do (volume)* atuais fábricas serão aproveitadas. O custo do
mercado em volume e da receita extra # de carros vendidos atual motor dos carros será o mesmo do Moto
por carro na página posterior. anualmente Perpetuo.
 O comprador possui 20% de market-share.
 Taxa de desconto: 15% (alto risco).
12B  Considerar apenas carros.

Lucro anual Market-share 20%

80B
Guia para o Entrevistador
Valuation (R$)  Essa é a parte mais difícil do case: Entender qual
Receita extra será o valor da patente para o setor
20K automobilístico.
20K por carro*
 A lógica é de precificação da patente por meio do
Lucro incremental por valor agregado que o produto tem na economia de
carro combustível.
 A receita extra será o quanto o comprador da
patente poderá cobrar a mais em cada carro. Esse
Taxa de desconto 15% Custos extras
0,00 valor será o quanto o proprietário do veículo
por carro deixará de gastar com combustível.
 O candidato deverá realizar todas as assumptions
necessárias para encontrar o valor final.

Clube de Consultoria UFC 167


Feedback aqui!
Case 15: The Moto Perpetuo
4. Estimation: Por quanto devemos vender a patente para um grande cliente do setor automobilístico?

Sugestão de Abordagem:
45M # de carros/ família 1.5 Guia para o Entrevistador
 É importante que o candidato
3M # de carros hoje # famílias no Brasil 200M/4 = 50M realize todas as assumptions,
30M mas que possua agilidade na
Substituição # de famílias com realização dos cálculos.
carro  Não considerar crescimento do
Vida útil (carro
3M para de funcionar) 15 anos % de possui carro 60% mercado com o intuito de facilitar
cálculos.
Tamanho do
mercado (volume)*  O case finaliza após encontrar o
0 (simplificar) # de carros hoje ~ valuation, sendo a criação de
uma recomendação algo
Crescimento opcional

% de crescimento Crescimento do PIB


0%
Crescimento populacional

5k 5,00 Guia para o Entrevistador


Custo/litro
 Os 4 anos de economia de combustível está considerando
20K Redução de custos que o proprietário do veículo terá seu break even com 4
com combustível Km rodados/ano
1000 10k anos. Podem-se utilizar outras abordagens, mas que
Receita extra por considerem a vantagem para o cliente final.
carro* (R$)
# de litros  O # de Km rodados por ano pode ser desdobrado na
4
quantidade média de km rodados diariamente.
 O case finaliza após encontrar o valuation, sendo a criação
# de anos Autonomia Km/l 10
de uma recomendação algo opcional.
Clube de Consultoria UFC 168
Feedback aqui!
Equipe do Projeto: Casebook CCUFC
Nome do Case Criador(a) Equipe de Validação

Manhattan Boom! Walter Viana Lucas Barbosa e Letícia Brício

Easy Money Walter Viana Lucas Barbosa e Gabriel Melo

La Gatta Gelato Tamara Medeiros Levi Dantas e Gabriela Kogan

Vale do Rio Doce Walter Viana Lucas Barbosa e Levi Dantas


Data de criação – Versão 1: 14/06/2021
Este casebook apresenta a contribuição de forma direta e indireta de Enfermeiro Anjo Walter Viana Gabriela Kogan e Gabriel Melo
todos os membros do ano de 2020 e de alguns do primeiro semestre
de 2021. Em especial, podemos destacar, logo abaixo, a equipe oficial Tele Fortal Walter Viana Mariana Alves e Gabriela Kogan
do projeto e, a direita, a equipe de suporte/validação do material:
Cards for Everyone Tamara Medeiros Levi Dantas e Gabriel Melo
Líder do projeto
 Walter Viana – Eng. De Produção UFC - 2020.2 Where’s the Party Tamara Medeiros Gabriela Kogan e Walter Viana

Safe Car Tamara Medeiros Letícia Brício e Levi Dantas


Equipe principal
 Tamara Medeiros – Eng. Energias Renováveis UFC - 2020.2 Greta Coffehouse Walter Viana Lucas Barbosa e Gabriel Melo
 Mariana Alves – Eng. Química UFC - 2021.2
 Lucas Barbosa – Eng. Química UFC – 2021.2 BR Rodovias Walter Viana Levi Dantas e Gabriela Kogan
 Danilo Siqueira – Eng. Civil UFC – 2022.2
Mobi Clean Walter Viana Lucas Barbosa e Letícia Brício
Design:
Organ Donation Tamara Medeiros Lucas Barbosa e Gabriel Melo
 Valéria Rocha – Design de Moda UFC - 2020.2
SpaceX – Terra Nova Walter Viana Gabriel Melo e Letícia Brício

The Moto Perpetuo Walter Viana Levi Dantas e Ian Guerreiro*

Clube de Consultoria UFC 169


Acompanhe Nossas
Redes Sociais
@clubedeconsultoria_ufc

Clube de Consultoria UFC

Clube de Consultoria UFC

clubedeconsultoria_ufc@gmail.com

“juntos vamos mais longe”

170

Você também pode gostar